You are on page 1of 101

Hướng tới Olympic Toán Việt Nam

UBND TỈNH BẮC NINH ĐỀ THI CHỌN ĐỘI TUYỂN


SỞ GIÁO DỤC VÀ ĐÀO TẠO DỰ THI HỌC SINH GIỎI QUỐC GIA NĂM 2020
¯¯¯¯¯¯¯¯¯¯¯¯¯¯¯¯¯¯¯¯¯¯ Môn thi: Toán
ĐỀ CHÍNH THỨC Thời gian làm bài: 180 phút (không kể thời gian giao đề)
(Đề thi có 01 trang) Ngày thi thứ nhất: 24/9/2019
¯¯¯¯¯¯¯¯¯¯¯¯¯¯¯¯¯¯¯¯¯¯¯¯¯¯¯¯¯¯¯¯¯¯¯¯¯¯¯¯¯¯

Câu 1 (5,0 điểm)


Cho hai dãy số (un ), (vn ) xác định như sau u 0  a; v 0  b với hằng số thực a, b cho

un  vn
trước thỏa mãn 0  a  b và un 1  , vn 1  un 1.vn với mọi số tự nhiên n .
2
a) Chứng tỏ hai dãy đã cho đều hội tụ và có giới hạn bằng nhau.
b) Tìm giới hạn đó theo a, b .

Câu 2 (5,0 điểm)


Cho số nguyên tố p . Chứng minh rằng tồn tại vô số số tự nhiên n thỏa mãn điều kiện

2020n 2019  n  2018 (mod p) .

Câu 3 (5,0 điểm)


Cho tam giác nhọn ABC không cân. Gọi H ,O lần lượt là trực tâm, tâm đường tròn

ngoại tiếp tam giác ABC ; D, E lần lượt là chân đường cao hạ từ các đỉnh A, B của tam

giác ABC . Các đường thẳng OD và BE cắt nhau tại K , các đường thẳng OE và AD cắt
nhau tại L. Gọi M là trung điểm cạnh AB. Chứng minh ba điểm K , L, M thẳng hàng khi và

chỉ khi bốn điểm C , D,O, H cùng nằm trên một đường tròn.
Câu 4 (5,0 điểm)

 
Tìm tất cả các đa thức f x có hệ số thực và bậc là số tự nhiên lẻ sao cho:

   
f x 2  1  f 2 x  1, x  .

------------ Hết ------------


Thí sinh không được sử dụng tài liệu và máy tính cầm tay. Cán bộ coi thi không giải thích gì thêm.
Họ và tên thí sinh:.....................................................................Số báo danh :…………...

1
UBND TỈNH BẮC NINH HƯỚNG DẪN CHẤM
SỞ GIÁO DỤC VÀ ĐÀO TẠO ĐỀ CHỌN ĐỘI TUYỂN
DỰ THI HỌC SINH GIỎI QUỐC GIA NĂM 2020
Môn thi: Toán
(Hướng dẫn chấm có 04 trang)

Câu Đáp án Điểm


Cho 2 dãy số (un ) , (vn ) xác định như sau:
u0 a; v 0 b với hằng số thực a, b cho trước thỏa mãn 0 a b và
1.a un vn 2,0
un 1
, vn 1
un 1.vn với mọi số tự nhiên n .
2
a) Chứng tỏ hai dãy đã cho đều hội tụ và có giới hạn bằng nhau.

Ta chứng minh quy nạp rằng un un 1


vn và un 1
vn 1
vn với mọi n .
1,0
Do đó, 2 dãy đã cho là đơn điệu và bị chặn bởi u 0 a; v 0 b nên hội tụ.
un vn lim un lim vn
Từ un 1
, cho qua giới hạn ta được lim un 1
hay
2 2 1,0
lim un lim vn (đpcm).
1.b b) Tìm giới hạn đó theo a, b . 3,0
a
Do 0 a b nên đặt cos với (0; ) . Ta chứng minh rằng
b 2
1,0
2
un b cos .cos ...cos .cos
21 22 2n 1
2n

Và vn b cos .cos ...cos với mọi số nguyên dương n .


21 22 2n 1,0

b sin b sin
Từ đó rút gọn biểu thức ta được vn n
.
n
2 sin
2n
1,0
b2 a2
Vậy lim un lim vn .
a
arccos
b
Cho số nguyên tố p . Chứng minh rằng, tồn tại vô số số tự nhiên n thỏa mãn điều
2 kiện 2020n 2019 n 2018(mod p) . 5,0

Ta xét 2 trường hợp.


Trường hợp 1. Nếu p là ước nguyên tố của 2020, khi đó, chỉ cần chọn
n 2018 p là thỏa mãn. 1,0
Việc này chứng tỏ tìm được vô số n .

Trường hợp 2. Nếu p không là ước nguyên tố của 2020, khi đó (p, 2020) 1.
n 2019 0(mod p 1) 2,0
Chọn
n 2018 1(mod p)

2
Theo định lí Fecma a p 1
1(mod p) ta được
2020p 1
1(mod p) 2020n 2019
1(mod p) 1,0
nên 2020 n 2018(mod p) (thỏa mãn đề bài).
n 2019

Lại có, theo định lí Trung Hoa về dư thì hệ phương trình đồng dư luôn có nghiệm
1,0
n vì p, p 1 1 nên có vô số tự nhiên n thỏa mãn. (đpcm)
Cho tam giác nhọn ABC không cân. Gọi H , O lần lượt là trực tâm, tâm đường
tròn ngoại tiếp tam giác ABC ; D, E lần lượt là chân đường cao hạ từ các đỉnh
A, B của tam giác ABC . Các đường thẳng OD và BE cắt nhau tại K , các
3 5,0
đường thẳng OE và AD cắt nhau tại L . Gọi M là trung điểm cạnh AB . Chứng
minh ba điểm K , L, M thẳng hàng khi và chỉ khi bốn điểm C , D,O, H cùng nằm
trên một đường tròn.
C

x
Áp dụng định lí Menelaus
cho tam giác HAB và ba
Q
D
điểm K , L, M ta có:
P K , L, M thẳng hàng khi và
L KB LH MA 0,5
E O chỉ khi . . 1
K KH LA MB
H
KB LA
(1)
KH LH
A B
M

KB SBOD LA SAOE
Ta lại có (cùng cạnh đáy OD), (cùng cạnh đáy OE) và
KH S HOD LH S HOE
gọi R là bán kính đường tròn ngoại tiếp tam giác ABC và c AB thì
1 1 1
SAOE AE .d(O, AE ) c.cos A.R .cos B R.c.cos A.cos B 1,0
2 2 2
1
Tương tự SBOD R.c.cos A.cos B
2
Nên S AOE S BOD .
Từ các kết quả trên ta có (1) S HOD S HOE khi và chỉ khi OH || DE (nếu H và
O cùng phía đối với DE ) hoặc OH đi qua trung điểm ED (nếu H và O khác 0,5
phía đối với DE ).
Trước hết, vẽ tiếp tuyến C x của đường tròn ngoại tiếp tam giác ABC tại C , dễ
dàng chứng minh CED  ABC  ACx suy ra DE || Cx từ đó dẫn đến 0,5
CO vuông góc với DE (2).

3
Ta chứng minh (1) xảy ra khi chỉ khi OH || DE .
Thật vậy, nếu xảy ra trường hợp còn lại, tức là OH đi qua trung điểm ED .
Khi đó, gọi P,Q lần lượt là trung điểm của ED , HC . Dễ thấy tứ giác CEHD nội
tiếp đường tròn tâm Q , suy ra QP vuông góc với ED . Kết hợp (2) suy ra
2,0
QP || CO .
Xét tam giác CHO có Q là trung điểm HC và QP || CO suy ra P là trung điểm
OH nên EHDO là hình bình hành, suy ra OD || EH . Điều này trái với giả thiết
OD cắt BE .
Vậy (1) xảy ra khi và chỉ khi OH || DE , mà do (1) nên điều này khi và chỉ khi
0,5
CO OH khi và chỉ khi C , D,O, H cùng nằm trên một đường tròn.

Tìm tất cả các đa thức f x hệ số thực, có bậc là số tự nhiên lẻ sao cho


4 5,0
f x2 1 f2 x 1, x

2
Thay x bằng x ta có f 2 x 1 f x 1 f x2 1 f2 x 1

Suy ra f 2 x f2 x
0,5
f ( x) f (x ) x A
Nên trong đó A B .
f ( x) f (x ) x B

Nếu tập A vô hạn hay phương trình f x f x 0 có vô số nghiệm mà bậc


của f là hữu hạn nên f ( x ) f (x ) 0 f x f x , x (1)

Lại có deg f là lẻ nên trong hai giới hạn lim f (x ) và lim f ( x ) có đúng một
x x

giới hạn là và một là , do đó tồn tại x 0 (đủ lớn) sao cho f x và f x


1,0
trái dấu (suy ra không bằng nhau) khi x x 0 điều này mâu thuẫn với (1) nên tập
A không thể là vô hạn.

Suy ra tập B là vô hạn hay phương trình f x f x 0 có vô số nghiệm mà


bậc của f là hữu hạn nên f x f x , x .

f 0 f 0 f 0 0
Chọn x 0 2
và f 1 1. 0,5
f 1 f 0 1 f 1 1

Xét dãy số a 0 1;an 1


an 1

Dễ thấy an 1, n 0

Ta chứng minh an 1
an , n 0 1 .
1,5
Thật vậy n 0 a1 2 1 a 0 bài toán đúng với n 1

Giả sử (1) đúng đến n , suy ra an 2


an 1
1 an 1 an 1 . Vậy (1) được
chứng minh.
4
Bây giờ ta chứng minh f an an , n 0 2 bằng quy nạp.

Với n 0 f a0 f 1 a 0 suy ra (2) đúng với n 0

Giả sử f an an ta chứng minh f an 1


an 1 , ta có

f 2 an 1
1 f a 2n 1
1 1 f an 1 an a 2n 1
1,0

f an 1
an 1
hoặc f an 1
an 1

Nếu f an 1
an 1
f 2 an 1
1 f a 2n 2
1 1 f an 1
1 an 1
0
(vô lí) do đó f an 1
an 1 .

Vậy (2) được chứng minh, do đó phương trình f x x có vô số nghiệm nên


f x x với mọi x R. 0,5

Thử lại ta thấy f x x thỏa mãn yêu cầu bài toán.

---------------Hết---------------

5
UBND TỈNH BẮC NINH ĐỀ THI CHỌN ĐỘI TUYỂN
SỞ GIÁO DỤC VÀ ĐÀO TẠO DỰ THI HỌC SINH GIỎI QUỐC GIA NĂM 2020
¯¯¯¯¯¯¯¯¯¯¯¯¯¯¯¯ Môn thi: Toán
ĐỀ CHÍNH THỨC Thời gian làm bài: 180 phút (không kể thời gian giao đề)
(Đề thi có 01 trang) Ngày thi thứ hai: 25/9/2019
¯¯¯¯¯¯¯¯¯¯¯¯¯¯¯¯¯¯¯¯¯¯¯¯¯¯¯¯¯¯¯¯¯¯¯¯¯¯¯¯¯¯

Câu 5 (7,0 điểm)


Tìm tất cả các hàm f :  thỏa mãn điều kiện:

f (xy  1)  f (x )f (y )  2xy  1 với mọi x , y  .

Câu 6 (7,0 điểm)


Cho tam giác nhọn ABC , D là một điểm bất kì trên cạnh BC . Trên cạnh AC , AB lần lượt
lấy các điểm E , F sao cho ED  EC , FD  FB . Gọi I , J , K lần lượt là tâm đường tròn nội tiếp

các tam giác ABC , BDF ,CDE .

a) Gọi H là trực tâm của tam giác JDK . Chứng minh rằng tứ giác IJHK nội tiếp.
b) Chứng minh rằng khi D chuyển động trên BC , đường tròn ngoại tiếp tam giác IJK luôn
đi qua một điểm cố định khác điểm I .
Câu 7 (6,0 điểm)

Cho một đa giác đều A1A2 ...A20 có 10 đỉnh của đa giác được tô màu xanh, 10 đỉnh còn lại

được tô màu đỏ. Ta nối các đỉnh với nhau.


a) Gọi a là số các đoạn thẳng nối hai đỉnh màu đỏ liên tiếp, b là số các đoạn thẳng nối hai
đỉnh màu xanh liên tiếp. Chứng minh a  b.

b) Xét tập hợp S gồm đường chéo A1A4 và tất cả các đường chéo khác của đa giác mà có

cùng độ dài với nó. Chứng minh trong tập hợp đó, số đường chéo có hai đầu là màu đỏ bằng với số
đường chéo có hai đầu là màu xanh. Gọi k là số đường chéo có hai đầu là màu xanh trong S , tìm
tất cả các giá trị có thể có của k .

------------ Hết ------------


Thí sinh không được sử dụng tài liệu và máy tính cầm tay. Cán bộ coi thi không giải thích gì thêm.
Họ và tên thí sinh:.....................................................................Số báo danh :…………...

1
UBND TỈNH BẮC NINH HƯỚNG DẪN CHẤM
SỞ GIÁO DỤC VÀ ĐÀO TẠO THI CHỌN ĐỘI TUYỂN
¯¯¯¯¯¯¯¯¯¯¯¯¯¯¯¯ DỰ THI HỌC SINH GIỎI QUỐC GIA NĂM 2020
(Đề thi có 04 trang) Môn thi: Toán
Ngày thi thứ hai: 25/9/2019
¯¯¯¯¯¯¯¯¯¯¯¯¯¯¯¯¯¯¯¯¯¯¯¯¯¯¯¯¯¯¯¯¯¯¯¯¯¯¯¯¯¯

Câu Đáp án Điểm


Tìm tất cả các hàm f : thảo mãn điều kiện:
5 f (xy 1) f (x )f (y ) 2xy 1 với mọi x , y . (1) 7,0

Thay x 0 vào (1), ta có f ( 1) f (x ).f (0) 1 với mọi x .


Nếu f (0) 0 thì f sẽ là hàm hằng. Thay vào (1) dễ thấy không thỏa mãn.
Vì vậy f (0) 0 .
1,0
2
Thay x y 1 vào phương trình (1) ta thu được f (1) 1 . Nghĩa là f (1) 1 hoặc
f (1) 1.
Trường hợp 1: f (1) 1 .
Thay x bởi xy và y 1 vào phương trình (1) ta thu được
f (xy 1) f (xy ) 2xy 1 với mọi x , y . 1,0
Kết hợp (1) ta thu được:
f (xy ) f (x )f (y ) với mọi x , y (2)
Tiếp theo ta thay y 1 vào phương trình (1) ta nhận được
f (x 1) f (x ) 2x 1 hay f (x 1) 2x 1 f (x ) với mọi x .(3)
Lại thay và x bởi x 1 và y 1 vào phương trình (1) ta nhận được
f (x 1) f (x ) 2x 1 hay f (x 1) 2x 1 f (x ) với mọi x .(4)
Cho y bởi x vào phương trình (1) và sử dụng (2) (3) và (4) ta nhận được 2,0
2 2 2 2
2x 1 f (x 1) f (x ) f (x 1).f (x 1) f (x )
(2x 1 f (x )).(2x 1 f (x )) f 2 (x ) 2f 2 x 4xf x 4x 2 1
2
Suy ra 2 f (x ) x 0 dẫn đến f (x ) x với mọi x .
Trường hợp 2: f 1 1.
Bằng cách thay tương tự như ở trường hợp 1, ta có:
f xy f x f y với mọi x , y . (5)
f x 1 2x 1 f (x ) với mọi x . (6)
f (x 1) 2x 1 f (x ) với mọi x . (7)
2,0
Tiếp theo, thay y bởi x vào phương trình (1) và sử dụng (5) (6) và (7) ta nhận được
2x 2 1 f (x 2 1) f 2 (x ) f (x 1).f (x 1) f 2 (x )
(2x 1 f (x )).( 2x 1 f (x )) f 2 (x )
2f x 4x 2 1
Suy ra f (x ) x 2 với mọi x .
Thử lại ta thấy cả hai hàm ở cả hai trường hợp đều thỏa mãn đề bài.
Vậy có hai hàm số thỏa mãn yêu cầu bài toán 1,0
f (x ) x với mọi x .

2
và f (x ) x 2 với mọi x .
Cho tam giác nhọn ABC , D là một điểm bất kì trên cạnh BC . Trên cạnh AC , AB lần lượt
lấy các điểm E , F sao cho ED EC , FD FB . Gọi I , J , K lần lượt là tâm đường tròn
nội tiếp các tam giác ABC , BDF ,CDE .
6 7,0
a) Gọi H là trực tâm của tam giác JDK . Chứng minh rằng IJHK nội tiếp.
b) Chứng minh rằng khi D chuyển động trên BC , đường tròn ngoại tiếp tam giác
IJK luôn đi qua một điểm cố định khác I .

Do các tam giác FBD, EDC lần lượt cân tại F , E nên JD JB, KD KC .
Ta có JDK 1800 JDB KDC 1800 IBC ICB BIC JIK
6.a Lại có H là trực tâm tam giác JDK nên JHK 180 0
JDK , 2,0
0
suy ra JHK JIK 180
Vậy tứ giác IJHK nội tiếp.
Trước hết ta chứng minh bổ đề sau (bài toán Simson đảo)
Cho tam giác MJK . Từ điểm L nằm ngoài tam giác. Gọi B,C , D lần lượt là điểm đối xứng
6.b của L qua MJ , MK , JK . Giả sử B,C , D thẳng hàng. Chứng minh rằng MJLK là tứ giác
nội tiếp.

3
Chứng minh bổ đề: Gọi giao của BL, DL,CL với MJ , JK , KM lần lượt là E ,G , H . Do
B,C , D thẳng hàng E ,G , H thẳng hàng. Tứ giác JGLE và GHKL nội tiếp
1,0
JLK ELH (g.g) JLK ELH mà ELH EMH 1800
JLK EMH 1800 MJLK nội tiếp (đpcm).
Quay trở lại bài toán:
Gọi M là điểm chính giữa cung BC của đường tròn (O ) ngoại tiếp tam giác ABC , L là
1,0
điểm đối xứng với D qua JK .
Ta có JIK JDK JLK suy ra L (IJK ) .
Mặt khác, JL JD JB, KL KD KC nên LJB 2 LDB LKC , suy ra
1,0
BLJ CLK .
Ta thu được
BLC JLK BIC JIK , suy ra L (BIC ) hay ML MB MC . 1,0
Suy ra JM , KM lần lượt là trung trực của LB, LC .
Vậy điểm đối xứng với L qua 3 cạnh tam giác JMK là B,C , D . Mà B,C , D thẳng hàng,
1,0
theo bổ đề trên suy ra LJMK nội tiếp. Vậy (IJK ) luôn đi qua điểm M cố định (đpcm).
Cho một đa giác đều A1A2 ...A20 có 10 đỉnh của đa giác được tô màu xanh, 10 đỉnh còn lại
được tô màu đỏ. Ta nối các đỉnh với nhau.
a) Gọi a là số các đoạn thẳng nối hai đỉnh đỏ liên tiếp, b là số các đoạn thẳng nối 6,0
hai đỉnh xanh liên tiếp. Chứng minh a b.
7
b) Xét tập hợp S gồm đường chéo A1A4 và tất cả các đường chéo khác của đa giác
mà có cùng độ dài với nó. Chứng minh trong tập hợp đó, số đường chéo có hai đầu là màu
đỏ bằng với số đường chéo có hai đầu là màu xanh. Gọi k là số đường chéo có hai đầu là
màu xanh trong S , tìm tất cả các giá trị có thể có của .
Ta chia dãy các đỉnh A1, A2 ,..., A20 thành các cụm đỏ và cụm xanh, trong đó:
 Cụm đỏ là cụm đỉnh gồm các đỉnh đỏ liên tiếp.
 Cụm xanh là cụm đỉnh gồm các đỉnh xanh liên tiếp.
Giả sử ta có n cụm đỉnh xanh là : X1, X 2 ,..., X n ( 1 n 10 )
7.a. và x 1, x 2 ,..., x n tương ứng là số đỉnh xanh trong các cụm đó ( x i 1 i 1, n ). 1,0

Nếu n 1 , tức là chỉ có một cụm xanh hay 10 đỉnh xanh cạnh nhau, do đó 10 đỉnh đỏ cũng
phải cạnh nhau, tức là cũng chỉ có một cụm đỏ.
Nếu n 2 thì mỗi cụm điểm xanh nằm giữa hai cụm đỉnh đỏ và ngược lại mỗi cụm đỉnh đỏ
lại nằm giữa hai cụm màu xanh nên số cụm đỉnh xanh sẽ bằng số cụm đỉnh đỏ bằng n .
Giả sử n cụm đỉnh đỏ là: D1, D2 ,..., Dn và d1, d2 ,..., dn tương ứng là số đỉnh đỏ trong các 1,0

4
cụm đó ( di 1 i 1, n ).
Do trong một cụm Xi có x i đỉnh xanh cạnh nhau nên có x i 1 cặp đỉnh xanh liên tiếp. Nên
a (x 1 1) (x 2 1) ... (x n 1) x1 ... xn n 10 n .
Tương tự b (d1 1) ... (dn 1) d1 ... dn n 10 n .
Vậy a b (đpcm).
Trước hết, ta xét dãy các đỉnh ( gồm tất cả 20 đỉnh):
A1 A4 A7 A10 A13 A16 A19
A2 A5 A8 A11 A14 A17 A20 (*)
7.b. A3 A6 A9 A12 A15 A18 A1 2,0
Các đường chéo nối 2 đỉnh liên tiếp trong dãy trên đều có cùng độ dài với A1A4 nên tất cả
đều thuộc S . Ngoài ra, dễ thấy rằng tất cả các đường chéo thuộc S cũng tạo thành bởi 2 đỉnh
liên tiếp nào đó trong dãy trên.
Tương tự câu a) ta chia dãy các đỉnh (*) thành các cụm đỏ và cụm xanh, trong đó:
 Cụm đỏ là cụm đỉnh gồm các đỉnh đỏ liên tiếp trong dãy đỉnh (*) .
 Cụm xanh là cụm đỉnh gồm các đỉnh xanh liên tiếp trong dãy đỉnh (*). 1,0
Và ta cũng thu được số cụm đỉnh xanh sẽ bằng số cụm đỉnh đỏ, giả sử bằng
m (1 m 10)
Ta có số đường chéo có hai đầu màu đỏ trong tập S cũng chính là số đoạn thẳng nối hai
đỉnh đỏ liên tiếp trong dãy (*) và số đường chéo có hai đầu màu xanh trong tập S cũng
chính là số đoạn thẳng nối hai đỉnh xanh liên tiếp trong dãy (*) . Nên theo câu a) số đường 1,0
chéo có hai đầu màu đỏ trong tập S sẽ bằng số đường chéo có hai đầu màu xanh trong tập
S và bằng k 10 m , do 1 m 10 nên k {0,1,..., 9} .

5
SỞ GIÁO DỤC VÀ ĐÀO TẠO BÌNH PHƯỚC KỲ THI LẬP ĐỘI TUYỂN
HỌC SINH GIỎI QUỐC GIA THPT NĂM 2020
ĐỀ CHÍNH THỨC MÔN THI: TOÁN
Thời gian: 180 phút (không kể thời gian giao đề)
Ngày thi: 22/10/2019
(Đề thi gồm 01 trang)
Câu 1. (5.0 điểm). Cho các số thực x, y  1, 2  . Chứng minh
2019
 x3  y 3 
x  xy  1   y  xy  1   3xy  2 
2019 2019

2 2 2019

 x y 
Câu 2. (5.0 điểm). Cho tam giác nhọn ABC nội tiếp đường tròn tâm  O  . Kẻ đường cao AH . Gọi M là
trung điểm của BC , AM cắt OH tại G . Chứng minh rằng G nằm trên trục đẳng phương của đường tròn
ngoại tiếp tam giác BOC và đường tròn Euler của tam giác ABC .
Câu 3. (5.0 điểm). Cho x, y là các số nguyên dương. Nếu với mọi số nguyên dương n ta đều có

 1 chia hết cho  ny   1 . Chứng minh x  1


 n 2
x

(Với   n  là số các số nguyên dương nhỏ hơn n và nguyên tố cùng nhau với n )

Câu 4. (5.0 điểm). Cho S là một bội nguyên dương của tất cả các số từ 2 đến 2019 và n số nguyên dương
a1 , a2 ,..., an thuộc M : 1, 2,..., 2019 có tổng bằng 2S. Chứng minh rằng ta có thể chọn ra một vài số trong n
số này mà có tổng bằng S.
HẾT
Lưu ý: Thí sinh không được sử dụng tài liệu và máy tính bỏ túi, giám thị coi thi không giải thích gì
thêm.
Họ và tên thí sinh:.....................................................Số báo danh:...................................................
Chữ ký của giám thị 1:..............................................Chữ ký của giám thị 2:....................................
SỞ GIÁO DỤC VÀ ĐÀO TẠO HƯỚNG DẪN CHẤM KÌ THI LẬP ĐỘI TUYỂN HỌC
BÌNH PHƯỚC SINH GIỎI QUỐC GIA THPT NĂM 2020
Hướng dẫn chấm gồm 04 trang MÔN THI: TOÁN

Điểm
Câu Nội dung

Cho các số thực x, y  1, 2  . Chứng minh


2019
 x3  y 3  5.0
x  xy  1   y  xy  1   3xy  2 
2019 2019

2 2 2019

 x y 

a  b  c  d

Bổ đề Cho a  b  c  d Chứng minh: a  b  c  d  với   1
a  b  0, c  d  0

(*)
 su  sv 1.0
 a   c 
a  b  s c  d  s
Đặt  
2
và  
2
 s, u , v  0 
a  b  u b  s  u c  d  v d  s  v
 2  2
1
(5.0đ) Ta được: a  b  u  v  c  d .
 *   s  u    s  u    s  v    s  v   f  u   f  v  (**)
   

Với f  t    t  s    s  t  , t   0; s 
 
2.0

Suy ra: f '  t     s  t 


  s  t    0 t   0; s  .Vậy (**) đúng
 1  1
 
Trở lại bài toán: Giả sử 2  x  y  1 . Đặt

a  x 2  xy  1, b  y 2  xy  1, c  3xy  2, d  x 2  xy  y 2 ; a  b  c  d và

a  b  x 2  y 2  0, c  d  4 xy  2  x 2  y 2  x  2 y  x   y  2 x  y   2  0
2.0
a  b  c  d  x  y  4 xy  2  x  y  x  2 xy  1 đúng
2 2 2 2 2

Theo bổ đề a  b  c  d  với   2019  1 .

Cho tam giác nhọn ABC nội tiếp đường tròn tâm  O  . Kẻ đường cao AH . Gọi M
2 là trung điểm của BC , AM cắt OH tại G . Chứng minh rằng G nằm trên trục 5.0
(5.0đ) đẳng phương của đường tròn ngoại tiếp tam giác BOC và đường tròn Euler của tam
giác ABC .
A

E
L N

O
F
C
G

M
B H

Gọi P và Q theo thứ tự là giao điểm thứ hai của OM và OH với đường tròn
 BOC  ; E , F là chân các đường cao của tam giác ABC kẻ từ B, C tương ứng.
Gọi   là đường tròn Euler của tam giác ABC; d là trục đẳng phương của   ,
 BOC  . 1.0
Xét ba đường tròn   ,  BOC  và đường tròn đường kính BC (kí hiệu  BC  ).
Ta thấy: EF là trục đẳng phương của hai đường tròn   và  BC  ; BC là trục đẳng
phương của hai đường tròn  BC  và  BOC  ; d là trục đẳng phương của hai đường
tròn   và  BOC  . Do đó, EF, BC và d đồng quy tại I.
Rõ ràng OP là đường kính của đường tròn  BOC  nên HQP  900 nên H,M,P,Q
cùng thuộc một đường tròn.
Xét ba đường tròn  BOC  ,   và  HMPQ  ; ta thấy PQ là trục đẳng phương của
2.0
hai đường tròn  BOC  và  HMPQ  ; HM là trục đẳng phương của hai đường tròn
 và  HMPQ  ; d là trục đẳng phương của hai đường tròn   và  BOC  .
Do đó PQ, HM, d đồng quy tại I.
Gọi K là giao điểm thứ hai của AM và   ; N là trung điểm của AC.
Dễ thấy OA vuông góc với EF tại L; suy ra ONEL là tứ giác nội tiếp.
2.0
Do đó, AO. AL  AN . AE  AM . AK nên KMOL là tứ giác nội tiếp.
Mặt khác L,M,Q cùng nằm trên đường tròn đường kính OI nên tứ giác KOMQ nội
tiếp.
Do đó, GK .GM  GO.GQ nên G thuộc trục đẳng phương của đường tròn  BOC  và
  .
(Chú ý: Ba trục đẳng phương của ba đường tròn phân biệt luôn đồng quy là hiển
nhiên)
Cho x, y là các số nguyên dương. Nếu với mọi số nguyên dương n ta đều có

 1 chia hết cho  ny   1 . Chứng minh x  1


 n 2
x
5.0
Với   n  là số các số nguyên dương nhỏ hơn n và nguyên tố cùng nhau với

n
Bổ đề: ak  32 k y 2  1, k  0 với y nguyên dương cho trước, dãy a k có vô số
1.0
ước nguyên tố p  2  mod 3
Hiển nhiên luôn tồn tại số nguyên tố p thỏa mãn bổ đề.
Giả sử chỉ có hữu hạn p1 ,..., pm . Không mất tổng quát , ta có thể giả sử y
3 không chia hết cho 3 ( do a0  y 2  1  2  mod 3 ).
(5.0đ) Xét N   y 2  1 p1... pm thì 2.0

y 2  1  y 2  1 mod N   a  N    y 2  1  tN
2  N 
a  N   3

Hay a  N    y 2  1  tp1.. pm  1 có dạng 3l  1 nên tp1... pm  1 có dạng 3s+2.

Trở lại bài toán


Chọn n  3k và p  2  mod 3 là một ước nguyên tố của 32 k y 2  1
2.0
   1 mod p  x 2.3  1 mod p
 3k
    kết hợp x p1  1 mod p 
k 1
x

Ta được p | x 2  1 lấy p đủ lớn suy ra x  1 . (Do  p  1, 2.3k 1   2 ).


Cho S là một bội nguyên dương của tất cả các số từ 2 đến 2019 và n số nguyên
dương a1 , a2 ,..., an thuộc M : 1, 2,..., 2019 có tổng bằng 2S. Chứng minh rằng ta 5.0
có thể chọn ra một vài số trong n số này mà có tổng bằng S.
Với mỗi k  M , gọi xk là số lần xuất hiện của k trong a1 , a2 ,..., an . Từ giả thiết, ta
có:
x1  2 x2  ...  2019 x2019  a1  a2  ...  an  2S
Do S là bội của tất cả các số từ 2 tới 2019 và 2017,2018,2019 đôi một nguyên tố
cùng nhau nên 1.0
S  2017.2018.2019  2019 1  2  ...  2019 
Dẫn đến
x1  2 x2  ...  2019 x2019  2S  2019 1  2  ...  2019 
Kéo theo tồn tại k  M để xk  2019 . Có hai khả năng xảy ra:
S
TH1: kxk  S . Khi đó, vì k | S nên ta có thể lấy ra số k từ a1 , a2 ,..., an để có
k
một tổng là S. 2.0
TH2: kxk  S . Đặt B là tập các số trong a1 , a2 ,..., an  mà khác k. Lúc này B có
n  xk phần tử và
T : a
ai B
i  2S  kxk  S

Để chứng minh bài toán trong trường hợp này, ta chỉ cần chỉ cần chứng tỏ rằng có
thể chọn một vài số trong B sao cho tổng của chúng (kí hiệu S ' ) thỏa mãn
S  2019k  S '  S và k | S ' (vì khi đó có thể thêm không quá 2019 số k trong xk số
bằng k để có một tổng bằng S) .
Ngoài ra, ta có một bổ đề quen thuộc sau:
Bổ đề: Trong k số nguyên bất kỳ luôn chọn được một vài số có tổng là bội của k.
Chứng minh. Đặt
S1  a1
S 2  a1  a2
............
S k  a1  a2  ...  ak
TH1: Nếu tồn lại j sao cho S j k thì ta có ngay kết quả của bổ đề.
TH2: Nếu S j  k , j  1, k thì theo nguyên lý Dirichlet, ta có tồn tại
i, j : Si  S j  mod k  hay Si  S j k .
Trở lại bài toán. Áp dụng bổ đề , ta thấy rằng: vì B có tổng các phần tử T là một
bội của k nên B có thể được phân hoạch thành các tập B1 , B2 ,..., Bt sao cho Bi có 2.0
không quá k phần tử và k chia hết tổng các phần tử của Bi với mọi i  1, t . Do đó,
với mỗi i  1, 2,..., t nếu ta đặt si là tổng các số thuộc Bi thì ta có:
s1  s2  ...  st  T  S
Mặt khác, vì các số a1 , a2 ,..., an đều không lớn hơn 2019 nên si không vượt quá
2019k với mọi i  1, t . Theo nguyên lý liên tục trong rời rạc, tồn tại r  1, 2,..., t
sao cho
S  2019  s1  s2  ...  sr  S
r
Vì k | s1 , s2 ,..., sr nên các số cần chọn là tất cả các số trong Bi .
i 1
Ta có điều phải chứng minh.
SỞ GD & ĐT THANH HÓA KỲ THI CHỌN ĐT HSG CẤP TRƯỜNG DỰ THI CHỌN
TRƯỜNG THPT CHUYÊN LAM SƠN ĐT HSG QUỐC GIA NĂM HỌC 2019 - 2020
Môn thi: TOÁN
ĐỀ THI CHÍNH THỨC Ngày thi: 16/09/2019
( Đề thi có 01 trang) Thời gian làm bài: 180 phút, không kể thời gian phát đề

Bài 1 (5.0 điểm): Cho dãy ( xn ) xác định bởi:

 x1   ,  

  1 2  8
 x    1    x  , n  1
  
n 1 2 n 2
  n 1 ( n 1)  ( n 1)
Tìm số hạng tổng quát của dãy ( xn ) , từ đó tìm  để dãy ( xn ) có giới hạn hữu hạn.

Bài 2 (5.0 điểm): Tìm tất cả các cặp đa thức  P, Q  với hệ số thực thỏa mãn:

P  x  Q  y   Q  x  P  y 

với mọi số thực x và y.


Bài 3 (5.0 điểm): Cho tam giác ABC có AC  AB . Trên các cạnh AB , AC lần lượt lấy hai
điểm P , Q sao cho PQ song song với BC . Gọi O là giao điểm của hai đoạn thẳng BQ
và CP . Gọi A ' là điểm đối xứng của A qua BC . Đoạn A ' O cắt đường tròn ngoại tiếp (C )
của tam giác APQ tại S .
a) Gọi X là giao điểm thứ hai của đường thẳng qua A và song song với BC với
đường tròn (C ). Chứng minh ba điểm X , O , A ' thẳng hàng.
b) Chứng minh đường tròn ngoại tiếp tam giác BCS tiếp xúc với đường tròn (C ) .
Bài 4 (5.0 điểm): Một điểm M thực hiện các lần di chuyển trên mặt phẳng tọa độ, bắt đầu
từ điểm O (0;0). Mỗi lần di chuyển sẽ đến một điểm có toạ độ là số hữu tỉ và cách vị trí trước
đó đúng 1 đơn vị.
 1 16 
a) Chứng tỏ rằng điểm M có thể di chuyển đến điểm có tọa độ  ;  .
 5 13 
 1 1 
b) Điểm M có thể di chuyển đến điểm có tọa độ  ;  không? Tại sao?
 2019 2020 
---------------------------------- Hết ----------------------------------

Họ và tên: ……………………………………………. Số báo danh: ……………..

0
SỞ GD & ĐT THANH HÓA KỲ THI CHỌN ĐT HSG CẤP TRƯỜNG DỰ THI CHỌN
TRƯỜNG THPT CHUYÊN LAM SƠN ĐT HSG QUỐC GIA NĂM HỌC 2019 - 2020
Môn thi: TOÁN
ĐÁP ÁN ĐỀ THI CHÍNH THỨC Ngày thi: 16/09/2019
( Đáp án có 04 trang) Thời gian làm bài: 180 phút

 x1   ,  

Bài 1 (5.0 điểm): Cho dãy ( xn ) xác định bởi:   1 2  8
 xn 1  1  n  1  (n  1) 2  xn  (n  1) 2 , n  1
  
Tìm số hạng tổng quát của dãy ( xn ) , từ đó tìm  để dãy ( xn ) có giới hạn hữu hạn.
Đáp án.
n  n  3 xn  8
Từ giả thiết ta có  n  1 xn 1 
n 1

 n  1 xn1  nxn

8
(1.0 điểm)
 n  2  n  3  n  1 n  2   n  1 n  2  n  3
nxn  8
Đặt yn  , n  1  y1  . Ta có: yn 1  yn  , (1.0 điểm)
 n  1 n  2  6  n  1 n  2  n  3
suy ra
8
yn  yn 1 
n  n  1 n  2 
8
yn 1  yn  2 
 n  1 n  n  1
.....
8
y2  y1 
2.3.4
8 8 8
Cộng từng vế các đẳng thức trên ta được yn  y1    ...  ,
2.3.4 3.4.5 n  n  1 n  2 
n  
 n
1  1 1
Hay yn   8   4   
6 i  2 i  i  1 i  2  6 i  2  i  i  1  i  1 i  2  
 1   2  n  3n  4 
2
1
  4      (1.0 điểm)
6  2.3  n  1 n  2   6 3  n  1 n  2 
 n  1 n  2   2  n  3n  4 
2

Từ đó ta có xn  
6n 3n
 2   8
 xn     n   2  , n  1 (1.0 điểm)
 6 3 2 3n
 2
Từ đó, để dãy có giới hạn hữu hạn thì   0    4 .
6 3
4
Khi đó xn  , n  1 và lim xn  0 .
n
Đáp số   4 (1.0 điểm)

1
Bài 2 (5.0 điểm): Tìm tất cả các cặp đa thức  P, Q  với hệ số thực thỏa mãn:
P  x  Q  y    Q  x  P  y   với mọi số thực x và y.
Đáp án. Xét P  x  Q  y    Q  x  P  y   (1)
+) Nếu P  x  là hàm hằng tức là P  x   a , khi đó từ (1) ta có a  Q  x  a 
Thay x bởi x  a ta có a  Q  x  suy ra P  x   Q  x   a . Thử lại thấy đúng.
+) Nếu Q  x  là hàm hằng thì tương tự như trường hợp trên ta cũng có được P  x   Q  x   a .
(1.0 điểm)
+) Nếu P  x  và Q  x  đều không phải là hàm hằng, đặt:
P  x   ax n  bx n 1  R  x 
Q  x   cx m  dx m1  S  x 
m  1, n  1, a  0, c  0, deg R  n  1, deg S  m  1.
Ta thấy:
P  x  Q  y   là đa thức ẩn x có bậc là n và hệ số của số mũ cao nhất là a
Q  x  P  y   là đa thức ẩn x có bậc là m và hệ số của số mũ cao nhất là c
Vì có (1) nên a  c, m  n , khi đó:
P  x   ax n  bx n 1  R  x 
Q  x   ax n  dx n 1  S  x  (1.0 điểm)
Suy ra P  x  Q  y    a  x  Q  y    b  x  Q  y    R  x  Q  y    ax n  naQ  y  x n 1  bx n 1  ...
n n 1

Nên hệ số của x n 1 trong P  x  Q  y   là naQ  y   b


Tương tự hệ số của x n 1 trong Q  x  P  y   là naP  y   d
d b
Suy ra naQ  y   b = naP  y   d y  na Q  y   P  y    d  b  Q  y   P  y   .
na
d b
Đặt  t ta có Q  y   P  y   t y  Q  y   P  y   t y .
na
Thay y bởi x ta có Q  x   P  x   t x (2)
Nếu t  0  Q  y   P  y  y  Q  P . Thử lại thấy đúng. (1.0 điểm)
Nếu t  0 , thay (2) vào (1) ta có P  x  P  y   t   Q  x  Q  y   t  . Thay x bởi x  P  y  ta có:
P  x  P  y   P  y   t   P  x  P  y   P  y    t  P  x  t   P  x   t . Chọn:
x  0  P t   P  0  t
x  t  P  2t   P  t   t
x  2t  P  3t   P  2t   t
.........
x   k  1 t  P  kt   P  k  1 t   t
Suy ra P  kt   P  0   kt  P  kt   kt  P  0   0 k  *
. (3) (1.0 điểm)
Xét h  x   P  x   x  P  0  . Do (3) nên h  x  có vô số nghiệm, mà h  x  là đa thức bậc n nên
h  x   0  P  x   x  P  0   P  x   x  a (4)  Q  x   x  t  x  a  t  x  b  Q  x   x  b (5) .
Thử (4), (5) vào (1) ta có: P  x  y  b   Q  x  y  a   x  y  b  a  x  y  b  a (luôn đúng).
Như vậy đáp số của bài toán là: P  y   Q  x  hoặc P  x   x  a, Q  x   x  b, a  ,b 
(1.0 điểm)

2
Bài 3 (5.0 điểm): Cho tam giác ABC có AC  AB . Trên các cạnh AB , AC lần lượt lấy hai điểm P , Q
sao cho PQ song song với BC . Gọi O là giao điểm của hai đoạn thẳng BQ và CP . Gọi A ' là điểm đối
xứng của A qua BC . Đoạn A ' O cắt đường tròn ngoại tiếp (C ) của tam giác APQ tại S .
a) Gọi X là giao điểm thứ hai của đường thẳng qua A và song song với BC và đường tròn (C ).
Chứng minh ba điểm X , O , A ' thẳng hàng.
b) Chứng minh đường tròn ngoại tiếp tam giác BCS tiếp xúc với đường tròn (C ) .
Đáp án.

A'
t'
t

B J

P
S
O
I
A
Q C

a) (2.0 điểm) Ta thấy các cặp tam giác XPQ và AQP , A ' CB và AQP đồng dạng với nhau.
Suy ra XPQ và A ' CB là hai tam giác đồng dạng. Do đó, XQ // A ' B và XP // A ' C . (1.0 điểm)
Suy ra phép vị tự tâm O biến tam giác XPQ thành tam giác A’CB.
Vậy X, O, A’ là ba điểm thẳng hàng. (1.0 điểm)
b) (3.0 điểm) Gọi T là tâm đường tròn (AXA’). Do tam giác A’AX vuông tại A nên T là trung điểm A’X. Do
A’ đối xứng với A qua BC nên BC đi qua trung điểm AA’. Mặt khác, BC // AX, nên BC đi qua T. Ta có TA
= TX nên T thuộc vào đường trung trực của đoạn AX. (1.0 điểm)
Do APQX là hình thang cân nên T cũng thuộc đường trung trực của đoạn PQ. Suy ra, TP = TQ. Ta có,
XTC  SXA (do BC // AX) và SPB  SXA (do tứ giác APSX nội tiếp).
Do đó, XTC  SPB . Vì vậy, SPBT nội tiếp, tương tự đối với SQCT. Vì SPBT nội tiếp nên PSB  PTB . Vì
SQCT nội tiếp nên TQS  TCS . (1.0 điểm)
PTQ
Ta lại có PTB  90o   PQT  TQS  PQS .
2
Từ đó PSB  TCS  PQS . Gọi St , St ' lần lượt là 2 tiếp tuyến của hai đường tròn (C) và (BSC)
Suy ra PSB  BSt '  PSt  St  St ' nên St là tiếp tuyến chung của hai đường tròn (BSC) và (C).
Vậy đường tròn (BSC) tiếp xúc với đường tròn (C). (1.0 điểm)

3
Bài 4 (5.0 điểm): Một điểm M thực hiện các lần di chuyển trên mặt phẳng tọa độ bắt đầu từ điểm O (0; 0).
Mỗi lần di chuyển sẽ đến một điểm có toạ độ là số hữu tỉ và cách điểm trước đó đúng 1 đơn vị.
 1 16 
a) Chứng tỏ rằng điểm M có thể di chuyển đến điểm có tọa độ  ;  .
 5 13 
 1 1 
b) Điểm M có thể di chuyển đến điểm có tọa độ  ;  không? Tại sao?
 2019 2020 
Đáp án.
a) (2.0 điểm) Chúng ta sử dụng các bộ ba Pitago (m 2  n 2 ; 2mn; m 2  n 2 ) là (1;0;1) , (3; 4;5) và (12;5;13)
 1 16 
để xây dựng dãy các lần di chuyển của điểm M từ điểm O (0; 0) đến điểm  ;  theo sơ đồ sau:
 5 13 
3 4 6  1 
O(0;0)   ;    ;0    ;0   (1.0 điểm)
5 5 5  5 
 1 12 5   1 10   1 3   1 16 
  ;  ;    ;    ; . (1.0 điểm)
 5 13 13   5 13   5 13   5 13 
b) (3.0 điểm) Giả sử điểm M di chuyển từ điểm hữu tỉ A đến điểm hữu tỉ B, với toạ độ vectơ AB  (r ; s ),
a b
ở đó r , s là các số hữu tỉ. Gọi c là mẫu số chung của r , s , suy ra r  ; s  . Độ dài của mỗi bước di
c c
2 2
a b
chuyển là 1 đơn vị nên từ AB  1  2  2  1  a 2  b 2  c 2 . (1.0 điểm)
c c
Không mất tổng quát có thể xem c là số lẻ (bởi vì nếu c là số chẵn thì do a 2  b 2  c 2 nên a, b phải chẵn,
a 2a ' a ' b 2b ' b '
suy ra r    ;s   , nếu c’ chẵn thì tiếp tục quá trình trên suy ra mẫu số chung của
c 2c ' c ' c 2c ' c '
r , s là số lẻ). (1.0 điểm)
a b 
Giả sử ở bước di chuyển thứ i, điểm M di chuyển theo vectơ có toạ độ  i ; i  , với ci lẻ.
 ci ci 
 k a k b
Thì ở bước di chuyển thứ k toạ độ của điểm M là   i ;  i  .
 i 1 ci i 1 ci 
Từ tất cả các ci lẻ, suy ra điểm M không thể di chuyển tới vị trí có toạ độ với mẫu số chẵn.
 1 1 
Vậy điểm M không thể di chuyển đến điểm  ; . (1.0 điểm)
 2019 2020 

---------------------------------- Hết ----------------------------------

4
SỞ GD & ĐT THANH HÓA KỲ THI CHỌN ĐT HSG CẤP TRƯỜNG DỰ THI CHỌN
TRƯỜNG THPT CHUYÊN LAM SƠN ĐT HSG QUỐC GIA NĂM HỌC 2019 - 2020
Môn thi: TOÁN
ĐỀ THI CHÍNH THỨC Ngày thi: 17/09/2019
( Đề thi có 01 trang) Thời gian làm bài: 180 phút, không kể thời gian phát đề

Bài 5 (6.0 điểm): Tìm tất cả các hàm số f :  , liên tục trên và thỏa mãn điều kiện:
f  xy   f  x  y   f  xy  x   f  y 
với mọi số thực x và y.

Bài 6 (7.0 điểm): Cho đường tròn  O ; R  và một điểm I cố định, khác O ở trong đường
tròn đó, đường thẳng qua I vuông góc với OI cắt đường tròn tại C và D ; A là một điểm
nằm trên đường tròn, tia đối xứng với tia IA qua đường thẳng CD cắt đường tròn tại B .
Gọi M là trung điểm của AB.
a) Chứng minh đường thẳng AB đi qua một điểm cố định L khi A thay đổi trên
đường tròn  O ; R  .

b) Gọi N , P là giao điểm của đường thẳng OM với đường tròn  O ; R  ; điểm N

nằm trên cung ADB . Đường thẳng CN và DP cắt nhau ở Q . Chứng minh rằng các điểm
Q, N lần lượt là tâm của đường tròn nội tiếp và bàng tiếp của tam giác CMD .

Bài 7 (7.0 điểm): Tìm bộ ba các số nguyên dương  p, n, k  thỏa mãn p là số nguyên tố

Fermat và p n  n   n  1 . (số nguyên tố Fermat là số nguyên tố có dạng 22  1 với x là


k x

số tự nhiên).
---------------------------------- Hết ----------------------------------

Họ và tên: ……………………………………………. Số báo danh: ……………..

0
SỞ GD & ĐT THANH HÓA KỲ THI CHỌN ĐT HSG CẤP TRƯỜNG DỰ THI CHỌN
TRƯỜNG THPT CHUYÊN LAM SƠN ĐT HSG QUỐC GIA NĂM HỌC 2019 - 2020
Môn thi: TOÁN
ĐÁP ÁN ĐỀ THI CHÍNH THỨC Ngày thi: 17/09/2019
( Đáp án có 05 trang) Thời gian làm bài: 180 phút

Bài 5 (6.0 điểm): Tìm tất cả các hàm số f :  , liên tục trên và thỏa mãn điều kiện
f  xy   f  x  y   f  xy  x   f  y  , x, y  .
Đáp án. Xét phương trình f  xy   f  x  y   f  xy  x   f  y  , x, y  . (1)
Thay y  1 vào (1) ta được: f  x   f  x  1  f  2 x   f 1 , x  (2)
 f  x   f  x  1  f  2 x   f 1
 f  x   f  x  1  f  2 x   f 1  f  x  1  f  x  2   f  2 x  2 
 f  2 x  2   f  2 x   f  x  2   f  x  , x  . (1.0 điểm)
Do đó ta thu được:
x   x  x   x 
f  x  2   f  x   f   2   f    ...  f  k  2   f  k  , k  1
2  2 2  2 
  x   x 
 f  x  2   f  x   lim  f  k  2   f  k    f  2   f  0  .
k 
 2   2 
Từ đó suy ra:
f  x  2   f  x   f  2   f  0  , x  (3) (1.0 điểm)
Với n là số nguyên dương và đẳng thức (3) ta thu được:
f  x  2n   f  x  2n  2   f  2   f  0 
f  x  2n  2   f  x  2n  4   f  2   f  0 
..............
f  x  2  f  x   f  2  f  0
Cộng từng vế các đẳng thức trên ta được:
f  x  2n   n  f  2   f  0    f  x  , n  1, x  (4)
Tương tự ta có:
f  x  2n  1  n  f  2   f  0    f  x  1 , n  1, x  (5)
Thay y  2n vào (1) và kết hợp với đẳng thức (4) ta được:
f  2n  1 x   f  2n   f  2nx   f  x  2n   f   2n  1 x   f  2nx   f  x  2n   f  2n 
 f   2n  1 x   f  2nx   n  f  2   f  0    f  x  1  n  f  2   f  0    f 1
 f   2n  1 x   f  2nx   f  x   f  0  (6)
Tương tự ta có đẳng thức: f  2nx   f   2n  1 x   f  x  1  f 1 (7) (1.0 điểm)
Từ các đẳng thức (6) và (7) ta có:
f  2nx   f   2n  1 x   f  x  1  f 1
f   2n  1 x   f   2n  2  x   f  x   f  0 
..............
f  2 x   f  x   f  x  1  f 1
Cộng từng vế các đẳng thức trên ta được:
f  2nx   f  x   n  f  x  1  f 1    n  1  f  x   f  0   (1.0 điểm)
 f  2nx   n  f  x   f  x  1  f 1    n  1 f  0  . Kết hợp với đẳng thức (2) ta được:
f  2nx   nf  2 x    n  1 f  0  , x   f  nx   nf  x    n  1 f  0  , x  (8)
1
Trong (8) thay n  2, x  1 ta được:
f  2   2 f 1  f  0   2  f 1  f  0    f  2   f  0   f  0   f  1  f  1  2 f  0   f 1
Đặt a  f 1  f  0  ; b  f  0  . Khi đó với mỗi số nguyên dương n và từ đẳng thức (8) ta được:
f  n   nf 1   n  1 f  0   an  b
 1 1 1 1
f  n.   nf     n  1 f  0   f    a  b
 n n n n
 1   1  1 1
f  n.   nf      n  1 f  0   f     a  b (1.0 điểm)
 n   n  n n
m
Với mỗi số hữu tỷ r  luôn biểu diễn dưới dạng r  , trong đó m  * , n  nên theo đẳng thức (8)
n
và các đẳng trên ta được:
 1 1 m
f  r   f  m.   mf     n  1 f  0   a  b  ar  b  f  r   ar  b (9)
 n n n
Với mỗi x  , tồn tại dãy số hữu tỷ  xn  hội tụ đến x nên từ đẳng thức (9) và tính liên tục của f suy ra
f  x   ax  b . Thử lại thấy thỏa mãn. (1.0 điểm)
Bài 6 (7.0 điểm): Cho đường tròn  O; R  và một điểm I cố định ở trong đường tròn đó, đường thẳng qua
I vuông góc với OI cắt đường tròn tại C và D ; A là một điểm nằm trên đường tròn, tia đối xứng với tia
IA qua đường thẳng CD cắt đường tròn tại B . Gọi M là trung điểm của AB.
a) Chứng minh đường thẳng AB đi qua một điểm cố định L khi A thay đổi trên đường tròn  O; R  .
b) Gọi N, P là giao điểm của đường thẳng OM với đường tròn  O  ; điểm N nằm trên cung ADB . Đường
thẳng CN và DP cắt nhau ở Q . Chứng minh rằng các điểm Q, N là những tâm của đường tròn nội tiếp
và bàng tiếp của tam giác CMD .
Đáp án.
L

C I K D
E
Q
N
M
O
P

2
a) (3.0 điểm)
Gọi L là giao điểm của AB và OI; K là giao điểm của AB và CD.
Ta có IK OL và IK là phân giác của góc AIB , suy ra:  ABKL   1
Vì M là trung điểm của AB, theo hệ thức Newton suy ra : MA2  MB 2  MK .ML
= ( ML  LK ).ML = ML2  LK .LM (1.0 điểm)
OIK  KMO  900 suy ra tứ giác IOMK nội tiếp
Suy ra: LI .LO  LK .LM
Do đó: MA2  ML2  LK .LM  ML2  LI .LO
Suy ra: ML2  MA2  LI .LO  LO 2  OM 2  MA2  LI .LO (1.0 điểm)
Suy ra: OL2  OA2  LI .LO  ( LO  OI ) LO  LO 2  OI .OL
R2
Suy ra OA2  OI .OL . Suy ra OL  . Vậy L cố định. (1.0 điểm)
OI
b) (4.0 điểm) Trước hết ta chứng minh MK là phân giác của góc CMD .
Gọi E là giao điểm của OM với CD
OIE và OML đồng dạng suy ra OM .OE  OI .OL  OA2  R 2  OE 2  OM .OE  OE 2  R 2
 OE.ME  IE 2  OI 2  R 2  IE 2  ( R 2  OI 2 )  IE 2  IC 2
Ta có: PE / (OIMK )  KE.IE  OE.ME  KE.IE  IE 2  IC 2
 IC 2  IE 2  IE.KE  IE ( IE  KE )  IE.IK
Theo hệ thức Newton, ta suy ra  CDKE   1 (1) (1.0 điểm)

Mà MKME nên MK là phân giác trong của góc CMD . (2)


Theo chứng minh trên ta có: OM .OE  R 2  ON 2
  PNME   1   NPME   1 (3) (1.0 điểm)
Từ (1) và (3) ta suy ra: CN, PD, KM đồng quy tại Q.
Mà góc QDN  900 nên QMND là tứ giác nội tiếp
Suy ra : QDM  QNM  CDP
Suy ra DP là phân giác ngoài của góc CMD . (4)
Từ (2) và (4), ta có Q là tâm đường tròn nội tiếp tam giác CMD (1.0 điểm)
Ta lại có: DNDP suy ra DN là phân giác ngoài của góc CDM .
MLMN suy ra MN là phân giác ngoài của góc CMD .
Từ đó suy ra N là tâm đường tròn bàng tiếp ứng với góc MCD của tam giác CMD.
Vậy Q, N lần lượt là tâm của đường tròn nội tiếp và bàng tiếp của tam giác CMD. (1.0 điểm)

3
Bài 7 (7.0 điểm): Tìm bộ ba các số nguyên dương  p, n, k  thỏa mãn p là số nguyên tố Fermat và
p n  n   n  1 . (số nguyên tố Fermat là số nguyên tố có dạng 2 2  1 với x là số tự nhiên).
k x

Đáp án.
Cho p là số nguyên tố, a là số nguyên và  là số tự nhiên, nếu p | a và p 1 | a ta nói  là số mũ đúng
của p trong khai triển của a và kí hiệu vp (a)   . Ví dụ: v5 (5400)  v5 (53.32.2 2 )  3; v3 (5400)  2.
Bổ đề 1: Cho hai số nguyên lẻ x, y thỏa mãn 2 x  y và n là số nguyên dương chẵn. Khi đó:
v2  x n  y n   v2  x  y   v2  x  y   v2  n   1
Chứng minh: Ta có 4 x 2  y 2 nên đặt n  2 k .h  k, h  *
 , ƯCLN  h, 2   1. Khi đó ta có v  x  y  
2
n n

   v2  x 2
      ...  v
 x  y   k  1  v  x  y   v  x  y   v n  1
2k 1 2k 1
 v2 x h.2  y h.2  y2
k k
2 2

 
2 2 2 2

(1.0 điểm)
Bổ đề 2: Cho hai số nguyên x, y; n là số nguyên dương và p là số nguyên tố lẻ thỏa mãn:
p | x - y; p | x; p | y. Ta có vp  a n  b n   vp  a  b   vp  n  .
Chứng minh: Ta sẽ chứng minh quy nạp theo vp  n  . Trước hết, ta sẽ chứng minh khẳng định sau:
vp  x n  y n   vp  x  y   1 . Để chứng minh điều đó ta cần chỉ ra rằng:
p | x p 1  x p 2 y  x p 3 y 2  ...  xy p 2  y p 1 (1); p 2 | x p 1  x p 2 y  x p 3 y 2  ...  xy p 2  y p 1 (2).
Với (1), nhờ áp dụng x  y(mod p ) ta suy ra x p 1  ...  y p 1  px p 1  0(mod p ) (1.0 điểm)
Với (2), đặt y  x  kp, k  * . Khi đó 1  i  p  1 (i  ) thì
 i(i  1)
y i x p 1i   x  kp  x p 1i  x p 1i  x i  i  kp  x i 1   kp  x i 2  ... 
i 2

 2 
 x p 1i  x i  i  kp  x i 1   x p 1  ikpx p 2 (mod p 2 ). Do đó
x p 1  x p 2 y  ...  y p 1  x p 1   x p 1  kpx p 2    x p 1  2kpx p 2   ...   x p 1   p  1 kpx p 2 
p  1 2 p 2
 x p 1 
2
 
kp x  px p 1  0 mod p 2 . Như vậy, vp  x n  y n   vp  x  y   1 .

Tiếp theo, đặt n  p k .h ; b, k  , b  1, ƯCLN  b, p   1. Khi đó thì

vp a n  b n  vp  a p
         v a pk  b pk  v  a pk1
     
h h p k 1 p
 bp  bp
k k

 p p 
   

   1  v   a    b    ...  v x  y  k  v x  y  v n
p  p  p 
k 1 k 1 p p
p k 2 p k 2
 vp a p  b p 

p

Bổ đề được chứng minh. (1.0 điểm)


Trở lại bài toán, xét p n  n   n  1 (1). Đặt   2 x .
k

Nếu n  1 thì (1)  p  2 k  1  2  1  k  2;   1  p  3.


Nếu n  2. Ta gọi r là một ước nguyên tố của n.
Từ phương trình ta suy ra p n  1  mod n   p n  1  mod r  . Do đó ƯCLN  b, r   1. Đặt k là số nguyên
dương nhỏ nhất thỏa mãn p k  1  mod r  . Theo định lí Fermat nhỏ thì p r 1  1  mod r   k | r  1; k | n .
Do ƯCLN  r  1, n   1  k  1. Ta có r | p  1  r | 2  r  2  2 | n.
Ta có (1)  p n  1   n  1  n  1

k 1
 1  v2  p n  1  v2
   n  1 k 1

 1 . Nếu k  1 lẻ thì

v2  n  1 k 1

 1  v2  n   v2  p 2  1  v2  n   1  v2  p n  1 mâu thuẫn, vậy k  1 chẵn. (1.0 điểm)

Áp dụng Bổ đề 1: v2  p n  1  v2  n  1 k 1

 1  v2  p 2  1  v2  n   1  v2  n   v2  n  2   v2  k  1  1
 v2  p  1  v2  p  1  v2  n  2   v2  k  1
4
Nếu v2  k  1  v2  p  1  p  1 | k   n  1  n  1  mod p  (theo định lí Fermat nhỏ). Tuy nhiên theo
k

(1) thì n   n  1  mod p   n   n  1 mod p  mâu thuẫn.


k

Vậy v2  k  1  v2  p  1  1  v2  p  1  v2  2  2   v2  n  2   v2  n  2   2  n  2  mod 4 
+) Nếu p > 5 thì 22  1  5  x  2  p  2  mod 5  . Do
x

n  2  mod 4   p n  4  mod 5   4  n   n  1  mod 5  . Do 4  n   n  1 mod 5   k  1  mod 4  .


k

Do k lẻ nên k  3  mod 4   4  n   n  1  mod 5  .


3
(1.0 điểm)
Với n  0  mod 5   4  n   n  1  3  mod 5  mâu thuẫn, tương tự các trường hợp:
3

n  1  mod 5  ; n  2  mod 5  ; n  3  mod 5  ; n  4  mod 5  đều dẫn đến mâu thuẫn.


Vậy n  4   n  1  mod 5  n 
3 *
. Ta loại trường hợp p > 5.
+) Nếu p = 5 thì   2. Khi đó 3  v2  n  2   v2  k  1 . Do v2  n  2   2  v2  n  2   2; v2  k  1  1.
Ta cũng có 5n  n   n  1
k

-) Với n  2  k  3
-) Với n  3. Gọi q là ước nguyên tố lẻ của n thì q | 5   1  52  1  24. q | 3  q  3
n . q 1

 n  0  mod 6  . Kết hợp với n  2  mod 4   5n  1  mod13   n  1   n  1  mod13  . (1.0 điểm)


k

Áp dụng Bổ đề 2 ta có: v3  5n  1  v3  n  1  n



 1  1  v3    v3  k  1  v3  n   3 | k  1.
k 1

2
Ta cũng có k  3  mod 4   k  7  mod 12  . Theo định lí Fermat nhỏ suy ra:
 n  1   n  1    n  1 mod 13   n  1   n  1  mod 13   n   mod 13  , vô lí. (vì với
k 7

13 | n  5n  1  mod 13  , mâu thuẫn do 5n  5  mod 13  ).


Vậy  p, n, k    3,1, 2  ;  p, n, k    5, 2,3  . (1.0 điểm)
---------------------------------- Hết ----------------------------------

5
TRƯỜNG THPT CHUYÊN LÀO CAI ĐỀ THI HỌC SINH GIỎI CẤP TRƯỜNG LỚP 12
TỔ TOÁN TIN NĂM HỌC 2019 - 2020

Môn: TOÁN
Thời gian: 180 phút, không kể thời gian giao đề.
Ngày thi:

 1
ao  1; a1  2

Câu 1 (4.0 điểm). Cho hai dãy số  an  ,  bn  thỏa mãn b  1  2a , n 
 n n 1
 3
2bn1  2bn  an , n 

1 n bi
Với mỗi n  , đặt cn   . Tìm lim cn .
2n1 i 0 ai

Câu 2 (4.0 điểm). Tìm tất cả các hàm số f :  thỏa mãn


f  f  x   y 2   f  x 2   y 2 f  y   2 f  xy  , với mọi x, y  .
Câu 3 (4.0 điểm). Cho tam giác ABC nội tiếp đường tròn  O  với các đường cao
AD, BE , CF đồng quy tại H ; AA ' là đường kính của đường tròn  O  . Các đường thẳng
A ' B, A ' C cắt các đường thẳng AC , AB tương ứng tại M , N . Các điềm P, Q thuộc
đường thẳng EF sao cho PB, QC vuông góc với BC . Đường thẳng qua A vuông góc với
QN và PM lần lượt cắt đường tròn  O  tại các điểm X , Y . Các tiếp tuyến với đường tròn
 O  tại X , Y cắt nhau tại J .
1) Gọi S là trung điểm của đoạn AH . Chứng minh rằng SB || AY .
2) Chứng minh rằng JA '  BC.
Câu 4 (4.0 điểm). Cho dãy số (an ) xác định bởi: a1  34, an 1  4an3  104an2  107an với mo ̣i

số n nguyên dương. Tìm tấ t cả các số nguyên tố p thỏa mañ hai điề u kiê ̣n p  3 mod 4 và 
a2020  1 chia hế t cho p .
Câu 5 (4.0 điểm). Một cuộc thi giải Toán gồm 2 vòng, vòng 1 và vòng 2 với tổng cộng 28
bài toán. Mỗi thí sinh giải được đúng 7 bài và số thí sinh giải được mỗi bài là như nhau. Với
hai bài toán bất kì, có đúng 2 thí sinh giải được cả hai bài đó.
1) Hỏi có tất cả bao nhiêu thí sinh tham dự cuộc thi?
2) Chứng minh rằng ở vòng 1 có một thí sinh hoặc không giải được bài nào hoặc giải
được ít nhất 4 bài toán.
………. Hết……….

Thí sinh không được sử dụng máy tính cầm tay. Giám thị coi thi không giải thích gì thêm.

Họ và tên thí sinh:…………………………………...…....……...... Lớp:……….......….......


TRƯỜNG THPT CHUYÊN LÀO CAI HDC ĐỀ THI HSG CẤP TRƯỜNG LỚP 12
TỔ TOÁN TIN NĂM HỌC 2019 - 2020
Môn: TOÁN (Gồm 05 trang)
Lưu ý khi chấm bài:
- Đáp án chỉ trình bày một cách giải bao gồm các ý bắt buộc phải có trong bài làm của học
sinh. Khi chấm nếu học sinh bỏ qua bước nào thì không cho điểm bước đó.
- Nếu học sinh giải cách khác, giám khảo căn cứ các ý trong đáp án để cho điểm.
- Trong bài làm, nếu ở một bước nào đó bị sai thì các phần sau có sử dụng kết quả sai đó
không được điểm. Trong lời giải câu 3 nếu học sinh không vẽ hình thì không cho điểm.
 1
ao  1; a1  2

Câu 1 (4,0 điểm). Cho hai dãy số  an  ,  bn  thỏa mãn  1
bn   2an1 , n 
 3
2bn1  2bn  an , n 


1 n bi
Với mỗi n  , đặt cn  n1  . Tìm lim cn .
2 i 0 ai
Nội dung Điểm
4 5 1 1
Ta có b0  ; b1  ; 2bn 2  2bn1  an1  2bn1   bn   , n 
3 6 2  3  1,0
1 1 1
Đặt vn  bn  , n  ta có v0  ; v1  ; 4vn 2  4vn 1  vn , n 
3 3 2
1
Xét phương trình đặc trưng 4t 2  4t  1  0  t 
2
1 1
Do đó vn   a  bn  n , n  . Từ v0  1; v1  suy ra a  1; b  0
2 2
1,0
1 1 1 1
Suy ra vn  n , n   bn  n  , n   an  n , n 
2 2 3 2
n
b 2
Khi đó n   1, n 
an 3
1 n bi 1 n  2i  1  1  2  22  ...  2n  2n1  1 n
 cn       1  
2n1 i 0 ai 2n1 i 0  3  2n1 
 n  n 1
 n1 , n 
3  3.2 2
1,0
1 1 n  n  1
 cn   n1  n 2 , n 
3 3.2 2
1 1 n  n  1  1
Do đó lim cn  lim   n 1
  1,0
 3 3.2 2n  2  3
Câu 2 (4,0 điểm). Tìm tất cả các hàm số f :  thỏa mãn
f  f  x  y 2
  f  x   y f  y   2 f  xy  ,
2 2
với mọi x, y  .
Nội dung Điểm

Kí hiệu P  x, y  là khẳng
định f  f  x   y 2   f  x 2   y 2 f  y   2 f  xy  x, y  1 .
Với P  x,1  f  f  x   1  f  x 2   f 1  2 f  x   2  .
1,0
Với P  x, 1  f  f  x   1  f  x 2   f  1  2 f   x   3 .
Từ  2  và  3 suy ra f 1  2 f  x   f  1  2 f   x  , x  .
Từ đây, nếu x  1 thì f 1  f  1 và vì vậy f  x   f   x  , x  hay f là
hàm chẵn.
Với P 1,1  f  f 1  1  0, nói cách khác là tồn tại số thực b sao cho
f  b   0. Thay x  b vào  3 thì
f  f  b   1  f  b 2   f 1  f  1  f  b 2   f 1  f  b 2   0. 1,0

Với P  b,0   f  f  b    f  b 2   2 f  0   3 f  0   f  b 2   0.
Với P  0, y   f  y 2   y 2 f  y   4  . Ta có hai trường hợp sau:
Trường hợp 1 : Tồn tại b  , b  0 sao cho f  b   0. Như trên, ta có f  b 2   0.
P  b, y   f  f  b   y 2   f  b 2   y 2 f  y   2 f  by   f  y 2   y 2 f  y   2 f  by  1,0
 f  x   0, x   do (4)  .
Trường hợp 2 : f  b   0  b  0.
P  x, x   f  f  x   x 2   f  x 2   x 2 f  x   2 f  x 2   x 2 f  x   f  x 2   0
1,0
 f  x   x 2  0  f  x   x 2 , x  .
Thử lại, dễ thấy f  x   0, x  ; f  x   x 2 , x  thỏa mãn bài toán.

Câu 3 (4,0 điểm). Cho tam giác ABC nội tiếp đường tròn  O  với các đường cao
AD, BE , CF đồng quy tại H ; AA ' là đường kính của đường tròn  O  . Các đường thẳng
A ' B, A ' C cắt các đường thẳng AC , AB tương ứng tại M , N . Các điềm P, Q thuộc
đường thẳng EF sao cho PB, QC vuông góc với BC . Đường thẳng qua A vuông góc với
QN và PM lần lượt cắt đường tròn  O  tại các điểm X , Y . Các tiếp tuyến với đường tròn
 O  tại X , Y cắt nhau tại J .
1) Gọi S là trung điểm của đoạn AH . Chứng minh rằng SB || AY .
2) Chứng minh rằng JA '  BC.
Nội dung Điểm
A L

T
S

O
Q

E
F X
H
P
Y
C
B D K

A'
M

1. Gọi K  BC  AH . Lấy L   O  sao cho AL || BC . Kí hiệu  B  là đường tròn


điểm B (tâm B , bán kính 0) và  S  là đường tròn tâm S đường kính AH . 1,0
+) Ta có PM /  B   BM 2  ME.MA  PM /  S  (1) (vì các điểm E , F   S  )
+) Mặt khác PB || AH nên suy ra PBF  FAH  FEH  FEB . Do đó PB là tiếp
tuyến tại B của đường tròn  BEF  . Suy ra PB 2  PE.PF  PP /  B   PP /  S  (2)
Từ (1) và (2) suy ra MP là trục đẳng phương của hai đường tròn  B  ,  S  . 1,0
Do đó MP  SB .
Mà MP  AY nên suy ra SB || AY
2. Chứng minh tương tự câu 1 ta có SC || AX .
+) Dễ dàng chứng minh được tứ giác BHCA ' là hình bình hành nên K là trung
1,0
điểm của HA ' và BC . Suy ra SK || AA '
Mặt khác AL || ST || BC nên ta suy ra S  BCKT   A YXA ' L 
Mà K là trung điểm của BC nên S  BCKT   1  A YXA ' L   1 nên tứ giác
YA ' XL là tứ giác điều hòa.
Do JX , JY là các tiếp tuyến tại X , Y của đường tròn  O  nên JA ' đi qua L 1,0
Lại có AA ' là đường kính của đường tròn  O  nên JA '  AL và do AL || BC ta

suy ra JA '  BC .
Câu 4. (4,0 điểm) Cho daỹ số (an ) xác định bởi: a1  34, an 1  4an3  104an2  107an với mo ̣i

số n nguyên dương. Tìm tấ t cả các số nguyên tố p thỏa mañ hai điề u kiê ̣n p  3 mod 4 và  
a2020  1 chia hế t cho p .

Nội dung Điểm

Bổ đề : Cho p là số nguyên tố p  3 mod 4 . Nế u a, b    *


 
, a 2  b 2 p thì
a p, b p.
Thật vậy, đặt p  4k  3(k  ) , giả sử a / p khi đó b / p  (a, p)  1,(b, p)  1 . 1,0
Theo định lý Fecma a p 1  1(mod p)  a 2(2k 1)  1(mod p)
và b p 1  1(mod p)  b 2(2k 1)  1(mod p)
Theo giả thiết
 
a 2  b 2 p  a 2  b 2 (mod p)  (a 2 )2k 1  (b 2 )2k 1(mod p)  1  1(mod p) 1,0
 2  0(mod p) . Vô lý, vậy giả sử sai nên a p, b p (đpcm)

Ta làm bài toán tổng quát: Với mọi n tìm p nguyên tố thỏa mañ hai điề u
kiê ̣n p 3 mod 4 và an 1 1 chia hế t cho p .
2 an 1
27 2
an 1
27 an 27 2an 1 2an 1
an 27
1,0
Do đó
n ak 27 n 2
1
2ak 1 A2 an 1
27 7A2 an 1
1 7 22 A2 .
k 1 ak 27 k 1
2
Do an * ( n ) nên A *.
Giả sử p nguyên tố thỏa mãn hai điề u kiê ̣n p 3 mod 4 và an 1
1 chia hế t
cho p . 1,0
Nế u p 7 thì 2 2 2
A p 2 p (vô lý). Vâ ̣y p 7.

Câu 5. (4,0 điểm) Một cuộc thi giải Toán gồm 2 vòng, vòng 1 và vòng 2 với tổng cộng 28
bài toán. Mỗi thí sinh giải được đúng 7 bài và số thí sinh giải được mỗi bài là như nhau. Với
hai bài toán bất kì, có đúng 2 thí sinh giải được cả hai bài đó.
1) Hỏi có tất cả bao nhiêu thí sinh tham dự cuộc thi?
2) Chứng minh rằng ở vòng 1 có một thí sinh hoặc không giải được bài nào hoặc giải
được ít nhất 4 bài toán.
Nội dung Điểm
1. Gọi r là số thí sinh cùng giải được một bài toán bất kì. Số thí sinh này cũng
giải được 6r bài toán khác (có tính cả lặp) 1,0
Mặt khác mỗi bài toán trong số 27 bài còn lại được tính hai lần nên ta có
27
r  2 9
6
Do đó mỗi bài toán có 9 thí sinh giải được và số thí sinh tham dự cuộc thi là
28 1,0
9  36
7
2. Gọi n là số bài toán của vòng 1
*) Với n  1 , hiển nhiên kết luận của bài toán là đúng vì chỉ có đúng 9 thí sinh
giải được bài 1.
*) Với n  2 , ta chứng minh bài toán bằng phản chứng.
Giả sử mọi thí sinh hoặc giải được 1 bài hoặc 2 bài hoặc 3 bài ở vòng 1.
Gọi x, y , z tương ứng là số thí sinh giải được 1, 2, 3 bài ở vòng 1.
+) Khi đó ta có: x  y  z  36 1
+) Vì mỗi bài toán có 9 học sinh giải được nên ta có x  2 y  3z  9n  2  1,0
+) Ta quy ước mỗi thí sinh đạt được 1 phiếu cho mỗi cặp hai bài toán mà thí sinh
đó giải được trong vòng 1.
Do đó mỗi thí sinh trong số y thí sinh nhận được 1 phiếu; mỗi thí sinh trong số z
thí sinh nhận được C32  3 phiếu.
Suy ra tổng số phiếu mà các thí sinh nhận được là y  3 z
Mặt khác hai bài toán bất kì, có đúng 2 thí sinh giải được cả hai bài đó nên ta có
y  3z  2Cn2  3
 x  y  z  36 3x  3 y  3z  108
 
Từ đó ta có hệ:  x  2 y  3z  9n  3x  6 y  9 z  27n
 y  3z  2C 2 
 n 2 y  6 z  2n  n  1
2 1,0
 29  23
 y  2n  29n  108  2  n   
2
 0 (Mâu thuẫn).
 4  8
Vậy ở vòng 1 có một thí sinh hoặc không giải được bài nào hoặc giải được ít nhất
4 bài toán.
SỞ GIÁO DỤC VÀ ĐÀO TẠO KỲ THI LẬP ĐỘI TUYỂN DỰ THI QUỐC GIA
ĐẮK LẮK NĂM HỌC: 2019 - 2020

ĐỀ CHÍNH THỨC MÔN: TOÁN


(Đề thi có 01 trang) Thời gian: 180 phút (không kể thời gian giao đề)
Ngày thi: 24/09/2019

Câu 1. (5,0 điểm)


3 y  9 y 2  3x 2   x 4  x 2  3

Giải hệ phương trình  .
2020
x

3 y  2 x  5  x  1  4040 
Câu 2. (5,0 điểm)
Tìm hàm số f liên tục trên thỏa mãn f  x  1  3x 2  5 x  f  2 x  1 ; x 

.
Câu 3. (5,0 điểm)

Cho tam giác ABC có A  600 , AB  AC , H là giao điểm của hai đường cao
BE và CF . Gọi O là tâm đường tròn ngoại tiếp tam giác ABC . Trên các đoạn thẳng
MH  NH
BH và HF lần lượt lấy hai điểm M , N sao cho BM  CN . Tính .
OH
Câu 4. (5,0 điểm)
Cho M là tập hợp gồm n điểm trên mặt phẳng thỏa mãn hai điều kiện sau:
(1) Tồn tại 7 điểm thuộc M là 7 đỉnh của một thất giác lồi;
(2) Với 5 điểm bất kì thuộc M là 5 đỉnh của một ngũ giác lồi, tồn tại một điểm
thuộc M mà nó nằm ở miền trong ngũ giác đó.
Tìm giá trị nhỏ nhất của n .

---------------------- HẾT ----------------------


 Thí sinh không được sử dụng tài liệu.
 Cán bộ coi thi không giải thích gì thêm.
Họ và tên thí sinh: ...........................................................Số báo danh: ............................
SỞ GIÁO DỤC VÀ ĐÀO TẠO KỲ THI LẬP ĐỘI TUYỂN DỰ THI QUỐC GIA
ĐẮK LẮK NĂM HỌC: 2019 - 2020

MÔN: TOÁN
Thời gian: 180 phút (không kể thời gian giao đề)
Ngày thi: 24/09/2019

ĐÁP ÁN, BIỂU ĐIỂM VÀ HƯỚNG DẪN CHẤM


(gồm 04 trang)

Câu Đáp án Điểm


3 y  9 y  3x   x  x  3
2 2 4 2
1
1 
Giải hệ phương trình  .
(5đ) 2020
x
 
3 y  2 x  5  x  1  4040  2
x  0 không thỏa. Xét x  0 , chia hai vế phương trình (1) cho x 3 , ta được:
3
 3y   3y  1
   3    x  3x (*).
3

 x  x
3y
Hàm số f (t )  t 3  3t đồng biến trên nên từ (*) ta được:  x hay 3y  x 2 1
x
x 1 
Thay vào phương trình (2) ta được: 2020 ( x  1)  4  ( x  1)   2
2
1
 
Hàm số g ( x)  2020 x 1  ( x  1) 2  4  ( x  1)  có:
 
 1 
g'( x)  2020 x 1  ( x  1) 2  4  ( x  1)   ln 2020    0, x  1
   2
 
 ( x 1) 4 
nên hàm số đồng biến trên
1
Suy ra x  1; y  là nghiệm duy nhất của hệ phương trình đã cho. 1
3
2
Tìm hàm số f liên tục trên thỏa mãn f  x  1  3x 2  5 x  f  2 x  1 ; x  .
(5đ)
Ta có:
f  x  1  3x 2  5x  f  2x  1
 f  x  1  4x 2  4x  1  6x  3  f  2x  1  x 2  2x  1  3x  3
1

 f (x  1)   x  1  3  x  1  f  2x  1   2x  1  3  2x  1
2 2

Đặt g  x   f  x   x 2  3x thì g  x  là hàm liên tục và g  x  1  g  2x  1 ; x  .


u  2x  1
u 1  u 1 
, được g  u   g 
1
Thay x    u 1  ; u 
2  2  x  1  2 
a1  x

Với mỗi x  , xét dãy  a n  :  a n  1 , có: g  a n   g  a n 1  . 1
a n 1  2
a 1 a 1 a 1
Vì a n 1  n  a n 1  1  n  a n 1  1  1 n .
2 2 2
1
 1 
 lim  a n 1  1   a1  1 lim  n   0  lim a n 1  lim a n  1
2 
 g  x   g  a1   g  a 2   ...  limg  a n   g  lim a n   g 1  C .
 f  x   x 2  3x  C (thử lại, đúng). 1
Vậy mọi hàm số f thỏa mãn đề ra có dạng f  x   x 2  3x  C; x  .
Cho tam giác ABC có A  600 , AB  AC , H là giao điểm của hai đường cao
BE và CF . Gọi O là tâm đường tròn ngoại tiếp tam giác ABC . Trên các đoạn
3
thẳng BH và HF lần lượt lấy hai điểm M , N sao cho BM  CN . Tính
(5đ)
MH  NH
.
OH

Có BOC  2 A  1200 .
Tứ giác AEHF nội tiếp nên EHF  1800  A  1200  BHC  EHF  1200 .
1
Do BOC  BHC nên tứ giác BOHC nội tiếp  OBH  OCH .
Lấy K trên đoạn BE thỏa BK  CH .
OBK  OCH (c.g.c) nên OK  OH và BOK  COH . 1
Có HOK  HOC  COK  BOK  COK  BOC  1200
OHK : HK 2  OH 2  OK 2  2OH .OK .cos HOK  3OH 2  HK  OH 3 .
1
Có BM  CN ; BK  CH  KM  NH
MH  NH MH  MK HK
Vậy    3. 1
OH OH OH
Cho M là tập hợp gồm n điểm trên mặt phẳng thỏa mãn:
(1) Tồn tại 7 điểm thuộc M là 7 đỉnh của một thất giác lồi;
4
(2) Với 5 điểm bất kì thuộc M là 5 đỉnh của một ngũ giác lồi, tồn tại một
(5đ)
điểm thuộc M mà nó nằm ở miền trong ngũ giác đó.
Tìm giá trị nhỏ nhất của n .
Giả sử thất giác lồi có các đỉnh thuộc M là A1 A2 A3 A4 A5 A6 A7 .
Có A1 A2 A3 A4 A5 là một ngũ giác lồi nên tồn tại điểm P1 thuộc M nằm trong nó.
1
Lại có A1P1 A5 A6 A7 là một ngũ giác lồi nên tồn tại điểm P2 thuộc M nằm trong
nó.
Khi đó tồn tại ít nhất 5 điểm trong  A1; A2 ; A3 ; A4 ; A5 ; A6 ; A7  không thuộc đường
thẳng P1P2 . Trong đó có ít nhất 3 điểm thuộc cùng một nửa mặt phẳng bờ là P1P2
1
. 3 điểm này cùng với P1 và P2 lập thành một ngũ giác lồi nên tồn tại điểm P3
thuộc M nằm trong nó.
Gọi  1 là nửa mặt phẳng bờ P1P2 không chứa tam giác P1P2 P3 và không chứa
điểm nào thuộc đoạn P1P2 . Tương tự ta định nghĩa  2 ,  3 .
Tồn tại một miền trong 3 miền  1 ,  2 ,  3 chứa ít nhất 3 điểm trong 7 điểm
 A1; A2 ; A3 ; A4 ; A5 ; A6 ; A7  . 1
Không mất tính tổng quát, giả sử 3 điểm A1 , A2 , A3 thuộc miền  1 thì
A1 , A2 , A3 , P1 , P2 là các đỉnh của một ngũ giác lồi. Do đó tồn tại điểm P4 thuộc M
nằm trong ngũ giác đó.
Do đó n  11 .
Trong mặt phẳng Oxy , xét tập hợp M gồm 11 điểm có tọa độ nguyên như hình
vẽ.

M thỏa mãn điều kiện (1).


Ta chứng minh M thỏa mãn điều kiện (2).
Giả sử tồn tại một ngũ giác lồi có các đỉnh thuộc M nhưng không có điểm nào
thuộc M nằm bên trong nó.
Trong các ngũ giác như vậy, tồn tại một ngũ giác có diện tích nhỏ nhất, kí hiệu
là ABCDE . Với các điểm tọa độ nguyên trong mặt phẳng Oxy , có 4 trường hợp:
(chẵn, chẵn), (chẵn, lẻ), (lẻ, chẵn), (lẻ, lẻ). Suy ra trong 5 điểm A, B, C , D, E tồn
tại ít nhất 2 điểm có cùng tính chẵn lẻ.
Khi đó trung điểm P của đoạn thẳng nối 2 điểm này cũng có tọa độ nguyên và
cũng thuộc M . Do P không nằm trong ABCDE nên P phải thuộc cạnh của nó,
giả sử là AB . Ngũ giác lồi PBCDE không chứa điểm nào thuộc M bên trong 1
nó và có diện tích nhỏ hơn ABCDE : vô lí. Suy ra M thỏa mãn điều kiện (2).
Vậy giá trị nhỏ nhất của n là 11.
SỞ GIÁO DỤC VÀ ĐÀO TẠO KỲ THI LẬP ĐỘI TUYỂN DỰ THI QUỐC GIA
ĐẮK LẮK NĂM HỌC: 2019 - 2020

ĐỀ CHÍNH THỨC MÔN: TOÁN


(Đề thi có 01 trang) Thời gian: 180 phút (không kể thời gian giao đề)
Ngày thi: 25/09/2019

Câu 5. (5,0 điểm)

 1  2  n
Cho dãy số  xn  xác định bởi xn  1  2 1  2  ... 1  2  . Tính lim  ln xn 
 n  n   n 
.
Câu 6. (5,0 điểm)
Cho đa thức P  x  có tất cả các hệ số đều là số nguyên dương và đa thức

Q  x   1  x  x 2  x 3  x 4 . Chứng minh rằng nếu đa thức P  x 3  chia hết cho đa thức

Q  x  thì đa thức P  x 2019  cũng chia hết cho đa thức Q  x  .

Câu 7. (5,0 điểm)


Cho trước p là một số nguyên tố lớn hơn 2. Tìm tất cả các số nguyên dương k

sao cho k 2  2 pk cũng là số nguyên dương.

Câu 8. (5,0 điểm)


Cho tam giác ABC nhọn, không cân, có các đường cao BM, CN và phân giác AD.
Các điểm E, F lần lượt là hình chiếu của D trên AB, AC. Chứng minh rằng các đường
thẳng MN, EF và BC đồng quy.

---------------------- HẾT ----------------------


 Thí sinh không được sử dụng tài liệu.
 Cán bộ coi thi không giải thích gì thêm.
Họ và tên thí sinh: ...........................................................Số báo danh: ............................
SỞ GIÁO DỤC VÀ ĐÀO TẠO KỲ THI LẬP ĐỘI TUYỂN DỰ THI QUỐC GIA
ĐẮK LẮK NĂM HỌC: 2019 - 2020

MÔN: TOÁN
Thời gian: 180 phút (không kể thời gian giao đề)
Ngày thi: 25/09/2019

ĐÁP ÁN, BIỂU ĐIỂM VÀ HƯỚNG DẪN CHẤM


(gồm 03 trang)

Câu Đáp án Điểm


 1  2  n
5 Cho dãy số  xn  xác định bởi xn  1  2 1  2  ... 1  2  . Tính lim  ln xn 
 n  n   n 
(5đ)
.
x2
Trước hết, ta chứng minh bất đẳng thức: x   ln( x  1)  x,  x  0 (*).
2
x2
Xét hàm số f ( x)  ln( x  1)  x  , x  0 .
2 1
1
Ta có: f '( x)   1  x  0, x  0
x 1
nên hàm số đồng biến trên  0;   . do đó: f ( x)  0, x  0 .
Xét hàm số g ( x)  ln( x  1)  x, x   0;   ; chứng minh được:
1
ln( x  1)  x,  x  0.

Ta có: ln xn  ln 1 
1   2  n
2 
 ln 1  2   ...  ln 1  2  . Áp đụng (*), ta được:
 n   n   n 
1
n
1
2 
1
  1
1  2  ...  n   4 12  22  ...  n 2  lnx n  2 1  2  ...  n 
2n n
1 n(n  1) 1 n(n  1)(2 n  1) 1 n(n  1)
hay 2 .  4.  lnx n  2 . 1
n 2 2n 6 n 2
1 n(n  1) 1 n(n  1)(2 n  1)   1 n(n  1)  1
Vì lim  2 .  4.   lim  2 . 
n 2 2n 6  n 2  2
1
1
nên lim  lnx n   .
2
Cho đa thức P  x  có tất cả các hệ số đều là số nguyên dương và đa thức
6 Q  x   1  x  x 2  x 3  x 4 . Chứng minh rằng: nếu đa thức P  x 3  chia hết cho
(5đ)
đa thức Q  x  thì đa thức P  x 2019  cũng chia hết cho đa thức Q  x  .
Viết lại mỗi đơn thức n.x k  x k  x k  ...  x k (n ký hiệu x k ).
Khi đó mỗi đa thức P(x) có tất cả các hệ số đều là số nguyên dương luôn viết 1
được dưới dạng:
n
P(x)  x a1  x a 2  ...  x a n   x a i với 0  a1  a 2  ...  a n ;a i  .
i 1
Và ta nói đa thức P  x  tương ứng dãy số tự nhiên  a n  .
Đặt a i  5bi  ci ; bi , ci  ;0  ci  4 .

 
n n
Có P(x)   x a i  x ci   x ci .
i 1 i 1


ci ci
 5bi

x  x  x x  1 x5  1 Q  x   1  x  x 2  x3  x 4
ai
 1
Mà  n , do đó:
deg  x  4  deg Q  x 
ci
 i 1
n n
P(x) Q(x)   x ci
Q(x)   x ci  C.Q  x   C  Cx  Cx 2  Cx 3  Cx 4 với C 
i 1 i 1
 Trong dãy  ci  có đúng C số là 0, C số là 1, C số là 2, C số là 3, C số là 4.
Vậy điều kiện cần và đủ để P(x) chia hết cho đa thức Q(x) là dãy  a i  tương ứng
1
của P  x  có tính chất “cân bằng” là: “số lượng các số hạng a i chia 5 lần lượt
nhận các số dư 0, 1, 2, 3, 4 đều bằng nhau và bằng số tự nhiên C nào đó”.
Xét bảng đồng dư 5:
x  mod 5  0 1 2 3 4
3x  mod 5  0 3 1 4 2 1
2019x  mod 5  0 4 3 2 1
Từ bảng đồng dư ta có:
Nếu đa thức P  x 3  chia hết cho đa thức Q  x  thì dãy  3a n  tương ứng đa thức
P  x 3  có tính chất “cân bằng”, do đó dãy  a n  có tính chất “cân bằng”, nên dãy 1
 2019a n  có tính chất “cân bằng”, vì vậy đa thức P  x 2019
 tương ứng cũng chia
hết cho đa thức Q  x  .
7 Cho trước p là một số nguyên tố lớn hơn 2. Tìm tất cả các số nguyên dương k
(5đ) sao cho k 2  2 pk cũng là số nguyên dương.

Giả sử k 2  2 pk  n với n  .
1
 k 2  2 pk  n 2  0  k  p  p 2  n 2 .

Do k  nên p 2  n 2 là số chính phương. 1
Đặt p 2  n 2  m 2 với m  thì m 2  n 2  p 2
1
  m  n  m  n   p 2 .
 p2  1
 m 
m  n  1 2
Do p là số nguyên tố lớn hơn 2 nên   1
m  n  p n  p  1
2 2

 2
2 p   p 2  1
k  pm .
2
1
 p  1
2

Do k  nên k .
2
Cho tam giác ABC nhọn, không cân, có các đường cao BM, CN và phân giác AD.
8
Các điểm E, F lần lượt là hình chiếu của D trên AB, AC. Chứng minh rằng các
(5đ)
đường thẳng MN, EF và BC đồng quy.
A

M
N
E F
1
I B C
P D
Vì tam giác ABC không cân nên MN cắt BC tại I.
Hạ đường cao AP.
IB NA MC
Theo định lý Menelauyt, I, M, M thẳng hàng nên: . .  1 1 .
IC NB MA
PB NA MC
Theo định lý Ceva, AP, BM, CN đồng quy nên: . .  1  2  . 1
PC NB MA
 AE  AF
Vì AD là phân giác nên  , do đó:
 DE  DF 1
EA FC EA FC FC DF : tan C tan B
.  .   
EB FA EB FA EB DE : tan B tan C
PA : PB PC PC  2 NA MC 1 IC
    .  1
PA : PC PB PB NB MA IB
IB EA FC
Do đó . .  1 , nên theo định lý Menelauyt thì I, E, F thẳng hàng.
IC EB FA 1
Vậy các đường thẳng MN, EF và BC đồng quy tại I.
1

SỞ GIÁO DỤC VÀ ĐÀO TẠO KỲ THI CHỌN HỌC SINH GIỎI CẤP TỈNH
GIA LAI LỚP 12 THPT, NĂM HỌC 2019-2020
.................. Môn thi : TOÁN-BẢNG A
ĐỀ CHÍNH THỨC Thời gian làm bài 180 phút
(Đề thi gồm 01 trang) (không kể thời gian giao đề)
Ngày thi : 18/10/2019

Câu 1 (5 điểm). Xét g ( x ) = a2020 x2020 + a2019 x2019 + · · · + a1 x + a0 là một đa thức bậc 2020 bất kỳ
nhận x = 2020 làm nghiệm và giả sử rằng g ( x ) = ( x − 2020) . f ( x ) với

f ( x ) = b2019 x2019 + b2018 x2018 + · · · + b1 x + b0 .

Đặt A = max {| a0 | ; | a1 | ; . . . ; | a2020 |} và B = max {|b0 | ; |b1 | ; . . . ; |b2019 |} . Chứng minh A ≥ B.

Câu 2 (5 điểm). Cho tam giác ABC nhọn. Trên hai cạnh AB, AC lần lượt lấy hai điểm M, N sao
cho BM = CN. Giả sử đường tròn ngoại tiếp tam giác ABC cắt đường tròn ngoại tiếp tam giác
AMN tại hai điểm A và K.

a) Chứng minh rằng K là điểm chính giữa của cung BAC của đường tròn ngoại tiếp ∆ABC.

b) Gọi I và J lần lượt là tâm của đường tròn ngoại tiếp tam giác ABN và ACM. Chứng minh rằng
bốn điểm A, K, I, J cùng nằm trên một đường tròn.

Câu 3 (4 điểm). Cho trước số abcde f có sáu chữ số. Chứng minh rằng ta có thể sắp xếp lại các chữ
số của số đã cho sao cho hiệu giữa tổng ba chữ số đầu tiên và ba chữ số cuối cùng nằm giữa 0 và 9
(tức là sắp xếp lại các chữ số của số đã cho thành số xyztuv sao cho 0 ≤ x + y + z − t − u − v ≤ 9).

Câu 4 (6 điểm). Tìm tất cả hàm số f : N → N thoả mãn f (0) =1 và

n
Åï òã Åï
n òã
f (n) = 2 f + 3f , ∀ n ∈ N∗ .
5 25

————————————–Hết————————————–
Họ và tên thí sinh...............................................................Số báo danh..................................
Thí sinh không sử dụng tài liệu, máy tính cầm tay. Cán bộ coi thi không giải thích gì thêm.
SỞ GIÁO DỤC VÀ ĐÀO TẠO KỲ THI LẬP ĐỘI TUYỂN CỦA TỈNH
TỈNH QUẢNG NINH DỰ THI CHỌN HỌC SINH GIỎI QUỐC GIA THPT NĂM 2020
Môn thi: Toán
ĐỀ THI CHÍNH THỨC Ngày thi: 01/10/2019
Thời gian làm bài: 180 phút, không kể thời gian giao đề
(Đề thi này có 01 trang)

Bài 1. (5 điểm):

1
Cho dãy số (un ) xác định bởi: u1  1; un1  , n  1. Chứng minh rằng
u1  u2  ...  un
tồn tại N  *
sao cho:
u1  u2  ...  uN  2020.

Bài 2. (5 điểm):

Cho tam giác ABC nhọn, không cân, đường cao AD , trực tâm H . Dựng đường tròn
tâm M đường kính BC . Từ A kẻ các tiếp tuyến AE , AF tới đường tròn  M  ( E , F là các
tiếp điểm). Các đường thẳng EF và BC cắt nhau tại N . I là trung điểm của AH . Đường
thẳng qua D vuông góc với IM cắt các đường thẳng AB , AC lần lượt tại P và Q . Chứng
minh các điểm M , N , P, Q cùng nằm trên một đường tròn.

Bài 3. (5 điểm):

Cho đa thức Pm  x   x 4   2m  4  x 2   m  2  với m là tham số. Tìm tất cả các giá


2

trị nguyên dương của m để Pm  x  viết được thành tích của hai đa thức hệ số nguyên có bậc
lớn hơn hoặc bằng 1 .

Bài 4. (5 điểm):

Cho phương trình:

x 1 y 1
 3
y x

với x, y là các số nguyên dương.

a. Tìm các nghiệm ( x; y ) của phương trình sao cho x, y nguyên tố cùng nhau;

b. Chứng minh phương trình đã cho có vô số nghiệm.

............................Hết...........................
Thí sinh không được sử dụng tài liê ̣u và máy tính cầ m tay. Cán bộ coi thi không giải thích gì thêm.
Họ và tên thí sinh: ................................................................... Số báo danh: .................................
Chữ ký của giám thị 1:.................................... Chữ ký của giám thị 2:............................................
SỞ GIÁO DỤC VÀ ĐÀO TẠO HƯỚNG DẪN CHẤM THI LẬP ĐỘI TUYỂN CỦA TỈNH
TỈNH QUẢNG NINH DỰ THI CHỌN HỌC SINH GIỎI QUỐC GIA THPT NĂM 2020
Môn thi : Toán
Ngày thi: 01/10/2019
ĐỀ THI CHÍNH THỨC
(Hướng dẫn này có 03 trang)

Bài Sơ lược lời giải Điểm


Bài 1 Chỉ ra dãy dương.
0,5
(5 điể m)
1 1
un 1    un nên dãy giảm, suy ra dãy hội tụ.
u1  u2  ...  un u1  u2  ...  un 1 1,5

1 1 1 1
Biến đổi được un 1   nên un   (1).
u1  u2  ...  un 1
 un un 1 un
1,0
un

1 1
Nếu lim un  L  0 thì từ (1) suy ra L   (vô lý) nên lim un  0 . Do đó
L L
1 1,0
lim   .
un

1
Mà u1  u2  ...  un  nên lim  u1  u2  ...  un    .
un 1
1,0
Suy ra đpcm.
A
Bài 2
(5 điể m)

I
J
F

K
H
E
Q

N B M C
D

2,0
P

Gọi J, K là chân các đường cao kẻ từ B và C của tam giác ABC.

Xét 3 đường tròn đường kính AM, đường kính BC, đường tròn Euler của tam giác
ABC có 3 trục đẳng phương là EF, DM, KJ nên chúng đồng quy tại N.

Chỉ ra (BCDN) là hàng điểm điều hòa, mà M là trung điểm BC nên MD.MN  MB 2
(1). 1,0
Chỉ ra JK  IM nên JK / / PQ . Mà tứ giác BCJK nội tiếp nên tứ giác BPCQ nội
tiếp suy ra DB.DC  DP.DQ . 1,0

Cuối cùng ta chỉ cần chứng minh DB.DC  DM .DN hay


( DM  MB)( DM  MC )  DM ( DM  MN ) tương đương với MB.MC  DM .MN
đúng do đẳng thức (1). 1,0

Từ đó suy ra M, N, P, Q cùng thuộc một đường tròn.

Bài 3 Xét Pm  x   x 4   2m  4  x 2   m  2   0 :  '   m  2    m  2   8m , tính


2 2 2

(5 điể m)
được các nghiệm của Pm  x  là: 1,0
x1  m  2, x2  m  2, x3   m  2, x4   m  2

-  
Nếu m  2 , P2  x   x 4  8 x 2  x 2 x 2  8 thỏa mãn đề bài.
- Nếu m  2 , Pm  x  có 4 nghiệm phân biệt x1 , x2 , x3 , x4 như trên. Nếu
1,0
Pm  x   H ( x).G  x  thỏa mãn đề bài, với deg G  deg H thì deg G  1 hoặc
deg G  2 , đồng thời G  x  nhận 1 hoặc 2 trong số x1 , x2 , x3 , x4 là nghiệm.
+ Nếu deg G  x  =1, G  x  monic thì nghiệm của G  x  phải là nghiệm
nguyên suy ra m  2 , đã xét. 0,5

+ Nếu deg G  x  =2, G  x  monic, nên tổng và tích các nghiệm của G  x 
phải là các số nguyên.
 x1  m  2
TH1:  thì x1  x2  2 m   m  n 2 .
 x2  m  2
 x1  m  2
TH2:  thì x1  x3  2 2  .
 x3   m  2
 x1  m  2
TH3: 
 x4   m  2

thì x1.x4   m  2  2 2m   m  2n 2 .  1,5
 x2  m  2
TH4:  như TH3.
 x3   m  2
 x2  m  2
TH5:  tương tự TH2.
 x4   m  2
 x3   m  2
TH6:  như TH1.
 x4   m  2

Tóm lại ta có m  n 2 hoặc m  2n 2 với n nguyên dương. Thử lại:

Pn2  x   x 4   2n 2  4  x 2   n 2  2    x 2  2nx  n 2  2  x 2  2nx  n 2  2 


2 1,0
.
2

P2 n2  x   x 4   4n 2  4  x 2   2n2  2   x 2  2  n  1
2
 x 2
 2  n  1
2
.
Vậy m  n 2 hoặc m  2n 2 với n nguyên dương.

Bài 4 a, Dễ thấy x  y không thỏa mãn đề bài.


(5 điể m)
TH1: x  y  x  y  1 . Biến đổi về x 2  (1  3 y ) x  y ( y  1)  0 .

Suy ra y ( y  1) x nên y  1 x . Do đó x  y  1 . Thế vào phương trình tính được


y  2; x  3 . 1,0

TH2: x  y tương tự TH1 được x  2, y  3 .

Vậy các nghiệm cần tìm là (2;3), (3; 2).

b, Xét dãy (un ) xác định bởi u1  2; u2  3 và un  2  3un 1  un  1 (n  1) . 2,0

Ta chứng minh (un ; un 1 ) đều là các nghiệm của phương trình bằng quy nạp. 1,0

Với n  1 khẳng định đúng. Giả sử khẳng định đúng với n  k .

Với n  k  1 , khi đó uk2 2  (1  3uk 1 )uk  2  uk21  uk 1 

uk21  (1  3uk )uk 1  uk2  uk  (uk  2  uk )(uk  2  3uk 1  uk  1)  0 .

Chỉ ra dãy (un ) là dãy các số nguyên dương, tăng nên (uk ; uk 1 ) phân biệt. 1,0

Suy ra đpcm.

Chú ý khi chấm:


1. Hướng dẫn chấm này chỉ trình bày sơ lược bài giải. Bài làm của học sinh phải chi tiết, lập luận chặt chẽ,
tính toán chính xác mới được điểm tối đa. Các cách giải khác nếu đúng vẫn cho điểm. Tổ chấm trao đổi và
thống nhất chi tiết nhưng không được quá số điểm dành cho câu, phần đó.
2. Có thể chia điểm thành từng phần nhưng không dưới 0,25 điểm và phải thống nhất trong cả tổ chấm.
Điểm toàn bài là tổng số điểm các phần đã chấm, không làm tròn điểm.
3. Mọi vấn đề phát sinh trong quá trình chấm phải được trao đổi thống nhất trong tổ chấm và ghi vào biên
bản.

SỞ GIÁO DỤC VÀ ĐÀO TẠO QUẢNG NINH

............................. Hết ...........................


SỞ GIÁO DỤC VÀ ĐÀO TẠO KỲ THI LẬP ĐỘI TUYỂN CỦA TỈNH
TỈNH QUẢNG NINH DỰ THI CHỌN HỌC SINH GIỎI QUỐC GIA THPT NĂM 2020
Môn thi: Toán
ĐỀ THI CHÍNH THỨC Ngày thi: 02/10/2019
Thời gian làm bài: 180 phút, không kể thời gian giao đề
(Đề thi này có 01 trang)

Bài 1. (5 điểm):

Chứng minh rằng:

3a 2  1 3b2  1 3c2  1
  6
b  c 2 c  a 2 a  b2

với mọi số thực a, b, c thỏa mãn a  b2  0; b  c 2  0; c  a 2  0 .

Bài 2. (5 điểm):

Tìm tất cả các hàm f :  thỏa mãn:

 
xf  x  xy   xf  x   f x 2 f  y  ; x, y  .

Bài 3. (5 điểm):

Cho tam giác ABC nhọn nội tiếp đường tròn (O) , các tiếp tuyến tại B và C của
(O) cắt nhau tại M , các tiếp tuyến tại A và C của (O) cắt nhau tại N . Các đường thẳng
AM và BC cắt nhau tại D . Các đường thẳng BN và CA cắt nhau tại E . Các điểm I , J
lần lượt là trung điểm của AD , BE .

1. Chứng minh ABI  BAJ .

2. Tính tỉ số các cạnh của tam giác ABC để góc ABI có số đo lớn nhất.

Bài 4. (5 điểm):

Cho số nguyên dương n  2 . Tìm số các tập con của tập Sn  {1;2;3;...; n} chứa
đúng hai số nguyên dương liên tiếp.

............................Hết...........................

Thí sinh không được sử dụng tài liê ̣u và máy tính cầ m tay. Cán bộ coi thi không giải thích gì thêm.
Họ và tên thí sinh: ................................................................... Số báo danh: .................................
Chữ ký của giám thị 1:.................................... Chữ ký của giám thị 2:............................................
SỞ GIÁO DỤC VÀ ĐÀO TẠO HƯỚNG DẪN CHẤM THI LẬP ĐỘI TUYỂN CỦA TỈNH
TỈNH QUẢNG NINH DỰ THI CHỌN HỌC SINH GIỎI QUỐC GIA THPT NĂM 2020
Môn thi : Toán
Ngày thi: 02/10/2019
ĐỀ THI CHÍNH THỨC
(Hướng dẫn này có 04 trang)

Bài Sơ lược lời giải Điểm


Bài 1 a, b, c thỏa mãn đề bài ta có:
(5 điể m)
b 2  1 2 b 2  1  2c 2  3b  1  2  3c  1
2 2

0bc  2
c  
2 2 6
2,0
3a  1
2
3a  1 2
x
  6 với x  3a 2  1, y  3b 2  1, z  3c 2  1 .
bc 2
 3b  1  2  3c  1 y  2 z
2 2

Ký hiệu  f  a, b, c   f  a, b, c   f b, c, a   f  c, a, b  ta có:


 x  y  z   6 ta có đpcm.
2
3a 2  1  x   x2  2,0
S   6   
 6 6
bc 2
 y  2z   xy  2 xz  3  xy  yz  zx 

Đẳng thức xảy ra  a  b  c  1 . 1,0


Bài 2 xf  x  xy   xf  x   f  x 2  f  y  , x, y  1
(5 điể m)
Giả sử tồn tại hàm f thỏa mãn đề bài. Ký hiệu P  a; b  là thao tác thay x bởi a , thay y
bởi b .

(1): P  0; 0   f 2  0   0  f  0   0 .
1,0
(1): P 1; y   f  y  1  f 1  f 1 f  y  ,  y 

 nếu f 1  0 thì f  x   0,  x  . Thử lại thỏa mãn.

Xét f 1  a  0  f  x  1  a. f  x   a,  x  .

f  0   0, f 1  a, f  2   a 2  a, f  3  a 3  a 2  a, f  4   a 4  a 3  a 2  a .

1 : P  2;1 : 2 f  4   2 f  2   f  4  f 1


 2  a 4  a3  a 2  a   2  a 2  a    a 4  a3  a 2  a  a 1,0
 a 5  a 4  a 3  a 2  0  a  1

Nếu a  1  f  x  1   f  x   1, x   f  0   0, f 1  1, f  1  1

1 : P  1; 1 : 1 f  0   1 f  1  f 1 f  1  0  2 vô lý nên f 1  a  1 .


f  x  1  f  x   1, x   f  n   n,  n  , f  x  n   f  x   n, x  , n  .

1 : P  x; 1  xf  0   xf  x   f  x 2  f  1 , x   f  x 2   xf  x  , x   2


Thay vào (1):

xf  x 1  y    xf  x   xf  x  f  y  , x, y  1,0
 xf  x 1  y    xf  x  1  f  y   , x, y 
 xf  x 1  y    xf  x  f 1  y  , x, y 
 f  xy   f  x  f  y  , x  0, y 
 f  xy   f  x  f  y  , x, y 

 f  x  0
Thay vào (2) suy ra f 2  x   xf  x  , x   x  : .
 f  x   x 1,0

Nếu b : f  b   0  f  b  1  f  b   1  1  0  b  1  1  b  0

 f  x   x, x  . Thử lại, hàm này thỏa mãn đề bài.


1,0

Vậy có 2 hàm số thỏa mãn đề bài: f  x   0, x  , hoặc f  x   x, x  .

Bài 3
N
(5 điể m)

K
I E

O
H J

B D C
1,0

DB AB 2 c 2
a. Chỉ ra AD và BE là các đường đối trung của tam giác ABC nên   .
DC AC 2 b 2

Gọi H, K là chân các đường vuông góc kẻ từ D và I xuống AB, ta có


1,0
1
BK 2 
BA  BH 
BA BH c
cot ABI      2
 cot B
KI 1 DH DH c
DH hc
2 c2  b2

b 2  c 2 a 2  c 2  b 2 a 2  b 2  3c 2
   .
2S 4S 4S

a 2  b 2  3c 2
Tương tự ta được cot BAJ  nên suy ra hai góc bằng nhau. 1,0
4S

b. Đánh giá được

16S 2  (a  b  c)(a  b  c)(b  c  a)(c  a  b)

  (a  b) 2  c 2  c 2  (a  b) 2    2(a 2  b 2 )  c 2  c 2
1,0
2
1  2(a  b )  c  7c 
2 2 2 2

1
 2(a 2  b2 )  c 2  (7c 2 )  
7

7 2 

Suy ra cot ABI  7 .

a  b
Do đó góc ABI lớn nhất khi  2 hay a : b : c  2 : 2 :1 .
2(a  b )  c  7c 1,0
2 2 2

Bài 4 Gọi un  n  2  là số các tập con của S n  {1; 2;3;...; n} không chứa hai số nguyên dương
(5 điể m)
liên tiếp. Tính trực tiếp u2  3, u3  5 .

Xét S n  2  {1; 2;3;...; n  2} . Ta chia các tập con của S n  2 không chứa hai số nguyên
dương liên tiếp thành 2 loại:

- Loại 1: Các tập con chứa n  2 . Khi đó nó không chứa n  1 nên số tập con loại 1
của S n  2 bằng số các tập con không chứa 2 số nguyên dương liên tiếp của 1,5
S n  2 \ {n  2; n  1}  S n nên bằng u n .
- Loại 2: Các tập con không chứa n  2 . Số tập con loại 2 của S n  2 bằng số tập con
không chứa 2 số nguyên dương liên tiếp của S n  2 \{n  2}  S n 1 và bằng un 1 .
u2  3, u3  5
  un  :  .
un  2  un 1  un , n  2.
Từ đây suy ra: 0,5

5 1 5 5 1 5
n n
3 3 
 n2   n2  , n  2
1
un       
2 5  2  2 5  2  5

1 5 1 5
với   ,  .
2 2
Gọi vn  n  2  là số các tập con của S n  {1; 2;3;...; n} chứa đúng hai số nguyên dương 1,5
liên tiếp. Kiểm tra trực tiếp v2  1, v3  2 .

Xét S n  2  {1; 2;3;...; n  2} . Ta chia các tập con của S n  2 chứa đúng hai số nguyên
dương liên tiếp thành 3 loại:

- Loại 1: chứa n  2 và n  1 , khi đó nó không chứa n . Số tập con loại 1 của S n  2


bằng số tập con không chứa hai số nguyên dương liên tiếp của S n 1 và bằng un 1 .
- Loại 2: chứa n  2 nhưng không chứa n  1 . Số tập con loại 2 bằng số tập con
chứa đúng 2 số nguyên dương liên tiếp của S n 2 \ {n+1;n  2}  S n và bằng vn .
- Loại 3: không chứa n  2 . Số tập loại 3 bằng số tập con chứa đúng 2 số nguyên
dương liên tiếp của S n  2 \ {n  2}  S n 1 và bằng vn 1 .
v2  1; v3  2
  vn  :  (quy ước thêm u1  2, u0  1 )
vn  2  un 1  vn  vn 1 , n  2.

2nun 1   n  1 un  2 1,0
Chứng minh bằng quy nạp được vn  , n  2 .
5

0,5
 2 n     n  1    , n  2 .
1
 
n 1 n 1
Từ đây theo trên ta được vn 
n n

5 5

1 5 1 5
(trong đó   ,  ).
2 2

Chú ý khi chấm:


1. Hướng dẫn chấm này chỉ trình bày sơ lược bài giải. Bài làm của học sinh phải chi tiết, lập luận chặt
chẽ, tính toán chính xác mới được điểm tối đa. Các cách giải khác nếu đúng vẫn cho điểm. Tổ chấm trao
đổi và thống nhất chi tiết nhưng không được quá số điểm dành cho câu, phần đó.
2. Có thể chia điểm thành từng phần nhưng không dưới 0,25 điểm và phải thống nhất trong cả tổ chấm.
Điểm toàn bài là tổng số điểm các phần đã chấm, không làm tròn điểm.
3. Mọi vấn đề phát sinh trong quá trình chấm phải được trao đổi thống nhất trong tổ chấm và ghi vào biên
bản.

SỞ GIÁO DỤC VÀ ĐÀO TẠO QUẢNG NINH

............................. Hết ...........................


SỞ GIÁO DỤC VÀ ĐÀO TẠO KỲ THI CHỌN ĐỘI TUYỂN DỰ THI
HÀ TĨNH HỌC SINH GIỎI QUỐC GIA THPT
NĂM HỌC 2019 − 2020
Môn: TOÁN − Ngày thi 19/9/2019

HƯỚNG DẪN CHẤM


- Mọi cách làm khác đáp án, nếu đúng đều cho điểm tương ứng;
- Các giám khảo thống nhất để chia điểm đến 0,25 hoặc 0,5 (nếu thấy cần thiết).
Câu Nội dung Điểm
Câu 1a Từ giả thiết ta có ∀ε , 0 < ε < 2, ∃N ∈ ℕ unn − 2 < ε ∀n > N 0.5
2điểm
 2 − ε < u < 2 + ε  ln(2 − ε ) < n ln un < ln(2 + ε )
n
n 0.5
1 1
 ln(2 − ε ) < ln un < ln(2 + ε )
n n 0.5
Do đó lim(ln un ) = 0
Vì hàm số y = ln x liên tục nên ln(lim un ) = lim(ln un ) = 0  lim un = 1 0.5
Câu 1b Bổ đề: lim x = a > 0 ⇔ lim n( xn − 1) = ln a
n
n 0.5
3điểm Chứng minh:
Nếu lim xnn = a > 0 , chứng minh tương tự như câu 1a ta có: lim xn = 1 .
Từ đó ta có 0.5
x −1
lim n( xn − 1) = lim n n ln xn = lim n ln xn = lim ln xnn = ln(lim xnn ) = ln a
ln xn
Nếu lim n( xn − 1) = ln a  ln a − ε < n( xn − 1) < ln a + ε
1 1 0.5
 ( ln a − ε ) < xn − 1 < ( ln a + ε )  lim xn = 1
n n
ln xn
 lim n ln xn = lim n( xn − 1) = lim n( xn − 1) = ln a
xn − 1 0.5
 lim ln x = ln a  lim x = a (đpcm)
n
n
n
n

Áp dụng bổ đề ta có: lim n(un − 1) = ln 2; lim n(vn − 1) = ln 3.


 2un + 3vn   2(un − 1) + 3(vn − 1) 
Suy ra lim n  − 1 = lim n  
 5   5  0.5
2 3 2 3
= lim n ( un − 1) + lim n ( vn − 1) = ln 2 + ln 3 = ln 5 2233
5 5 5 5
 2un + 3vn 
n

Áp dụng bổ đề ta có lim   = 23
5 2 3
0.5
 5 
Câu 2 Giả sử 0 ≤ x1 ≤ x2 ≤ .... ≤ x2019 ≤ 1
5điểm Vì hàm số f ( x) = x( x + 1)( x + 2) đồng biến trên [0; +∞) 1.0
nên 0 ≤ f ( x1 ) ≤ f ( x2 ) ≤ .... ≤ f ( x2019 )
Ta có F =  max { f ( xi ); f ( x j )} =  f (xj )
1≤i < j ≤ 2019 1≤i < j ≤ 2019
2019
= f ( x2 ) + 2 f ( x3 ) + 3 f ( x4 ) + ... + 2018 f ( x2019 ) =  (i − 1) f ( xi ) 1.0
i =1

=  (i − 1) ( xi3 + 3 xi2 + 2 xi ) =  (i − 1) xi3 + 3  (i − 1) xi2 + 2  (i − 1) xi (*)


2019 2019 2019 2019

i =1 i =1 i =1 i =1

1
Áp dụng bất đẳng thức Trebusep cho 2 dãy đơn điệu cùng chiều
( x1 , x2 ,...., x2019 ); (0,1, 2,..., n − 1) ta có
2019
1
 (i − 1) x i ( 0 + 1 + 2 + ... + 2018) ( x1 + x2 + ... + x2019 ) = 1004
≥ (1) 1.0
i =1 2019
1
có “=” khi x1 = x2 = ... = x2019 =
2019
Tương tự:
2019
1
 (i − 1) x 2
i ≥ ( 0 + 1 + 2 + ... + 2018 ) ( x12 + x22 + ... + x2019
2
) (2)
i =1 2019
2 0.5
 2019 
  xi 
≥ 1004  i =1  =
1004 1
; có “=” khi x1 = x2 = ... = x2019 =
2019 2019 2019
2019
1
 (i − 1) xi3 ≥ ( 0 + 1 + 2 + ... + 2018 ) ( x13 + x23 + ... + x2019
3
)
i =1 2019
3 1.0
1  2019  1004 1
2  i
≥ 1004 x = 2
(3) có “=” khi x1 = x2 = ... = x2019 =
2019  i =1  2019 2019
Thay (1); (2); (3) vào (*) ta được:
 1 3   1  1 
F ≥ 1004  2
+ + 2  = 1004  + 1  + 2
 2019 2019   2019   2019  0.5
1
có “=” khi x1 = x2 = ... = x2019 =
2019
Câu 3a
2.5
điểm

Gọi O lần lượt là tâm ngoại tiếp tam giác ABC, E là trung điểm CH
1.0
Ta chứng minh được CH = 2OM suy ra OM = EH hay tứ giác MOEH là
hình bình hành hay MH // EO (1)
Mặt khác CH là đường kính của đường ngoại tiếp tam giác CA1B1 nên E là
tâm đường tròn ngoại tiếp tam giác CA1B1 1.0
Do đó EO ⊥ CN, mà HN ⊥ CN suy ra HN // EO (2)
Từ (1) và (2) ta có M, H, N thẳng hàng
Suy ra H là trực tâm tam giác KCM, hay KH ⊥ CM .
Mặt khác HP ⊥ CM nên K, H, P thẳng hàng (đpcm) 0.5

2
Câu 3b Ta có CH là đường kính của đường tròn ngoại tiếp tam giác CA1B1
1.0
2.5 Ta có CPB1 = CA1 B1 = CAB suy ra A, M, P, B1 cùng nằm trên một đường
điểm tròn, tương tự bốn điểm B, A1, P, M cùng nằm trên một đường .
Ta có M là tâm đường tròn ngoại tiếp tam giác ABB1 nên MA=MB1 và
1.0
MPA = MB1 A = MAB1 = MQB
hay AP song song QB.
Tương tự BP song song AQ
Do đó ta có tứ giác APBQ là hình bình hành nên AQ=BP (đpcm) 0.5

Câu 4 Ta sẽ chứng minh bằng quy nạp với bảng (2n+1)(2n+1) (n > 2)
1.0
5 điểm thì số hình chữ L ít nhất cần dùng là (n+1)2
Thật vậy với bảng (2n+1)(2n+1) ô vuông ta xét các ô vuông vị trí (i,j) với i,
j lẻ. Rõ ràng các ô đó chứa số 1 và mỗi hình chữ L chỉ chứa được một ô.
1.0
Mặt khác số các ô vuông vị trí (i,j) với i, j lẻ là (n+1)2.
Vì vậy số hình chữ L cần dùng tối thiểu là (n+1)2
Ta sẽ chứng minh có thể dùng (n+1)2 để phủ được theo yêu cầu của bài
toán.
Khi n=3 ta dễ thấy phủ được bảng ô vuông 7x7 bằng 16 hình chữ L
Cách vẽ như ở bảng bên dưới (thể hiện 8 hình chữ L ở phần trên, phần còn
dưới làm tương tự bằng cách lấy đối xứng qua ô ở vị trí (4;4))
1.0
1 0 1 0 1 0 1
0 1 0 1 0 1 0
1 0 1 0 1 0 1
0 1 0 1 0 1 0
1 0 1 0 1 0 1
0 1 0 1 0 1 0
1 0 1 0 1 0 1

Giả sử với bảng (2n+1)(2n+1) có thể phủ được bởi (n+1)2 hình chữ L thỏa
mãn. Xét bảng (2n+3)(2n+3), chia bảng này thành 4 bảng con đó là
(2n+1)x(2n+1); 2x2; 2x(2n+1) và (2n+1)x2.
Ta thấy mỗi bảng (2n+1)x2 và 2x(2n+1) có thể phủ được bằng n+1 hình
1.0
chữ L, cụ thể như sau:
Đối với hình 2x(2n+1) chia làm 2 phần gồm 1 hình 2x3 và (n-1) hình 2x2
Hình 2x3 phủ bởi 2 hình chữ L; mỗi hình 2x2 phủ bởi 1 hình chữ L, như
vậy có n+1 hình chữ L phủ hết các chữ số 1 của hình 2x(2n+1)
Tương tự cho hình (2n+1)x2
Bảng 2x2 có thể phủ được bằng một hình chữ L.
Do đó theo giả thiết quy nạp thì số hình chữ L có thể dùng để phủ là
1.0
(n+1)2+2(n+1)+1= (n+2)2. Do đó ta có đpcm.
Kết luận k = 10092

HẾT

3
SỞ GIÁO DỤC VÀ ĐÀO TẠO KỲ THI CHỌN ĐỘI TUYỂN
HÀ TĨNH DỰ THI HỌC SINH GIỎI QUỐC GIA THPT
NĂM HỌC 2019 − 2020
Môn: TOÁN − Ngày thi 20/9/2019
HƯỚNG DẪN CHẤM
- Mọi cách làm khác đáp án, nếu đúng đều cho điểm tương ứng;
- Các giám khảo thống nhất để chia điểm đến 0,25 hoặc 0,5 (nếu thấy cần thiết).

Câu Nội dung Điểm


Câu 5a Bổ đề: Nếu m, n là hai số nguyên, p m − n với p là một số nguyên tố có
(4 điểm)
dạng 3k+2 (k là số nguyên) thì p m 2 + n 2 + mn (*)
Chứng minh 0.5
Th1:Nếu tồn tại 1 trong 2 số chia hết cho p thì(*)đúng
Th2: Nếu cả 2 số m, n không chia hết cho p, có:
Nếu p| m + n + mn(1) thì m3 ≡ n 3 (mod p)
3
m
   ≡ 1(mod p)
n
m 1.0
Vì m ≠ n (mod p) và p∤ m, p ∤ n nên ≠ 1 (mod p)
n
2
m
Nếu   = 1(mod p)  m 2 − n 2 ⋮ p ⇔ m + n ⋮ p  (m + n) 2 ⋮ p
n
Kết hợp (1) ta có m.n ⋮ p vô lý
 3= ord ( )  3 | p − 1  p có dạng 3k+1  trái với giả thiết
 (*) đúng trong trường hợp này
Vậy bổ đề được chứng minh.
Ta có: f (x) − f (y) = (x − y)(x 2 + y 2 + xy + 14x + 14y − 2)
Nếu 101 (x − y) thì 101 | ( f (x) − f (y) ) (1) 0.5
Nếu101 ∤ x-y thì 101 | ( f (x) − f (y) ) ⇔ 101 | (x 2 + y 2 + xy + 14x + 14y − 2) 0.5
Lại có 101 ≡ 2 (mod 3).
x 2 + y 2 + xy + 14x + 14y − 2 ≡
≡ (x − 29) 2 + (y − 29) 2 + (x − 29)(y − 29) (mod 101) 0.5
Đặt a = x − 72 , b = y − 29  101 ∤ (a − b)
Vậy theo (*), có101∤ a + b + ab
 101 ∤ x + y + xy + 14x + 14y − 2
 101∤ ( f (x) − f (y) ) nếu x ≠ y (mod 101) (2) 1.0
Từ (1), (2) có đpcm.

Cách khác: Ta có: f (x) = (x − 29) − 25.101x + 101x + c


2 2

f (x) ≡ f (y)(mod101) ⇔ (x − 29)3 ≡ (y − 29)3 (mod101) (1)


Nếu x − 29 ≡ 0(mod101)  y − 29 ≡ 0(mod101)  x − y ≡ 0(mod101)
Nếu (x − 29,101) = 1  (y − 29,101) = 1 , khi đó:

1
(x − 29)100 ≡ 1(mod101)
  (x − 29)100 ≡ (y − 29)100 (mod101) (2)
(y − 29) ≡ 1(mod101)
100

Lại từ (1) ta có: (x − 29) ≡ (y − 29) (mod101)


99 99

 (x − 29)100 ≡ (y − 29)99 (x − 29)(mod101) (3)


Từ (2)và (3) ta có:
(y − 29)100 ≡ (y − 29)99 (y − 29)(mod101)  x ≡ y(mod101)
Từ đó ta có đfcm.
Câu 5b Với mỗi x ∈ {1, 2,...101} , xét f (x), f (x),……, f (x)thì luôn tồn tại g >
3 điểm h là các số tự nhiên thuộc {1, 2,...,102} mà f (x) ≡ f (x) (mod101)
g h

 f h ( f g − h (x) ) ≡ f h (x) (mod101)


Áp dụng câu a, h lần ta có: 1.0
f g −h (x) ≡ x (mod101)
Vậy với mọi i ∈ {1; 2;...;102} thì tồn tại n i để f ni (i) ≡ i (mod 101)
Dễ thấy i ≡ f ni (i) ≡ f 2ni (i) ≡ ... ≡ f kni (i) (mod 101)
1.0
Đặt n = n1 .n 2 .....n101 ta thấy f n (i) ≡ i (mod101), ∀i = 1, 2,...,101 .
Từ đó suy ra: f n (x) ≡ x (mod101), ∀x ∈ ℤ . 1.0
Câu6a Gọi N là trung điểm AQ . Gọi P đối
3.5 điểm xứng M qua H thì BP = MB = MC .
Lại có 1.0
AN = 2MH = MP suy ra NP = AM .
Lại có: ∆BPM cân tại B nên
∠BPM = ∠BMP  ∠BPN = ∠AMC
1.0
Từ đó suy ra: ∆BPN = ∆CMA (c.g.c)
Suy ra:
∠BNP = ∠MAC  ∠BNQ = ∠NAC 0.5
Lại có: BN = AC và QN = NA
⇒ ∆NBQ = ∆ACN (c.g.c)
⇒ ∠NBQ = ∠NCA
1.0
⇒ Tứ giác BDNC nội tiếp
⇒ đfcm.

Câu6b Qua N kẻ đường thẳng vuông góc với AQ cắt (BDC) tại K ≠ N và BQ tại
3.5 điểm Z; 1.0
Mà N là trung điểm AQ nên KN là trung trực AQ
⇒ ∆ZQA cân tại ⇒ ∠ZQA = ∠ZAQ (1)
Mặt khác : ∆BNQ = ∆CAN (theo câu a) ⇒ ∠BQN = ∠ANC (2)
Từ (1) và (2) ta có ∠ZQA = ∠ANC  AZ∥NC 0.75
 ∠ZAD = ∠NCD = ∠NKD ( góc nội tiếp chắn cung ND) 0.75
⇒Tứ giác ZDAK nội tiếp
Khi đó, ∠DKA = ∠DZA = 2∠DQA mà K thuộc trung trực AQ nên K là
tâm đường tròn ngoại tiếp ∆ADQ 1.0
Ta có điều phải chứng minh.

2
Cách Khác
Gọi T là tâm đường tròn ngoai tiếp ∆ADQ thì TN là đường trung trực của
AQTa có: ∠QTN = 180 − ∠QDC = ∠BDC
∠QTD = ∠2QAD = ∠QAD + ∠ANB
Do đó: ∠NTD = ∠QAD + ∠ANB − ∠BDC = ∠DBN
Suy ra: 4 điểm B,D,N,T nằm trên 1 đường tròn  đpcm
Câu7a Ta chỉ ra một cách thỏa mãn đề bài với 5 lần cân.
2.0 điểm Gọi khối lượng bốn viên bi lần lượt là A, B, C, D . 0.5
Lần 1. Cân A và B . Không mất tính tổng quát, giả sử A < B .
Lần 2. Cân C và D . Không mất tính tổng quát, giả sử C < D .
Lần 3. Cân B và D . Không mất tính tổng quát, giả sử B < D . 0.5
Đến đây ta sắp xếp cho ba viên bi A, B, D theo thứ tự tăng dần như sau
A < B < D.
Do đó, ta chỉ còn xác định vị trí của C trong dãy trên là xong. 0.5
Lần 4. Cân C và A . Nếu C<A thì ta có C<A<B<D. Nếu ngược lại, ta tiếp
tục cân.
Lần 5. Cân C và B . Nếu C>B, ta có A<B<C<D. Nếu ngược lại, ta có
A<C<B<D. 0.5
Vậy ta sắp xếp được 4 viên bi theo thứ tự khối lượng tăng dần sau 5 lần
cân.
Câu7b Bổ đề: Cho hai dãy số dương a = ( a1 ,a 2 ,...,a n ) và b = ( b1 , b 2 ,..., b n ) sắp xếp
4.0 điểm theo thứ tự giá trị tăng dần và tất cả 2n số này đều đôi một khác nhau. Sau 1.0
2n − 1 lần so sánh, ta sẽ sắp xếp được 2n số này theo thứ tự giá trị tăng
dần.
Thật vậy:
Bước 1: Ta so sánh a1 và b1 . Không mất tính tổng quát, giả sử rằng a1 < b1 .
Ta viết hai số này lên bảng theo thứ tự tăng dần.
Bước 2: ta so sánh b1 và a 2 .
0.5
TH1: b1 < a 2 ta có dãy số a1 < b1 < a 2 .
TH2: a 2 < b1 ta có dãy số a1 < a 2 < b1 .
Ở các bước tiếp theo, mỗi bước ta sẽ thêm được một số vào dãy tăng dần
này.
Giả sử ta đã có được một dãy tăng dần a1 < ... < a i < b j .
Nếu i = n thì dễ có a1 < ... < a i < b j < b j+1 < ... < b n , ta kết thúc quá trình so
sánh.
Nếu i < n − 1 , ta sẽ so sánh a i +1 và b j .
TH1: a i +1 < b j , ta có dãy a1 < ... < a i < a i +1 < b j .
TH2: a i +1 > b j , ta có dãy a1 < ... < a i < b j < a i +1 .
Trường hợp còn lại a1 < ... < b j < a i thì tương tự ta so sánh a i với b j+1 và 0.5
viết thêm b j+1 vào dãy.
Như vậy sau 2n − 1 lần so sánh, ta có thể sắp xếp 2n số này theo thứ tự
tăng dần.

3
Quay trở lại bài toán.
Ta chứng minh bằng nguyên lí quy nạp.
Với n=2 khẳng định đúng theo câu (a):
Chỉ cần dùng 5 lần cân để sắp xếp 4 viên bi theo thứ tự khối lượng tăng
dần.
Giả sử khẳng định đúng với n = k , ta chứng minh nó cũng đúng với 1.0
n = k + 1.
Thật vậy, theo giả thiết quy nạp:
Lần 1 đến lần thứ ( k − 1) 2k + 1 , ta cân và sắp xếp được các viên bi từ 1 đến
2k theo thứ tự tăng dần : A1 < A 2 < ... < A 2k
Lần thứ ( k − 1) 2k + 2 đến lần thứ 2 ( k − 1) 2k + 1 , ta cân và sắp xếp được
các viên bi từ 2k + 1 đến 2k +1 theo thứ tự tăng dần: B1 < B2 < ... < B2k .
Áp dụng bổ đề, ta chỉ cần 2k +1 − 1 lần cân nữa để sắp xếp hai dãy viên bi
{ } { }
A = A1 , A 2 ,..., A 2k và B = B1 , B2 ,..., B2k này theo thứ tự khối lượng
tăng dần. 1.0
Vậy ta cần dùng tổng cộng: 2 ( k − 1) 2k + 1 + 2k +1 − 1 = k.2k +1 + 1 lần cân.
Do đó khẳng định đúng với n = k + 1 .
Theo nguyên lí quy nạp khẳng định đúng với mọi n = 2,3,...

4
SỞ GIÁO DỤC VÀ ĐÀO TẠO KỲ THI CHỌN ĐỘI TUYỂN
HÀ TĨNH DỰ THI HỌC SINH GIỎI QUỐC GIA THPT
NĂM HỌC 2019 - 2020
ĐỀ THI CHÍNH THỨC
(Đề thi có 1 trang, gồm 3 câu) Môn thi: TOÁN
Ngày thi: 20/9/2019
Thời gian làm bài: 180 phút

Câu 5 (7 điểm) Cho đa thức f (x) = x 3 + 14x 2 − 2x + 1 .


a) Chứng minh với mọi x, y nguyên dương thì:
f (x) − f (y) chia hết cho 101 khi và chỉ khi x − y chia hết 101.
b) Chứng minh tồn tại số nguyên dương n thỏa mãn: f n (x) − x chia hết cho 101 với
mọi x nguyên, với f n (x) = f (f (...f (x)...)) có n cặp dấu ngoặc đơn.

Câu 6 (7 điểm) Cho tam giác ABC nhọn với AB < AC và M là trung điểm BC . H là
hình chiếu của B lên AM . Lấy điểm Q trên tia đối của tia AM sao cho AQ = 4MH,
gọi D là giao điểm của AC và BQ .
a) Chứng minh rằng đường tròn ngoại tiếp tam giác DBC đi qua trung điểm AQ .
b) Chứng minh rằng tâm đường tròn ngoại tiếp tam giác ADQ nằm trên đường tròn
ngoại tiếp tam giác DBC .

Câu 7 (6 điểm) Cho một số viên bi có khối lượng khác nhau đôi một và một cái cân thăng
bằng. Biết rằng chiếc cân này không cho phép đo chính xác khối lượng của viên bi
mà mỗi lần cân chỉ cho phép so sánh khối lượng của hai viên bi bất kỳ. Mục tiêu
cuối cùng là có thể sắp xếp các viên bi này theo thứ tự khối lượng tăng dần bằng
một số lần cân hữu hạn.
a) Chứng minh rằng với 4 viên bi bất kỳ thì chỉ cần sử dụng 5 lần cân.
b) Chứng minh rằng với 2n viên bi bất kỳ thì chỉ cần sử dụng ( n − 1) 2n + 1 lần cân
(n = 2,3,...) .

−−−−−−−−−−−−HẾT−
−−−−−−−−−−−−

• Thí sinh không được sử dụng tài liệu và máy tính cầm tay;
• Giám thị không giải thích gì thêm.

Họ và tên thí sinh…………………………………….Số báo danh…………


SỞ GIÁO DỤC VÀ ĐÀO TẠO KỲ THI CHỌN ĐỘI TUYỂN
HÀ TĨNH DỰ THI HỌC SINH GIỎI QUỐC GIA THPT
NĂM HỌC 2019 − 2020
ĐỀ THI CHÍNH THỨC

(Đề thi có 01 trang, gồm 4 câu) Môn thi: TOÁN


Ngày thi: 19/9/2019
Thời gian làm bài: 180 phút

Câu 1 (5 điểm) Cho các dãy số (un ); (vn ) thỏa mãn: lim unn = 2; lim vnn = 3 ; un , vn ≠ 1 ∀n
a) Chứng minh lim un = 1 .

 2u + 3vn 
n

b) Tìm lim  n  .
 5 
Câu 2 (5 điểm) Cho hàm số f ( x) = x( x + 1)( x + 2) , xét 2019 số thực không âm
x1 ; x2 ;...x2019 thay đổi thỏa mãn: x1 + x2 + ... + x2019 = 1 . Với i, j ∈ {1;2;...2019} , đặt

F=  max { f ( xi ); f ( x j )} . Tìm giá trị nhỏ nhất của F.


1≤i < j ≤ 2019

Câu 3 (5 điểm) Cho tam giác ABC nhọn không cân. Các đường cao AA1, BB1 của tam
giác ABC cắt nhau tại H. Đường tròn ngoại tiếp tam giác ABC cắt đường tròn
ngoại tiếp tam giác A1B1C tại N (khác C). Gọi M là trung điểm AB, K là giao điểm
của CN và AB. Đường thẳng CM cắt đường tròn ngoại tiếp tam giác CA1B1 tại
điểm thứ hai P, cắt đường tròn ngoại tiếp tam giác ABC tại điểm thứ hai Q.
a) Chứng minh K, H, P thẳng hàng
b) Chứng minh AQ=BP.
Câu 4. (5 điểm) Cho bảng ô vuông 2019x2019, ta điền vào các ô vuông đơn vị của bảng
các số 0, 1 xen kẻ nhau. Biết bốn ô vuông ở bốn góc của bảng đều được điền số 1.
Tìm số k nhỏ nhất các hình chữ L ( hình vuông 2x2 bỏ đi một ô vuông bất kỳ) sao
cho có thể phủ tất cả các ô vuông chứa số 1 bởi k hình chữ L không chồng lên
nhau.

−−−−−−−−−−−−HẾT−
−−−−−−−−−−−−

• Thí sinh không được sử dụng tài liệu và máy tính cầm tay;
• Giám thị không giải thích gì thêm.

Họ và tên thí sinh…………………………………….Số báo danh……………


SỞ GIÁO DỤC VÀ ĐÀO TẠO KỲ THI CHỌN ĐỘI TUYỂN
HẢI PHÒNG DỰ THI HỌC SINH GIỎI QUỐC GIA
NĂM HỌC 2019 - 2020
ĐỀ CHÍNH THỨC
ĐỀ THI MÔN: TOÁN HỌC
(Ngày thi thứ nhất 28/9/2019)
(Đề thi có 01 trang, gồm 04 bài) Thời gian làm bài 180 phút, không kể thời gian giao đề

Bài 1: (5.0 điểm) Tìm tất cả các hàm số f :  sao cho:

f  f  x    x  y  f  y    4 x  2  x  y  f  y  x, y  .

Bài 2: (5.0 điểm) Cho số thực a không âm và dãy  un  xác định như sau:

1 1
u0  a, un1  un2  un  1 n  .
8 4
Tìm a để dãy  un  có giới hạn hữu hạn và tính giới hạn đó.

Bài 3: (5.0 điểm) Cho một dãy gồm 100 ô vuông. Trong mỗi ô vuông ta điền một trong bốn chữ
số 2, 0, 1, 9. Hỏi có bao nhiêu cách điền sao cho tổng các số trong 100 ô vuông là một số chia
hết cho 4.

Bài 4: (5.0 điểm) Cho ABC là tam giác nhọn  AB  AC  ngoại tiếp đường tròn  I  .
D, E , F lần lượt là các tiếp điểm của đường tròn  I  với BC , CA, AB . AD cắt đường tròn
I  tại Q  Q  D  . Tiếp tuyến tại Q của đường tròn  I  cắt EF tại S .

a) Chứng minh bốn điểm S , D, B, C thẳng hàng và theo thứ tự lập thành hàng điểm điều hoà.

b) Gọi K là giao điểm của EF và DI . AK cắt BC tại M . Kẻ CH  AB  H  AB  .


Chứng minh MH tiếp xúc với đường tròn ngoại tiếp tam giác SHD.

----------- HẾT -----------


(Thí sinh không sử dụng tài liệu, cán bộ coi thi không cần giải thích gì thêm)

Họ và tên: ............................................................ Số báo danh .........................................................

Cán bộ coi thi số 1: .............................................. Cán bộ coi thi số 2: ..............................................

Trang 1/3
SỞ GIÁO DỤC VÀ ĐÀO TẠO KỲ THI CHỌN ĐỘI TUYỂN
HẢI PHÒNG DỰ THI HỌC SINH GIỎI QUỐC GIA
NĂM HỌC 2019 - 2020
ĐỀ CHÍNH THỨC HƯỚNG DẪN CHẤM MÔN: TOÁN
(Hướng dẫn gồm 03 trang) Ngày thi thứ nhất: 28/09/2019

Bài Nội dung Điểm


f  f  x    x  y  f  y    4 x  2  x  y  f  y  , x, y  (*)
1.0
 
Từ (*) cho y  x ta có f f  x   4 x suy ra f là song ánh

1   
Khi đó f  0   f  4.0   f f f  0   4 f  0   f  0   0

    
(5.0 Từ (*) cho x  0, y  1 ta có f f 1  2 f 1 , kết hợp f f 1  4 suy ra
điểm)
  2.0


f 1  2 . Do đó f  2   f f 1  4 . 
 
Từ (*) cho y  1 ta có f f  x   2  x  1  4 x  4  x  1  4  f  2 
2.0
Mà f là song ánh suy ra f  x   2  x  1  2  f  x   2 x x  (thử lại: đúng)
1 1 h  2
+ Ta có un  0 n và nếu lim un  h thì h  h 2  h  1   (*)
n 8 4 h  4
1 2 1
Đặt f  x   x  x  1 thì un1  f  un 
8 4
1
Có f '  x    x  1  0, x  0  f  x  là hàm đồng biến trên [0, +)
4
Bảng biến thiên:
1.0
x 0 2 4 +
+
2
f(x) 4
(5.0
2
điểm) 1

+ Xét u1  u0  f  u0   u0  a 2  a  1  a   a  2  a  4  .
1 1 1
8 4 8
Do đó có các trường hợp sau:
a) Nếu a   0, 2  u1  u0  0  u1  u0  f  u1   f  u0   u2  u1      un1  un
Mặt khác từ bảng bt ta có: x  0,2   f  x   0,2  un  0,2 n  (un) là dãy
tăng và bị chặn nên nó hội tụ và kết hợp với (*) ta có: lim un  2 2.0
n
b) Nếu a   2; 4   u1  u0  0  u1  u0  f  u1   f  u0   u2  u1  ...  un1  u n
Mặt khác từ bảng bt ta có: x   2;4   f  x    2;4   un   2;4  n  (un) là dãy
Trang 2/3
giảm và bị chặn nên nó hội tụ và kết hợp với (*) ta có: lim un  2
n
c) Nếu a  4 thì un = 4 n nên lim un  4 2.0
n

d) Nếu a  4  u1  u0  0  u1  u0  f  u1   f  u0   u2  u1  ...  un1  un


Mặt khác từ bảng bt ta có: x  4  f  x   4  un  4 n   un  là dãy tăng và
nếu nó hội tụ thì lim un  4 (mâu thuẫn với (*)), do đó  un  không hội tụ.
n
Ta xét bài toán tổng quát có n ô vuông; kí hiệu an , bn , cn , d n lần lượt là số cách điền
vào n ô vuông sao cho tổng các số chia 4 dư 0, 1, 2, 3.
an 1  an  cn  2d n
 b  b  d  2a 2.0

Ta có:  n 1 n n n

 cn 1  cn  an  2bn
 d n 1  d n  bn  2cn
3
Suy ra an 1  cn 1  2d n  2bn  4cn 1  4an 1
(5.0
điểm) a2 n  c2 n   4 n 1  a2  c2    4 n 1  2  6    4 n 1.5

 a2 n1  c2 n1   4   a1  c1    4  1  1  0
n n

Lại có an  bn  cn  d n  4  an 1  cn 1  2.4  
n n
 1
2

a2 n   4   2.4
n 2 n 1

 a2 n1  42 n 1.5

Vậy số cách điền thỏa mãn đề bài là a100  299  499 .


A

Q E
K

F
P I
H

4
(5.0 B D M C
S
điểm)
a) Tứ giác DEQF điều hoà nên các tiếp tuyến tại D và Q của đường tròn  I  và
1.0
đường chéo EF đồng quy tại S . Suy ra S , B, C , D thẳng hàng.
DB EC FA
Xét ABC có    1  AD, BE , CF đồng quy (định lý Ceva).
DC EA FB 1.0
Suy ra S , D, B, C lập thành hàng điểm điều hoà (1).
b) Ta sẽ c/m M là trung điểm BC . Xét BFS , cát tuyến AKM :
AF MB KS MB AB KF 2.0
   1 (định lý Menelauyt)    (2).
AB MS KF MS AF KS

Trang 3/3
MC AC KE
Hoàn toàn tương tự    (3).
MS AE KS
MB MC  AB KF   AC KE  MB AB KF
Từ (2) và (3)  :   :     (4).
MS MS  AF KS   AE KS  MC AC KE
KF KF KE sin FIK sin EIK sin FIK sin ABC AC
Lại có   :  :    (5).
KE KI KI sin IFK sin IEK sin EIK sin ACB AB
MB
(4) và (5)   1  MB  MC.
MC
(1)  MD.MS  MB 2  MH 2  MH tiếp xúc với đường tròn ngoại tiếp SHD . 1.0

---------- HẾT ----------

Lưu ý khi chấm bài:


- Đáp án chỉ trình bày một cách giải bao gồm các ý bắt buộc phải có trong bài làm của
học sinh. Khi chấm nếu học sinh bỏ qua bước nào thì không cho điểm bước đó.
- Nếu học sinh giải cách khác, giám khảo căn cứ các ý trong đáp án để cho điểm.
- Trong bài làm, nếu ở một bước nào đó bị sai thì các phần sau có sử dụng kết quả sai đó
không được điểm.
- Học sinh được sử dụng kết quả phần trước để làm phần sau.
- Trong lời giải bài hình, nếu học sinh không vẽ hình thì cho tối đa một nửa số điểm. Nếu
học sinh vẽ sai hình thì không cho điểm.
- Điểm toàn bài tính lẻ đến 0,5 và không làm tròn.

Trang 4/3
SỞ GIÁO DỤC VÀ ĐÀO TẠO KỲ THI CHỌN ĐỘI TUYỂN
HẢI PHÒNG DỰ THI HỌC SINH GIỎI QUỐC GIA
NĂM HỌC 2019 - 2020
ĐỀ CHÍNH THỨC
ĐỀ THI MÔN: TOÁN HỌC
(Ngày thi thứ hai 29/9/2019)
(Đề thi có 01 trang, gồm 04 bài) Thời gian làm bài 180 phút, không kể thời gian giao đề

Bài 1: (5.0 điểm) Xác định các đa thức P  x  , Q  x  hệ số thực thỏa mãn đồng thời hai điều
kiện sau:

i) Q  x  khác đa thức không và deg Q  x   2.

 
ii) P x3  1  x3 P  x  1 .  P  x  1  4  x 6Q  x  x  .

Bài 2: (5.0 điểm) Cho tam giác ABC nhọn (AB < AC); các đường cao AD, BE, CF đồng quy tại
H. Đường thẳng EF cắt các đường thẳng AH, BC lần lượt tại L và G. Gọi M là trung điểm của
BC. AM cắt GH tại I; LI cắt AG tại K.
a) Chứng minh bốn điểm E, K, F, I cùng thuộc một đường tròn.
b) Đường tròn ngoại tiếp tam giác LID cắt GH tại J  J  I  . Chứng minh J nằm trên
đường trung trực của LD.
Bài 3: (5.0 điểm)
a) Tìm tất cả các số tự nhiên a thoả mãn 3a  1 và 4a  1 đều là những số chính phương.
b) Chứng minh nếu số tự nhiên a thoả mãn 3a  1 và 4a  1 đều là những số chính phương thì
a  a  4  13.

Bài 4: (5.0 điểm)


Cho X là một bát giác đều tâm O. Gọi A là tập tất cả các đỉnh
của X và các giao điểm của hai đường chéo bất kì của X . Gọi
B là tập 8 điểm thuộc A không trùng O và gần O nhất (hình
vẽ). Gọi Y là tập tất cả các cạnh của X và các đoạn thẳng nối
hai điểm thuộc A kề nhau trên một đường chéo bất kì của X .
Mỗi điểm thuộc A được tô bởi một trong hai màu xanh hoặc đỏ. O
Có tất cả 26 điểm đỏ. Biết rằng O được tô đỏ, hai trong số
những điểm đỏ là đỉnh của X , ba trong số những điểm đỏ
thuộc B . Các đoạn thẳng thuộc Y được tô màu theo quy tắc:
nếu đoạn thẳng có hai đầu mút đỏ thì nó được tô màu đỏ, nếu
hai đầu mút xanh thì nó được tô màu xanh, nếu một đầu mút đỏ
và một đầu mút xanh thì nó được tô màu vàng. Biết rằng có 20
đoạn thẳng trong Y màu vàng. Hỏi có bao nhiêu đoạn thẳng
trong Y màu xanh?

----------- HẾT -----------


(Thí sinh không sử dụng tài liệu, cán bộ coi thi không cần giải thích gì thêm)

Trang 1/3
SỞ GIÁO DỤC VÀ ĐÀO TẠO KỲ THI CHỌN ĐỘI TUYỂN
HẢI PHÒNG DỰ THI HỌC SINH GIỎI QUỐC GIA
NĂM HỌC 2019 - 2020
ĐA CHÍNH THỨC HƯỚNG DẪN CHẤM MÔN: TOÁN
(Hướng dẫn gồm 03 trang) Ngày thi thứ hai: 29/09/2019

Bài Nội dung Điểm


P  x  1  x P  x  1 .  P  x  1  4  x 6Q  x  (1).
3 3

Dễ thấy P  x  khác đa thức hằng. Thay x  0 vào (1): 1.0


P  1  0  P  x    x  1 P1  x   n  , P1  x   x  , P1  1  0  .
n *

Ta có deg P  x   3 , vì nếu deg P  x   4 thì deg x 6Q  x   3deg P  x   12


1.0
 deg Q  x   12  6  6 , mâu thuẫn. Vậy 3  deg P  x   n.
TH1: n  1 . (1)  x3 P1  x3  1  x 4 P1  x  1 .  P  x  1  4  x 6Q  x 
 P1  x3  1  xP1  x  1 .  P  x  1  4  x 3Q  x  (2) 1.0
Thay x  0 vào (2): P1  1  0 , mâu thuẫn, loại n  1 .
TH2: n  2 . Ta có deg P1  x   deg P  x   2  1  P1  x   a hoặc P1  x   ax  b .
1 + Nếu P1  x   a  a  0  . (1)  ax  a.  a  x  2   4  xQ  x  (3)
2

(5.0  
điểm) Thay x  0 vào (3): 4a  4  0  a  1  P  x     x  12 , Q  x    x  5 .
+ Nếu P1  x   ax  b  a  0; a  b  . 1.0
(1)   ax3  a  b  x   ax  a  b  .  ax  a  b  x  2   4  xQ  x  (4)
2
 
So sánh hệ số gắn với x suy ra a  a  0  a  1 .
4 2

Thay x  0 vào (4): b  2  P  x    x  2  x  1 , Q  x   9 x  3 .


2

TH3: n  3 , P  x   a  x  1  a  0  .
3

ax9  ax6  a  x  2   4  x 6Q  x   Q  x   ax3  a 2 x3  6a 2 x 2  12a 2 x  8a 2  4a ,


3
 
loại. 1.0
Vậy P  x     x  1 , Q  x    x  5 hoặc P  x    x  2  x  1 , Q  x   9 x  3 , thử
2 2

lại thỏa mãn bài toán.

Trang 2/3
A

K E
L
F I
H
J
N

G C
B D M

a) Tứ giác BFEC nội tiếp đường tròn  1  tâm M (đường kính BC)
Theo định lí Brocard suy ra M là trực tâm của tam giác AHG  GH  AM
Giả sử MH vuông góc AG tại K’. Gọi   2  là đường tròn đường kính GM,   3  là
đường tròn đường kính AH.
+ Tứ giác AEHF nội tiếp   3  và BFEC nội tiếp  1  nên EF là trục đẳng phương 1.0
2
(5.0 của  1  và   3  (3)
điểm) + Tứ giác GK’IM nội tiếp   2  và AK’HI nội tiếp   3  nên K’I là trục đẳng phương
của   2  và   3  (4).
+ Lại có AK '. AG  AH . AD  AE. AC  PA/ 2   PA/ 1   A thuộc trục đẳng
phương của hai đường tròn  1  và   2  ; lại có AD  BC hay AD vuông góc với
đường nối tâm của  1  và   2  , do đó AD là trục đẳng phương của hai đường tròn
1.0
này (5).
Từ (3),(4) và (5) suy ra các đường thẳng EF, K’I, AD đồng quy. Do EF cắt AD tại L
nên có I, L, K’ thẳng hàng, suy ra K '  K . Do đó E, K, F, I cùng thuộc đường tròn
đường kính AH.
b) Hạ JN vuông góc LD tại N. Gọi N’ là trung điểm của LD.
1.0
Có JNA  JIA  900 nên tứ giác JNIA nội tiếp, suy ra HA.HN  HJ .HI (1).
Có G  AHEC   1 nên  AHLD   1 , N’ là trung điểm của LD nên theo hệ thức
1.0
Maclaurin có HA.HN '  HL.HD (2).
Tứ giác LIDJ nội tiếp nên HI .HJ  HL.HD (3)
Từ (1),(2) và (3) có HA.HN  HA.HN '  N  N ' , suy ra J nằm trên trung trực của 1.0
LD.
a) Do  3a  1;4a  1  1 , nên gt   3a  1 4a  1  x 2 1 x  .
1.0
1   24a  7   48 x 2  1 . Đặt y  24a  7  y 2  48 x 2  1 (PT Pell loại 1).
2

3
   7  
m m
(5.0 7  48 48
điểm)  y  m   . 1.0
2
Chú ý rằng y  7  24a 24  m  2n  1 n  . 1.0

Trang 3/3
   
2 n 1 2 n 1
7  48  7  48  14
Vậy a   n   thoả mãn đề bài (2).
48
   
2 n 1 2 n 1
7  48  7  48  14
b) Xét dãy un  n  
48
7 7  7   7 
 un  là dãy truy hồi tuyến tính: un 2   194  un1     un   1.0
24 24  24   24 
 un  2  194un1  un  56  un 2  un1  un  195un1  56  56  mod13
 un 2  un1  un  un 3  un  2  un 1  mod13  un  un 3  mod13 .
Có u0  0  mod13 ; u1  56  4  mod13 ; u2  195.56  0  mod13 
 a  0  mod13 1.0
u3n  0  mod13 ; u3n 1  4  mod13 ; u3n  2  0  mod13     đpcm.
 a  4  mod13
Ta có: A  4.8  3.8  1  57, Y  14.8  3.8  136, B  8. 1.0
Gọi số đoạn thẳng trong Y được tô đỏ là a.
Ta gọi số lần xuất hiện điểm đỏ với vai trò là đầu mút của một đoạn thẳng trong Y
1.5
là N và ta đếm N bằng hai cách:
4 + Có 20 đoạn thẳng vàng và a đoạn thẳng đỏ nên N  20  2a (1)
(5.0 + Mỗi đỉnh của X là đầu mút chung của 7 đoạn trong Y, mỗi điểm thuộc B là đầu
điểm) mút chung của 6 đoạn trong Y, O là đầu mút chung của 8 đoạn trong Y, tất cả các
điểm còn lại mỗi điểm đều là đầu mút chung của 4 đoạn trong Y. 1.5
Từ giả thiết suy ra: N  8  2.7  3.6   26  1  2  3 .4  120 (2)
Từ (1) và (2) suy ra: 20  2a  120  a  50
1.0
Vậy số đoạn thẳng trong Y được tô xanh là: 136  20  50  66.

---------- HẾT ----------

Lưu ý khi chấm bài:


- Đáp án chỉ trình bày một cách giải bao gồm các ý bắt buộc phải có trong bài làm của
học sinh. Khi chấm nếu học sinh bỏ qua bước nào thì không cho điểm bước đó.
- Nếu học sinh giải cách khác, giám khảo căn cứ các ý trong đáp án để cho điểm.
- Trong bài làm, nếu ở một bước nào đó bị sai thì các phần sau có sử dụng kết quả sai đó
không được điểm.
- Học sinh được sử dụng kết quả phần trước để làm phần sau.
- Trong lời giải bài hình, nếu học sinh không vẽ hình thì cho tối đa một nửa số điểm. Nếu
học sinh vẽ sai hình thì không cho điểm.
- Điểm toàn bài tính lẻ đến 0,5 và không làm tròn.

Trang 4/3
SỞ GIÁO DỤC VÀ ĐÀO TẠO KỲ THI CHỌN ĐỘI TUYỂN HỌC SINH GIỎI THPT
THÀNH PHỐ HỒ CHÍ MINH NĂM HỌC 2019 - 2020
MÔN THI: TOÁN
Ngày thi thứ nhất: 18/9/2019
ĐỀ CHÍ NH THỨC
Thời gian làm bài: 180 phút (không kể thời gian phát đề)

Bài 1. (5 điểm)
1 un 1
a) Cho dãy số un xác định bởi: u1 ;u 1 với n 1, 2, 3,...
3 n 1
un2 1

Chứng minh dãy số un có giới hạn hữu hạn khi n và tìm giới hạn đó.

b) Cho các số dương a, b, c, d thỏa mãn a 2 b2 c2 d2 1. Chứng minh rằng


2
4(1 a )(1 b) (c d) .

Bài 2. (5 điểm)

Tìm tất cả các hàm số f : liên tục tại 0 thoả mãn:


f (2018x ) f (2019x ) 2020x, x .

Bài 3. (5 điểm)
Cho tam giác ABC có ba góc nhọn, nội tiếp đường tròn O , có trực tâm H và
AB AC . Lấy điểm T A trên đường tròn O sao cho AT song song với BC . Giả sử
AH cắt BC tại K và TH cắt O tại điểm D thuộc cung nhỏ BC . Gọ i L là trung điể m
củ a HT .
a) Chứng minh các điểm A, L,O, K , D cùng nằm trên mọ t đường tròn.
b) Gọi P là giao điểm thứ hai của AO với O . Đường thẳng đi qua H và song song với
BC cắt đường thẳng PD tại X . Chứng minh XA là tiếp tuyến của đường tròn O .

Bài 4. (5 điểm)
Cho đa thức hệ số thực P (x ) có bậc 2019 và hệ số bậc cao nhất bằng 1. Biết rằng P (x ) có
đúng 2019 nghiệm thực phân biệt không phải là số nguyên. Giả sử mỗi đa thức
P (2x 2 4x ) và P (4x 2x 2 ) đều có đúng 2692 nghiệm thực phân biệt.
a) Hỏi có bao nhiêu nghiệm của P (x ) thuộc khoảng ( 2;2) ?
b) Chứng minh rằng tồn tại 3 đa thức cùng bậc A(x ), B(x ),C (x ) có hệ số thực sao cho
A(x )B(x )C (x ) P (x ), x và B(x ) A(x )C (x ), x ( 1;1).

--- HẾT ---


SỞ GIÁO DỤC VÀ ĐÀO TẠO KỲ THI CHỌN ĐỘI TUYỂN HỌC SINH GIỎI THPT
THÀNH PHỐ HỒ CHÍ MINH NĂM HỌC 2019 - 2020
MÔN THI: TOÁN
Ngày thi thứ nhất: 18/9/2019
ĐÁP ÁN CHÍ NH THỨC
Thời gian làm bài: 180 phút (không kể thời gian phát đề)

Bài 1. (5 điểm)
1 un 1
b) Cho dãy số un xác định bởi: u1 ;u 1 với n 1, 2, 3,...
3 n 1
un2 1

Chứng minh dãy số un có giới hạn hữu hạn khi n và tìm giới hạn đó.

Giải.
Gọi P (n ) : 1 un 0. Ta sẽ chứng minh P (n ) đúng với n 1, 2, 3,...

Ta có P (1) đúng. Giả sử P (k ) đúng (k 1) , tức là 1 uk 0.

uk 1
Khi đó 0 1 0 uk 1
1 1, tức là P (k 1) đúng.
uk2 1

Theo nguyên lý quy nạp ta có P (n ) đúng với n 1, 2, 3,... 1 điểm

* un 1 *
Do 1 un 0, n nên un 1
1 un 1 1 un , n . 1 điểm
2
u n
1
Cách 1. Dãy số un giảm và bị chặn dưới nên có giới hạn hữu hạn.
1 L 1
Gọi lim un L thì 1 L . Ta có L 1 L 1. 1 điểm
3 L2 1
1 10
Cách 2. Ta có 1 un u1 un2 1 ; n *
.
3 3
1 3 *
un 1
1 un 1 un 1 với mọi n .
2
u n
1 10
2 n 1
3 3 3
0 un 1 un 1
1 un 2
1 u1 1 , n 1
10 10 10
n 1
3
Mà lim u1 1 0 lim un 1 0.
10
Suy ra lim un 1. 1 điểm
b) Cho các số dương a, b, c, d thỏa mãn a 2 b2 c2 d2 1. Chứng minh rằng
4(1 a )(1 b) (c d )2 .
Giải. Bất đẳng thức cần chứng minh tương đương với
4(1 a )(1 b) 1 a2 b2 2cd
a2 b2 4ab 4(a b) 3 2cd .

Do 2cd c2 d2 1 a2 b2 1 điểm

nên bất đẳng thức đã cho sẽ được chứng minh nếu


a2 b2 4ab 4(a b) 3 1 a2 b 2. (1)

Tuy nhiên điều này hiển nhiên đúng do


(1) 2a 2 2b 2 4ab 4(a b) 2 0 2(a b 1)2 0. 1 điểm
Bài 2. (5 điểm)

Tìm tất cả các hàm số f : liên tục tại 0 thoả mãn:


f (2018x ) f (2019x ) 2020x, x .
Giải.
Cho x 0 f (0) 0. 0,5 điểm
u1 x,
Lấy x . Ta xét dãy số sau: 2018
un u , n 1.
1
2019 n
n 1
2018 *
Dễ thấy un x, n nên lim un 0. 1 điểm
2019
2020 2020 2020
Ta có: f (u1 ) f (u2 )u1, f (u2 ) f (u 3 ) u ,. . ., f (un ) f (un 1 ) u .
2019 2019 2 2019 n
Cộng và trừ các đẳng thức trên (vế theo vế) ta được:
n
2018
1
2020 2020 2019
f (u1 ) ( 1)n 1 f (un 1 ) [u u2 ... ( 1)n 1un ]= u. . 2 điểm
2019 1 2019 1 2018
1
2019
2020
Lấy giới hạn hai vế kết hợp với f liên tục tại 0 , suy ra f (x ) x, x . 1 điểm
4037
2020
Thử lại thỏa. Vậy f (x ) x, x . 0,5 điểm
4037
Bài 3. (5 điểm)
Cho tam giác ABC có ba góc nhọn, nội tiếp đường tròn O , có trực tâm H và
AB AC . Lấy điểm T A trên đường tròn O sao cho AT song song với BC . Giả sử
AH cắt BC tại K và TH cắt O tại điểm D thuộc cung nhỏ BC . Gọ i L là trung điể m
củ a HT .
a) Chứng minh các điểm A, L,O, K , D cùng nằm trên mọ t đường tròn.
b) Gọi P là giao điểm thứ hai của AO với O . Đường thẳng đi qua H và song song với
BC cắt đường thẳng PD tại X . Chứng minh XA là tiếp tuyến của đường tròn O .

A T
I

X F O
E
H

B
K C
D

S P

Giải. a) Gọi S là giao điể m của AH với (O ). Vì AT BC nên suy ra SAT 900 , do đó
S ,O,T thẳng hàng. Theo tính chất đối xứng quen thuộc của trực tâm ta có KS KH .
Hơn nữa, HD.HT HA.HS HD.2HL HA.2HK HD.HL HA.HK tức là tứ giác
ALKD nội tiếp (1). 1 điểm

Ta có ALD 2ATD AOD nên tứ giác ALOD nội tiếp (2). Từ (1), (2) suy ra
A, L,O, K , D nằm trên cùng đường tròn . 1 điểm

b) Cách 1. Gọi giao điểm của DP và tiếp tuyến tại A của O là X .

1 1 1 1
Ta có AX D DAP sđDS sđSP sđDS sđAT DHK . 2 điểm
2 2 2 2

Suy ra tứ giác AHDX nội tiếp nên AHX ADX 90 hay X X , đpcm. 1 điểm

Cách 2. Gọi E , F là giao của XH với AC , AB và I , J là giao của BH ,CH với O . Gọi
P là giao của IE với O . Dễ thấy H , I đối xứng qua AC , dễ thấy rằng

ACB AHI ; BAH CAO nên ta có


1 1
sđAI sđCP ' AEI AEH ACB ABH BAH ABH CAO
2 2
1 1
sđAI sđCP , suy ra sđCP ' sđCP tức là P ' P tức là P , E , I thẳng hàng.
2 2
Tương tự, suy ra J , F , P thẳng hàng. 1 điểm
1 1
Do đó EPF sđIA sđJA ABH ACH 1800 2A (3).
2 2
1 1
Mặt khác BDH sđBT sđAC ABC AFE suy ra BDHF nội tiếp, tương tự
2 2
CDHE nội tiếp. Suy ra, EDF EDH FDH ECH FBH 1800 2A (4).

Từ (3) và (4) suy ra EPF EDF tức là tứ giác EFDP nội tiếp. 1,5 điểm
Vì EF || BC nên AEF tiếp xúc với O tại A. Xét ba đường tròn AEF , O , EFDP
có ba trục đẳng phương là: tiếp tuyến tại A của (O ), EF , PD. Vậy X là tâm đẳng phương
nghĩa là XA là tiếp tuyến của O . 0,5 điểm

Bài 4. (5 điểm)
Cho đa thức hệ số thực P (x ) có bậc 2019 và hệ số bậc cao nhất bằng 1. Biết rằng P (x ) có
đúng 2019 nghiệm thực phân biệt không phải là số nguyên. Giả sử mỗi đa thức
P (2x 2 4x ) và P (4x 2x 2 ) đều có đúng 2692 nghiệm thực phân biệt.
a) Hỏi có bao nhiêu nghiệm của P (x ) thuộc khoảng ( 2;2) ?
b) Chứng minh rằng tồn tại 3 đa thức cùng bậc A(x ), B(x ),C (x ) có hệ số thực sao cho
A(x )B(x )C (x ) P (x ), x và B(x ) A(x )C (x ), x ( 1;1).
Giải. a) Đặt P (x ) (x a1 )(x a2 ) (x a2019 ) với a1, a2 , , a2019 là nghiệm của P (x ) . Khi
đó P (2x 2 4x ) 0 2x 2 4x ai 0 với 1 i 2019. 1 điểm
Phương trình này không thể có đúng 1 nghiệm do 16 8ai 0.
Suy ra mỗi phương trình có 0 hoặc 2 nghiệm. Để ý rằng P (2x 2 4x ) có 2692 nghiệm
2692
phân biệt nên có tất cả 1346 phương trình có 2 nghiệm, tức là có đúng
2
1346 số ai 2 và 673 số ai 2. 1 điểm
Một cách tương tự thì P (4x 2x 2 ) 0 2x 2 4x ai 0 nên 16 8ai với
1 i 2019. Suy ra có 1346 số ai 2 và 673 số ai 2.
Suy ra có tất cả 1346 1346 2019 673 nghiệm ai ( 2;2). 1 điểm
b) Giả sử rằng a1 a2 a673 2 a674 a1346 2 a1347 a2019 ;
Gọi M a1, a2,..., a673 , N a 674 , a 675,..., a1346 , K a1347 , a1348 ,..., a2019 .
Chọn A(x ) (x m ), B(x ) (x n ),C (x ) (x k ) thì A(x ), B(x ),C (x ) có cùng
m M n N k K
bậc là 673 và A(x )B(x )C (x ) P (x ), x . 0,5 điểm
Xét số x 0 ( 1;1), vì tính đối xứng, ta có thể giả sử rằng x 0 [0;1) và khi đó với mọi
k K , ta có x 0 k 1 nên C (x 0 ) 1. 0,5 điểm
Mặt khác, với mọi m M,n N thì x 0 m x0 ( 2) x0 n nên A(x 0 ) B(x 0 ) .
Do đó A(x 0 ) C (x 0 ) B(x 0 ) , x 0 [0;1) . 1 điểm
Vì vậy B(x ) A(x )C (x ), x ( 1;1). Ta có đpcm.
--- HẾT ---
SỞ GIÁO DỤC VÀ ĐÀO TẠO KỲ THI CHỌN ĐỘI TUYỂN HỌC SINH GIỎI THPT
THÀNH PHỐ HỒ CHÍ MINH NĂM HỌC 2019 - 2020
MÔN THI: TOÁN
Ngày thi thứ hai: 19/9/2019
ĐỀ CHÍ NH THỨC
Thời gian làm bài: 180 phút (không kể thời gian phát đề)

Bài 5. (5 điểm)
a) Cho đa thức hai biế n P (x , y ) với hệ số thực. Chứng minh rằng tồn tại các đa thức
một biến S (x ),T (x ) với hệ số thực sao cho
P (x , y ) S (x ).y T (x ) (mod x 2 y 2 1).
b) Tồ n tạ i hay không một đa thức hai biế n P (x , y ) với hệ số thực sao cho
P 2 (x , y ) 1 chia hế t cho x 2 y 2 1?

Bài 6. (5 điểm)
Với n 2, hoán vị a1, a2, , an   của (1, 2, , n ) được gọi là “chuẩn” nếu ai 1
ai 1 với
i 1, 2, ,n 1. Tìm số các hoán vị “chuẩn” của (1, 2, , n ).

Bài 7. (5 điểm)
Chô hai đường tròn (O ),(O ) cố định, cắt nhau tại hai điểm B,C sao cho O,O nằm cùng
một phía đối với đường thẳng BC (điểm O gần BC hơn). Điểm A thay đổi trên (O ) sao
cho tam giác ABC nhọn không cân và các đôạn thẳng AB, AC cắt (O ) lần lượt tại F , E .
BE cắt CF tại I , AI cắt BC tại D, IB cắt DF tại M và IC cắt DE tại N .
a) Tia O I cắt đường tròn (O ) tại R . Chứng minh rằng AR, MN , BC đồng quy.
b) Chứng minh rằng khi A thay đổi trên (O ) thì đường phân giác trông và đường cao qua
đỉnh I của tam giác IMN lần lượt đi qua các điểm cố định.

Bài 8. (5 điểm)
Số nguyên dương n được gọi là số “đẹp” nếu nó thỏa mãn đồng thời các điều kiện sau:
i) n là số chính phương không chia hết cho 3.
ii) Với mỗi ước m 15 của n thì m 15 p k với k , p nguyên tố.
a) Chứng minh rằng nếu n là số “đẹp” và n có ước nguyên tố lẻ p thì p 7.
b) Tìm tất cả các số “đẹp” (Chú ý rằng thêô điều kiện trên thì n 1 là số “đẹp”).

--- HẾT ---


SỞ GIÁO DỤC VÀ ĐÀO TẠO KỲ THI CHỌN ĐỘI TUYỂN HỌC SINH GIỎI THPT
THÀNH PHỐ HỒ CHÍ MINH NĂM HỌC 2019 - 2020
MÔN THI: TOÁN
Ngày thi thứ hai: 19/9/2019
ĐÁP ÁN CHÍ NH THỨC
Thời gian làm bài: 180 phút (không kể thời gian phát đề)

Bài 5. (5 điểm)
c) Cho đa thức hai biế n P (x , y ) với hệ số thực. Chứng minh rằng tồn tại các đa thức
một biến S (x ),T (x ) với hệ số thực sao cho
P (x , y ) S (x ).y T (x ) (mod x 2 y 2 1).
d) Tồ n tạ i hay không một đa thức hai biế n P (x , y ) với hệ số thực sao cho
P 2 (x , y ) 1 chia hế t cho x 2 y 2 1?
Giải.
n
a) Ta có P(x , y ) Si (x )y i .
i 0

Sử dụ ng đồ ng dư thức y 2 (x 2 1) ( mod x 2 y2 1). 0,5 điểm


2 2
Từ đó xét thêô mod x y 1 thì tất cả các bậc từ 2 trở lên của y trong P (x , y ) đều
chuyển được về bậc 0 hoặc bậc 1 theo y . Do vậy tồn tại các đa thức S (x ),T (x ) có hệ số
thực sao cho
P (x , y ) S (x ).y T (x ) (mod x 2 y 2 1). 1 điểm
b) Ta chứng minh không tồ n tạ i đa thức P (x , y ) như vạ y. Giả sử ngược lạ i, tồ n tạ i đa
thức hai biế n P (x , y ) sao cho P 2 (x , y ) 1 chia hế t cho x 2 y 2 1 .
Theo a) ta có : P (x , y ) S (x ).y T (x ) (mod x 2 y2 1)

2
Từ đây suy ra S (x ).y T (x ) 1  0  mod  x 2 y2 1

Hay là S 2 (x )y 2 2yS (x )T (x ) T 2 (x ) 1  0  mod  x 2 y2 1

Suy ra S 2 (x )(x 2 1) 2yS (x )T (x ) T 2 (x ) 1  0  mod  x 2 y2 1 1 điểm


Suy ra
S (x )T (x ) S 2 (x )(x 2 1) T 2 (x ) 1 0.
Suy ra S (x ) 0 hoă ̣c T (x ) 0 . 1,5 điểm
Trong trường hơ ̣p thứ nhấ t ta suy ra T 2 (x ) 1 0 , mâu thuẫn. Trong trường hơ ̣p thứ hai ta có
1 – (x 2 1)S 2 (x ) 0 cũng mâu thuẫn. 1 điểm
Điề u mâu thuẫn trên chứng tỏ giả sử ban đầ u là sai. Tức là không tồ n ta ̣i đa thức hai biế n
P (x , y ) sao cho P 2 (x , y ) 1 chia hế t cho x 2 y 2 1 .
Bài 6. (5 điểm)
Với n 2, hoán vị a1, a2, , an   của (1, 2, , n ) được gọi là “chuẩn” nếu ai 1
ai 1 với
i 1, 2, , n 1. Tìm số các hoán vị “chuẩn” của (1, 2, , n ).
Giải. Gọi S (n ) là số các hoán vị “chuẩn” của (1, 2, , n ).
Ta định nghĩa hôán vị “chuẩn” của n số nguyên dương liên tiếp tương tự như trông đề.
Nhận xét: số các hoán vị “chuẩn” của n số nguyên liên tiếp luôn bằng S (n ) . 1 điểm
Cách 1.
Xét n 3 . Ta chia các hoán vị “chuẩn” của (1, 2, , n ) thành n nhóm, nhóm k là loại mà
ak 1.
Rõ ràng số các hoán vị “chuẩn” của nhóm 1 bằng số các hoán vị “chuẩn” của (2, 3 , n ) là
S (n 1).
Xét một hoán vị “chuẩn” của (1, 2, , n ) thuộc nhóm k , với k 2,..., n 2 .
Do 1 ak ak 1 1 nên ak 1 2 . Suy ra ak 1 2. Lý luận tiếp tục như vậy ta suy ra k số
hạng đầu của hoán vị phải là k, k 1, , 2,1 và n k số hạng cuối sẽ là một hoán vị
“chuẩn” của (k 1, , n ).
Theo nhận xét trên thì số các hoán vị “chuẩn” của nhóm k sẽ bằng S (n k ) .
Nhóm n 1 có đúng 1 hoán vị “chuẩn” là (n 1, ,1, n ).
Nhóm n có đúng 1 hoán vị “chuẩn” là (n, , 2,1).
Từ đó ta có công thức S (n ) S (n 1) S (n 2) ... S (2) 2, n 3. 3 điểm
Thay n n 1 ta có S (n 1) S (n ) S (n 1) ... S (2) 2 2S (n ), n 2.
Từ đó dô S (2) 2 nên S (n ) 2n 1, n 2. 1 điểm
Vậy số hoán vị “chuẩn” của (1, 2, , n ) bằng 2n 1.
Cách 2.
Xét n 3 . Ta chia các hoán vị “chuẩn” của (1, 2, , n ) thành 2 nhóm.
Nhóm 1 gồm các hoán vị “chuẩn” mà số 1 ở vị trí đầu tiên, rõ ràng số các hoán vị “chuẩn”
của nhóm 1 bằng số các hoán vị “chuẩn” của (2, 3 , n ) là S (n 1). 1 điểm
Nhóm 2 gồm các hoán vị “chuẩn” mà số 1 không ở vị trí đầu tiên. Khi đó số 1 phải đứng
liền sau số 2 .
Từ một hoán vị “chuẩn” thuộc nhóm 2, nếu bỏ số 1 thì ta thu được một hoán vị “chuẩn”
của (2, 3 , n ) và ngược lại từ một hoán vị “chuẩn” của (2, 3 , n ) , nếu bổ sung thêm số 1
liền sau số 2 thì ta thu được một hoán vị “chuẩn” thuộc nhóm 2.
Do đó số các hoán vị “chuẩn” của nhóm 2 cũng đúng bằng số các hoán vị “chuẩn” của
(2, 3 , n ) là S (n 1). 2 điểm
Từ đó suy ra S (n ) 2S (n 1) và do S (2) 2 nên S (n ) 2n 1. 1 điểm
Vậy số hoán vị “chuẩn” của (1, 2, , n ) bằng 2n 1 .
Lưu ý. Có thể tìm được song ánh từ tập các hoán vị “chuẩn” đến tập các tập con của tập
{1, 2,..., n 1}. Từ đó ta cũng suy ra số hoán vị “chuẩn” của (1, 2, , n ) bằng 2n 1.
Bài 7. (5 điểm)
Chô hai đường tròn (O ),(O ) cố định, cắt nhau tại hai điểm B,C sao cho O,O nằm cùng
một phía đối với đường thẳng BC (điểm O gần BC hơn). Điểm A thay đổi trên (O ) sao
cho tam giác ABC nhọn không cân và các đôạn thẳng AB, AC cắt (O ) lần lượt tại F , E .
BE cắt CF tại I , AI cắt BC tại D, IB cắt DF tại M và IC cắt DE tại N .
a) Tia O I cắt đường tròn (O ) tại R . Chứng minh rằng AR, MN , BC đồng quy.
b) Chứng minh rằng khi A thay đổi trên (O ) thì đường phân giác trông và đường cao qua
đỉnh I của tam giác IMN lần lượt đi qua các điểm cố định.
Giải. a) Gọi T là giaô điểm của EF và BC . Thêô định lý Brocard cho tứ giác toàn phần
BCEF .AT với đường tròn (O ) ngoại tiếp BCEF , ta có O I AT tại điểm Miquel R .
Mặt khác, R (ABC ) nên R R. Dô đó AR, EF , BC đồng quy. 1 điểm
Do AD, BE ,CF đồng quy tại I nên (BI , ME ) (CI , NF ) 1.
Suy ra MN , EF , BC đồng quy.
Từ đây suy ra AR, MN , BC đồng quy. 1 điểm

F O
I
N
M

O'

T B D C
J
x
L

b) Để ý rằng BAC , BEC , BFC cùng chắn cung BC của hai đường tròn (O ),(O ) nên số
đô của chúng không đổi. Suy ra BIC 360 (BAC AEI AFI ) cũng không đổi.
Dô đó, I di chuyển trên đường tròn cố định. Suy ra phân giác trong góc I của tam giác
IMN sẽ đi qua điểm cố định X là điểm chính giữa cung BC không chứa I của đường
tròn (BIC ), đpcm. 1 điểm
Giả sử đường thẳng qua I , vuông góc với IT cắt OO tại L, đường thẳng qua I , vuông
góc với EF cắt OO tại J và Ix là tia vuông góc với BC .
Xét chùm điều hòa T (AI , FD ) với IO TA (theo câu a), IJ TF , IL IT , Ix TB
nên theo tính chất chùm trực giaô thì ta cũng có (Ix , IJ , IL, IO ) 1.
Mặt khác, Ix OO nên theo tính chất chùm điều hòa thì J là trung điểm O L. 0,5 điểm
Giả sử đường caô đỉnh I trong tam giác IMN cắt OO tại K .
Từ chùm điều hòa T (EM , IB ) 1, trực giaô qua đỉnh I , ta có (IJ , IK , IL, Ix ) 1 nên
tương tự như trên, ta cũng có L là trung điểm JK . 1 điểm
Cách 1. Do BCEF nội tiếp nên EF đối song trong tam giác IBC suy ra J là tâm của
(BIC ). Suy ra J cố định. Do O , J cố định nên L, K cố định.
Vậy đường cao qua đỉnh I của tam giác IMN luôn đi qua điểm K cố định. 0,5 điểm
Cách 2. Gọi V là giaô điểm của AI và OO thì V là cực của BC đối với (O ), suy ra V cố
định. Lại có (O, J ,O ,V ) 1 nên J cố định. Do O , J cố định nên L, K cố định.
Vậy đường cao qua đỉnh I của tam giác IMN luôn đi qua điểm K cố định. 0,5 điểm
Bài 8. (5 điểm)
Số nguyên dương n được gọi là số “đẹp” nếu nó thỏa mãn đồng thời các điều kiện sau:
i) n là số chính phương không chia hết cho 3.
ii) Với mỗi ước m 15 của n thì m 15 p k với k , p nguyên tố.
a) Chứng minh rằng nếu n là số “đẹp” và n có ước nguyên tố lẻ p thì p 7.
b) Tìm tất cả các số “đẹp” (Chú ý rằng thêô điều kiện trên thì n 1 là số “đẹp”).
Giải. a) Ta thấy rằng nếu số nguyên tố lẻ p là ước của n thì p 3 và p 2 | n (do n là số
chính phương). Chú ý rằng p 3 nên p 2 15, thêô điều kiện ii) thì tồn tại m và số
nguyên tố q sao cho p 2 15 qm . 1 điểm
Vì p lẻ nên p 2 15 chẵn, kéo theo q m chẵn, suy ra q 2. Dô đó, ta có p 2 15 2m .
Xét theo modulo 3 thì p 2 15 1 (mod 3) nên 2m ( 1)m 1 (mod 3) và m chẵn. Đặt
m 2k với k nguyên dương thì 0 p 2 15 22k (2k p)(2k p) 15.
Để ý rằng (2k p) (2k p) 2p 6 nên 2k p 15, 2k p 1 .
Giải ra ta được p 7, k 3. 1 điểm
b) Để tìm tất cả các số “đẹp”, ta xét hai trường hợp:
- Nếu n là lũy thừa của 2 thì hiển nhiên n 1, 4 là các số “đẹp”. Kiểm tra trực tiếp, ta
thấy rằng
24 15 31, 25 15 47,
26 15 79, 27 15 143 11 13
2b
nên n 16, n 64 là các số “đẹp” và các số 2 , b 4 không phải số “đẹp” vì có ước là
7 7
2 15 nhưng 2 15 lại không là lũy thừa của số nguyên tố. 1 điểm
- Nếu n có ước nguyên tố lẻ p thì theo câu a, n sẽ có dạng 49a 4b với a và b .
Tương tự trên, kiểm tra trực tiếp, ta thấy rằng
72 15 64 26, 2 72 15 113,
22 7 15 43, 22 72 15 211
đều là các lũy thừa của số nguyên tố. Suy ra n 49,196 là các số “đẹp”. 1 điểm

Tuy nhiên 7 3 15 358 2.179, 8 72 15 407 11 37 không phải là lũy thừa của
số nguyên tố.
Điều này chứng tỏ a 2 hoặc b 2 đều dẫn đến n không phải là số “đẹp”. Vì thế nên
trông trường hợp này chỉ có 2 số “đẹp”.
Vậy tóm lại, có tất cả 6 số “đẹp” là n {1, 4,16, 64, 49,196}. 1 điểm

--- HẾT ---


SỞ GIÁO DỤC VÀ ĐÀO TẠO KỲ THI CHỌN ĐỘI TUYỂN HỌC SINH GIỎI
TỈNH ĐỒNG THÁP DỰ THI CẤP QUỐC GIA NĂM 2020
________________
Môn: TOÁN
ĐỀ CHÍNH THỨC Ngày thi: 24/7/2019
(Đề gồm có 01 trang) Thời gian làm bài: 180 phút, không kể thời gian phát đề

Câu 1. (3,0 điểm)


Cho dãy số thực dương ( xn ) xác định bởi công thức:
1
xn 21 , n 1.
2n 2 4n 4 1
Đặt Sn x1 x2 ... xn n 1.
a) Tính S 20 .
b) Chứng minh rằng tồn tại vô số số nguyên dương n sao cho Sn nhận giá trị nguyên.
Câu 2. (6,0 điểm)
a) Giải hệ phương trình sau:
 2  2 y2  4 y  4
x 
 y
 .
 2  2x  4x  4
2

y 
 x
b) Tìm tất cả các cặp  a ; b  với a, b là các số nguyên dương, sao cho
M  a 4  10a 2  2b
là số chính phương.
Câu 3. (2,0 điểm)
Cho a, b, c là các số thực thỏa mãn a  b  c  3. Chứng minh rằng:
(ab  bc  ca ) 2  9  18abc.
Câu 4. (5,0 điểm)
Cho đường tròn  O  và dây cung BC cố định. Điểm A (khác với B, C ) di động trên
đường tròn  O  sao cho AB  AC . Gọi D, E , F lần lượt là chân đường cao hạ từ A, B, C của
tam giác ABC. Đường thẳng EF cắt đường tròn  O  tại P và Q . Chứng minh rằng:
a) Đường tròn ngoại tiếp tam giác PQD đi qua một điểm cố định.
b) Tâm đường tròn ngoại tiếp tam giác PQD chạy trên một đường tròn cố định.
Câu 5. (4,0 điểm)
Xét bảng ô vuông kích thước 5  n  n    (bảng gồm 5 hàng và n cột). Hỏi với n nào
thì có thể lát được bảng bởi các viên gạch có dạng như hình bên dưới (hình có thể xoay theo
mọi hướng)?

--- HẾT---
Trang 1/5
SỞ GIÁO DỤC VÀ ĐÀO TẠO KỲ THI CHỌN ĐỘI TUYỂN HỌC SINH GIỎI
TỈNH ĐỒNG THÁP DỰ THI CẤP QUỐC GIA NĂM 2020
________________
Môn: TOÁN
HƯỚNG DẪN CHẤM Ngày thi: 24/7/2019
______________________________________
(HDC gồm có 05 trang)

I. Hướng dẫn chung


1) Nếu học sinh làm bài không theo cách nêu trong hướng dẫn chấm nhưng đúng, chính
xác, chặt chẽ thì cho đủ số điểm của câu đó.
2) Việc chi tiết hóa (nếu có) thang điểm trong hướng dẫn chấm phải bảo đảm không làm
sai lệch hướng dẫn chấm và phải được thống nhất thực hiện trong tổ chấm.

II. Đáp án và thang điểm


Câu Ý Nội dung Điểm
1. Cho dãy số thực dương ( xn ) xác định bởi công thức:
1
xn 21 , n 1.
2
2n 4n 4 1
Đặt Sn x1 x2 ... xn n 1. 3,0
a) Tính S 20 .
b) Chứng minh rằng tồn tại vô số số nguyên dương n sao cho Sn nhận
giá trị nguyên.
a) Tính S 20 2,0
Ta có xn 21 4n 4 1 2n 2
0,25
Do đó xn 4n 2 2 2 4n 4 1; n 1. 0,25
2
Ta thấy rằng 4n 2 2 2 4n 4 1 2n 2 2n 1 2n 2 2n 1 0,25

Nên xn 2n 2 2n 1 2n 2 2n 1. 0,25
2
Hay xn 2n 2 2n 1 2 n 1 2 n 1 1. 0,25
Suy ra:
Sn x1 x2 ... xn
5 1 13 5 0,25
2
... 2n 2 2n 1 2 n 1 2 n 1 1

S n  2n 2  2n  1  1. 0,25
Vậy S20  2.202  2.20  1  1  28. 0,25

b) Chứng minh tồn tại vô số n  sao cho Sn nhận giá trị nguyên 1,0
Ta có: 0,25
Trang 2/5
Sn   2n 2  2 n  1  a 2  a  

 2a 2   2n  1  1.
2
0,25
Ta chứng minh phương trình 2a 2  b 2  1 có vô số nghiệm nguyên
dương.
Xét hai dãy  an  ;  bn  được xác định bởi công thức: 0.25
an 1  3an  2bn , bn 1  4an  3bn
với a0  b0  1.
Kiểm tra bằng quy nạp ta được:
2an2  bn 2  1, n . 0.25
Suy ra điều phải chứng minh.

2. a) Giải hệ phương trình sau:


 2  2 y2  4 y  4
x 
 y 3,0
 .
 2  2x  4x  4
2

y 
 x
Điều kiện: x, y  0 . 0,25
2  2 y 2  4 y  4  xy 1

Hệ tương đương với  0,25
2  2 x 2  4 x  4  xy  2 
Trừ theo vế ta được:
 x  y  x  y  2   0 0,5
Trường hợp 1. Với x  y .
Thay vào hệ ta được:
 0,5
x  2
2

x  2  2x  4x  4   4
2 2

x  6x  4x  0

2

Giải hệ và so điều kiện, ta được x  1  3, 2 Khi đó y  1  3, 2 0,5


Trường hợp 2. Với x  y  2  0.
Thay y   x  2 vào hai vế của (1) ta có: 0,5
2x  4x  4    x  2x  2
2 2

Giải phương trình, ta được phương trình vô nghiệm. 0,25



Vậy nghiệm của hệ là  x ; y   1  3 ;1  3 ,  2; 2  .  0,25
b) Tìm tất cả các cặp  a ; b  với a, b là các số nguyên dương, sao cho
M  a 4  10a 2  2b 3,0
là số chính phương.
Giả sử a 4  10a 2  2b  n 2  n  
. 0,25
Trường hợp 1: Xét b  1: Khi đó n 2  a 4  10a 2  2.
Trang 3/5
+ Nếu a chẵn thì n 2  2  mod 4  , loại. 0,25
+ Nếu a lẻ thì n 2  5  mod8  , loại. 0,25
Trường hợp 2: Xét b  2 : Khi đó n 2  a 4  10a 2  4. Ta có:
0,25
(a 2  2) 2  n 2  (a 2  5) 2
Suy ra: n  a 2  3 hoặc n  a 2  4 . 0,25
Cả hai trường hợp này đều không có nghiệm.
Trường hợp 3: Xét b  3 : Tương tự như trường hợp b  2 ta tìm được
0,25
a  2.
Trường hợp 4: Xét b  4 : Khi đó (a 2  4) 2  M  (a 2  5) 2 nên M
0,25
không là số chính phương.
Trường hợp 5: Xét b  5 : Nếu a lẻ thì n 2  3  mod 4  , loại. 0,25
Suy ra a chẵn. Đặt a  2k .c , trong đó c lẻ. 0,25
Khi đó n 2  24 k .c 4  5.22 k 1.c 2  2b. 0,25
Vì số mũ của 2 trong n 2 chẵn nên b  2k  1. 0,25
Suy ra:
n 2  a 4  10a 2  2a 2   a 2  6   n  a 2  5.
2
0,25
Mà b  5 nên n  a  5 , loại. Vậy  a ; b    2;3 là cặp cần tìm.
2

3. Cho a, b, c là các số thực thỏa mãn a  b  c  3. Chứng minh rằng:


2,0
(ab  bc  ca ) 2  9  18abc.
Đặt a  x  1; b  y  1; c  z  1. 0,25
Khi đó x  y  z  0 và điều phải chứng minh tương đương với
0,25
P   xy  yz  zx   12( xy  yz  zx)  18 xyz  0
2

Vì ba số x, y , z luôn có hai số có tích không âm. Không mất tính tổng


0,25
quát, ta giả sử hai số đó là x, y .
Thay z    x  y  ta được
0,25
P  ( x 2  xy  y 2 ) 2  12( x 2  xy  y 2 )  18 xy ( x  y )
Vì x 2  xy  y 2  3xy  0 nên:
0,25
P  (3xy ) 2  12( x 2  xy  y 2 )  18 xy ( x  y )
Ta lại có:
3 0,25
x 2  xy  y 2  ( x  y )2
4
Do đó:
3 0,25
P  (3xy )2  12  ( x  y )2  18 xy ( x  y )
4
3
Mà (3xy ) 2  12  ( x  y ) 2  18 xy ( x  y )  9( xy  x  y ) 2 .
4 0,25
Nên P  0 (đpcm). Đẳng thức xảy ra khi và chỉ khi a  b  c  1 hoặc

Trang 4/5
a  b  1, c  5 và các hoán vị của nó.

4. Cho đường tròn  O  và dây cung BC cố định. Điểm A khác với B, C di


động trên đường tròn sao cho AB  AC. Gọi D, E , F lần lượt là chân
đường cao hạ từ A, B, C của tam giác ABC. Đường thẳng EF cắt  O  tại
P và Q . Chứng minh rằng: 5,0
a) Đường tròn ngoại tiếp tam giác PQD đi qua một điểm cố định.
b) Tâm đường tròn ngoại tiếp tam giác PQD chạy trên một đường
tròn cố định.

a) Chứng minh đường tròn ngoại tiếp tam giác PQD đi qua một điểm cố
2,0
định.

O
F

S C
B D M

Gọi M là trung điểm của BC . Ta thấy bốn điểm D, M , E , F cùng


0,25
thuộc một đường tròn (đường tròn Euler của tam giác ABC ) .
Dễ thấy bốn điểm B, C , E , F cùng thuộc một đường tròn. 0,25
Gọi S  PQ  BC. Ta có: SD.SM  SE.SF 0,25
mà SE.SF  SB.SC 0,25
và SB.SC  SP.SQ 0,25
Do đó SD.SM  SP.SQ 0,25
Suy ra M đường tròn ngoại tiếp tam giác PQD . 0,25
Mà M là trung điểm BC nên M cố định (đpcm) 0,25
b) Chứng minh tâm I của đường tròn ngoại tiếp tam giác PQD chạy trên
3,0
một đường tròn cố định.
Vì AEP  ABC  APC. 0,5
Suy ra APE đồng dạng ACP. 0,5
Trang 5/5
Do đó APE  ACP  AQP. 0,5
Vậy OA  EF . Suy ra I thuộc OA. 0,5
Hơn nữa, ADMO là hình thang và I thuộc trung trực của DM nên I là
0,5
trung điểm của OA.
1  1 
Ta có: OI  R nên I thuộc đường tròn  O; R  . 0,5
2 2  

5. Xét bảng ô vuông kích thước 5  n  n  


 (bảng gồm 5 hàng và n cột).
Hỏi với n nào thì có thể lát được bảng bởi các viên gạch có dạng như
hình bên dưới (hình có thể xoay theo mọi hướng)? 4,0

Trước tiên ta chứng minh n chẵn thì bảng được lát.


+ Ta ghép hai viên gạch trên (một viên theo chiều ngược lại) được một 0,75
khối 2  5 .
+ Vì n chẵn nên bảng có thể chia thành các hình chữ nhật 5  2 . Sử dụng
khối gạch đã ghép bên trên ta lát cho các hình chữ nhật. Như vậy với n 0,75
chẵn ta có thể lát bảng.
Ta sẽ chứng minh với n lẻ thì không thể lát bảng.
Giả sử lát được. Ta tô màu cả hàng, xen kẽ với hai màu đen và trắng
(như hình vẽ).

0,5

Khi đó mỗi viên gạch sẽ chiếm: 3 ô đen và hai ô trắng (loại 1) hoặc 3 ô
0,75
trắng và hai ô đen (loại 2).
Giả sử lát được bởi x viên loại 1 và y viên loại 2. Ta có hệ:
3x  2 y  3n x  n 0,5
 
3 y  2 x  2n y  0
Như vậy, nếu lát được thì chỉ có thể lát bằng n viên loại 1. Tuy nhiên, chỉ
0,5
dùng các viên loại 1 thì không thể lát được hết các ô hàng 2 (vì số ô lẻ).
Vậy bảng được lát bởi các viên gạch có dạng (có thể xoay theo mọi
hướng):

0,25

khi và chỉ khi n chẵn.

Trang 6/5
-HẾT-

Trang 7/5
SỞ GD&ĐT VĨNH PHÚC KỲ THI CHỌN HSG LỚP 12 THPT NĂM HỌC 2019-2020
ĐỀ THI MÔN: TOÁN – THPT CHUYÊN
ĐỀ CHÍNH THỨC Thời gian: 180 phút, không kể thời gian giao đề

Câu 1.
xn  1
a) Cho dãy số  xn  được xác định bởi x1  1 và xn 1  với mọi n  *
. Chứng minh rằng dãy
xn  2
số  xn  có giới hạn hữu hạn và tìm giới hạn đó.
b) Tìm tất cả các hàm số xác định và liên tục trong khoảng  0;   sao cho
 x 1  x  x 1
2

f  x  f   với mọi x  0.
 x2 x2
Câu 2.
a) Cho số tự nhiên a  2 sao cho a  1 có ước nguyên tố lẻ là p . Chứng minh rằng a p  1 p 2 .
2

b) Chứng minh rằng tồn tại vô số số tự nhiên n sao cho 2019n  1 n.

Câu 3. Cho tam giác ABC nhọn có chiều cao AH. Đường tròn nội tiếp I  tiếp xúc với
BC , CA, AB lần lượt tại D, E , F . Đường tròn  A  có tâm A bán kính AE cắt đoạn thẳng AH tại
điểm K. Đường thẳng IK cắt đường thẳng BC tại P. Các đường thẳng DK và PK cắt đường tròn  A 
lần lượt tại Q và T khác K .
a) Chứng minh rằng tứ giác TDPQ nội tiếp và ba điểm Q, A, P thẳng hàng.
b) Đường thẳng DK cắt  I  tại điểm thứ hai là X. Chứng minh rằng ba đường thẳng AX , EF , TI
đồng quy.
c) Chứng minh rằng đường tròn đường kính AP tiếp xúc với đường tròn  I  .

Câu 4. Cho P  x  là một đa thức khác hằng số với hệ số thực sao cho tất cả các nghiệm của nó đều
là số thực. Giả sử tồn tại một đa thức Q  x  hệ số thực sao cho  P( x)   P  Q  x   với mọi x  .
2

Chứng minh rằng tất cả các nghiệm của P  x  đều bằng nhau.
Câu 5. Một tập hợp gồm 3 số nguyên dương được gọi là tập Pytago nếu 3 số này là độ dài ba cạnh
của một tam giác vuông. Chứng minh rằng với hai tập Pytago P, Q bất kỳ, ta luôn tìm được m tập
Pytago P1 , P2 ,..., Pm (m  2) sao cho P1  P, Pm  Q và Pi Pi 1   với mọi 1  i  m  1.

------------Hết------------
Thí sinh không được sử dụng tài liệu. Cán bộ coi thi không giải thích gì thêm.

Họ và tên thí sinh:……….………..…….................……; Số báo danh:……….....……….................


SỞ GD&ĐT VĨNH PHÚC KỲ THI CHỌN HSG LỚP 12 THPT NĂM HỌC 2019-2020
(Đáp án có 05 trang) HƯỚNG DẪN MÔN: TOÁN – THPT CHUYÊN

I. LƯU Ý CHUNG:
- Hướng dẫn chấm chỉ trình bày một cách giải với những ý cơ bản phải có. Khi chấm bài học sinh
làm theo cách khác nếu đúng và đủ ý thì vẫn cho điểm tối đa.
- Điểm toàn bài tính đến 0,25 và không làm tròn.
- Với bài hình học nếu thí sinh không vẽ hình phần nào thì không cho điểm tương ứng với phần đó.
II. ĐÁP ÁN:
Câu Ý Nội dung trình bày Điểm
1 x 1
a) Cho dãy số  xn  được xác định bởi x1  1 và xn 1  n với mọi n  * .
xn  2
Chứng minh rằng dãy số  xn  có giới hạn hữu hạn và tìm giới hạn đó.
3,0
b) Tìm tất cả các hàm số xác định và liên tục trong khoảng  0;   sao cho
 x 1  x  x 1
2

f  x  f    , x  0
 x2 x2
a b 1 5 1
Xét số b>0 là nghiệm của phương trình b  b . Dễ thấy
b2 2
xn  0, n  1 nên ta có: 1,0
xn  1 b  1 xn  b 1
0  xn 1  b     xn  b
xn  2 b  2  xn  2  b  2  4
2 n
1 1 1
Suy ra 0  xn 1  b  xn  b    xn 1  b    x1  b
4 4 4
n 1,0
1 5 1
Do lim   x1  b  0 nên theo nguyên lý kẹp suy ra lim xn  b 
4 2
b  x 1  x  x 1  x 1  x 1
2

Ta có f  x   f    f  x  f   x
 x2 x2  x2 x2
 x 1  x 1
Suy ra f  x   x  f   , x  0 0,5
 x2 x2
 x 1 
Đặt g  x   f  x   x  g  x   g   , x  0 (1)
 x2
x 1
Chọn a  0 tùy ý, xét dãy  xn  xác định bởi x1  a; xn 1  n , n  * .
xn  2
5 1 0,25
Hoàn toàn tương tự phần (a) thì lim xn  b 
2
Từ (1) suy ra g  a   g  x1   g  x2   ...  g  xn  , n  *

Do hàm g  x  liên tục trên  0;   nên


 5 1 
g  a   lim g  xn   g  lim xn   g    c
 2  0,25
Suy ra g  x   c với mọi x  0. Thử lại ta thấy hàm số cần tìm là f  x   x  c với
mọi x  0, c là hằng số tùy ý.
2 a) Cho số tự nhiên a  2 sao cho a  1 có ước nguyên tố lẻ là p . Chứng 2,0
minh rằng a p  1 p 2 .
2

b) Chứng minh rằng tồn tại vô số số tự nhiên n sao cho 2019n  1 n.


Ta có a p  1   a p   1   m  1  m p 1  m p  2  ...  m  1   m  1 A , với m  a p .
p
a 2
0,5
Do p lẻ nên a  1 a  1 p  m  1 p  m  1 mod p  . Do đó
p

0,5
A  m p 1  m p  2  ...  m  1  p  0  mod p 
Suy ra  m  1 A p 2 , tức là a p  1 p 2 .
2
0,5
b Trước hết ta chứng minh mệnh đề sau bằng quy nạp theo k: Cho số tự nhiên a  2
sao cho a  1 có ước nguyên tố lẻ là p . Khi đó a p  1 p k , k  * (1)
k

Theo giả thiết thì ta thấy ngay (1) đúng với k  1.


Giả sử (1) đúng với k, ta chứng minh (1) đúng với k  1 .
   1   m  1  m p 1  m p  2  ...  m  1   m  1 A , trong đó
k 1 p
Ta có a p  1  a p
k
0,25
m  a p . Theo giả thiết quy nạp m  1 p k . Lại có m  1 p  m  1 mod p  .
k

Do đó A  m p 1  m p  2  ...  m  1  p  0  mod p 
Suy ra  m  1 A p k 1 , tức là a p  1 p k 1 . Vậy (1) đúng với k  1.
k 1

Trở lại bài toán: Với a  2019 thì a  1  2020 có ước nguyên tố lẻ là 5 nên theo
(1) các số n  5k sẽ thỏa mãn 2019n  1 n.
0,25
Chú ý: Nếu học sinh chứng minh trực tiếp 20195  1 5k , k  * thì vẫn cho tối
k

đã điểm.
3 Cho tam giác ABC nhọn có chiều cao AH. Đường tròn nội tiếp  I  tiếp xúc với
BC , CA, AB lần lượt tại D, E , F . Đường tròn  A  có tâm A bán kính AE cắt đoạn
thẳng AH tại điểm K. Đường thẳng IK cắt đường thẳng BC tại P. Các đường
thẳng DK và PK cắt đường tròn  A  lần lượt tại Q và T khác K .
3,0
a) Chứng minh rằng tứ giác TDPQ nội tiếp và ba điểm Q, A, P thẳng hàng.
b) Đường thẳng DK cắt  I  tại điểm thứ hai là X. Chứng minh rằng ba đường
thẳng AX , EF , TI đồng quy.
c) Chứng minh rằng đường tròn đường kính AP tiếp xúc với đường tròn  I  .
Q

E
F
K
I

B H D C
P

a) 1
Ta có TQD  TQK  TAK  900  AKT  HPK  TPD . Suy ra tứ giác TDPQ nội
2 1,0
tiếp.
Ta có KQA  AKQ  DKH  KDI (1)
Dễ thấy IF là tiếp tuyến của  A  nên ID 2  IF 2  IK .IT  IDK ITD
0,5
Suy ra KDI  ITD  KQP (2). Từ (1) và (2) suy ra KQA  KQP .
Do đó ba điểm Q, A, P thẳng hàng.
Q

T
X
E
Z
S
t F
K I

B H D P C

b Gọi Y là giao điểm thứ hai của AX với  I  . Ta có


IX 2  IF 2  IK .IT  ITX  IXK  IDX  AKX ( vì AK || ID ) 0,25
Lại có AK  AF  AX . AY  AKX  AYK . Suy ra ITX  AYK . Do đó tứ giác
2 2

XKYT nội tiếp.


Xét ba đường tròn:  XKYT  ;  I  ;  A  , lần lượt có trục đẳng phương là KT, XY, EF.
Do đó ba đường thẳng KT, XY, EF đồng quy tại tâm đẳng phương của ba đường 0,25
tròn trên. Vậy ba đường thẳng AX, EF, TI đồng quy.
c Gọi Z là giao điểm thứ hai của đường thẳng PT với đường tròn đường kính AP.
Khi đó AZ  KT và Z là trung điểm KT. Do IE và IF là tiếp tuyến của  A  nên 0,25
TKSI   1 , theo hệ thức Macloranh ta được SZ .SI  SK .ST  SX .SY .
Suy ra tứ giác XZYI nội tiếp, suy ra ZYX  ZIX 0,25
Mặt khác IXD  ITD  DQP  IX || PQ  ZIX  ZPA . Vậy ZYX  ZPA
0,25
Suy ra tứ giác AZYP nội tiếp, suy ra Y thuộc đường tròn đường kính AP.
Vẽ tiếp tuyến Yt của (I), ta có
1
tYX  XIY  900  IXY  900  IZY  900  YAP  YPA .
2 0,25
Do đó Yt là tiếp tuyến của đường tròn đường kính AP. Vậy đường tròn đường kính
AP tiếp xúc với đường tròn  I  tại điểm Y (đpcm).
4
Cho P  x  là một đa thức khác hằng số với hệ số thực sao cho tất cả các nghiệm
của nó đều là số thực. Giả sử tồn tại một đa thức Q  x  hệ số thực sao cho
1,0
 P( x)   P  Q  x   với mọi x  . Chứng minh rằng tất cả các nghiệm của
2

P  x  đều bằng nhau.


Giả sử P  x   A  x  x1  .  x  x2  ...  x  xk  , trong đó x1  x2  ...  xk là tất cả
1 d 2 d k d

0,25
các nghiệm thực của P  x  . Dễ thấy degQ  x   2  Q  x   ax 2  bx  c .
Khi đó ta được
k

A2  x  x1  1 .  x  x2  ...  x  xk   A  ax 2  bx  c  xi 
2d 2 d2 2 dk di

i 1

Do đó với mỗi chỉ số i thì nghiệm của đa thức ax 2  bx  c  xi là xs , xt , với s, t nào


b 0,25
đó. Theo định lý Viet ta được xs  xt   .
a
Như vậy tất cả các nghiệm của P  x  được chia thành các cặp  xs , xt  mà tổng của
2

b
hai số trong mỗi cặp bằng nhau và bằng  .
a
Giả sử 1 ghép cặp với s và k ghép cặp với xt . Từ x1  xt ; xs  xk và
x x x
x1  xs  xk  xt ta suy ra x1  xt ; xs  xk . Vậy x1 chỉ có thể ghép cặp với xk . Lập
luận hoàn toàn tương tự suy ra mỗi cặp chỉ có dạng  x j , xk 1 j  . Áp dụng định lý 0,25
c  xm
Viet ta có x j .xk 1 j  , với m nào đó.
a
c  xm  k  1
Do có đúng k giá trị và các số dạng x j .xk 1 j chỉ chứa nhiều nhất 
a  2 
 k  1 0,25
giá trị phân biệt nên k   . Từ bất đẳng thức này ta suy ra ngay k=1. Khi đó
 2 
P  x   A  x  x1  1 , và suy ra tất cả các nghiệm của P  x  đều bằng nhau (đpcm).
d

5 Một tập hợp gồm 3 số nguyên dương được gọi là tập Pytago nếu 3 số này là độ
dài ba cạnh của một tam giác vuông. Chứng minh rằng với hai tập Pytago P, Q
bất kỳ, ta luôn tìm được m tập Pytago P1 , P2 ,..., Pm (m  2) sao cho P1  P, Pm  Q 1,0
và Pi Pi 1   với mọi 1  i  m  1.
Bổ đề: Với mỗi số nguyên dương n  3 , luôn tồn tại một tập Pytago chứa số n.
Ta chứng minh mệnh đề trên bằng quy nạp theo n.
Dễ thấy mệnh đề đúng với n  3, 4,5 vì 3, 4,5 là một tập Pytago.
Xét n  6 , giả sử mệnh đề đúng với mọi số nhỏ hơn n, ta cần chứng minh mệnh đề
đúng với n.
+ Nếu n chẵn, n  2k thì 3  k  n. Theo giả thiết quy nạp, tồn tại 1 tập Pytago A 0,25
chứa số k . Giả sử A  k , a, b . Khi đó tập B  n, 2a, 2b là tập Pytago chứa số n.
 1 
+ Nếu n lẻ, ta thấy tập A  n;  n2  1 ;  n2  1  là tập Pytago chứa số n.
1
 2 2 
Vậy luôn tồn tại một tập Pytago chứa số n.
Nếu hai tập Pytago P, Q thỏa mãn yêu cầu của bài toán thì ta nói cặp  P, Q  là một
cặp “đẹp” và kí hiệu là P Q.
Như vậy ta cần chứng minh mọi cặp Pytago  P, Q  đều là cặp đẹp (1)
Nhận xét: Ta chỉ cần chứng minh mệnh đề (1) đúng trong trường hợp P  3, 4,5 .
0,25
Chứng minh: Xét P  3, 4,5 và giả sử cứ với tập Q là tập Pytago bất kì thì  P, Q 
là cặp đẹp. Xét hai tập Pytago bất kì là Q, R , khi đó  P, Q  và  P, R  là cặp đẹp
nên tồn tại dãy Q1 , Q2 ,..., Qm và R1 , R2 ,..., Rt sao cho
Q1  P1  3; 4;5 ; Qm  Q; Rt  R và Qi Qi 1  ; Ri Ri 1  
Khi đó dãy Qm , Qm 1 ,..., Q1 , R2 , R3 ,..., Rt thỏa mãn yêu cầu bài toán. Suy ra  Q, R  là
cặp đẹp.
Qua phép chứng minh trên ta cũng suy ra rằng nếu  P, Q  và  P, R  là hai cặp đẹp
thì  Q, R  cũng là cặp đẹp.
Trở lại bài toán, xét P  3, 4,5 , ta tiếp tục chứng minh bài toán bằng quy nạp theo
phần tử nhỏ nhất của Q. Giả sử min Q  n
+ Nếu 3  n  5 thì hiển nhiên  P, Q  là cặp đẹp.
+ Xét n  6 , giả sử mệnh đề đúng với mọi số 3  min Q  n .
* Nếu n chẵn, n  2k thì 3  k  n . Theo bổ đề và giả thiết quy nạp thì tồn tại một
tập Pytago Q’ chứa k và  P, Q ' là cặp đẹp.
Dễ thấy rằng khi nhân tất cả các phần tử của một cặp đẹp với số 2 thì lại cho ta một 0,25
cặp đẹp mới. Do đó nếu gọi Q '  k ; x; y thì các cặp sau là đẹp:
n; 2 x; 2 y ;6;8;10 ; n; 2 x; 2 y ; Q  (vì có giao khác rỗng)
Mặt khác 6;8;10 ; 3; 4;5  cũng là cặp đẹp do chuỗi xây dựng các tập đẹp Pytago
sau: 6;8;10 8;15;17 9;12;15 5;12;13 3; 4;5
Vậy Q và 3; 4;5 tạo thành cặp đẹp.
 1 
n;  n  1 n  1 ;  n  1  . Theo bổ đề thì tồn tại tập Pytago
1 2
* Nếu n lẻ thì Q
 2 2 
1
R chứa  n  1 và tập Pytago H chứa n  1 .
2
1
Từ 3   n  1  n và 3  n  1  n nên theo giả thiết quy nạp ta có 0,25
2
R  3; 4;5 ; H  3; 4;5 . Do đó
 1 
n;  n  1 n  1 ;  n  1
1 2
 n  1 R  n  1 P 3H 33;4;5  9;12;15 3;4;5
 2 2 
Vậy Q và 3; 4;5 tạo thành cặp đẹp và bài toán được chứng minh hoàn toàn.

-----------Hết----------
Hướng tới kỳ thi chọn HSG quốc gia 2019-2020

ĐỀ MOCK TEST VMO 2020


Thời gian làm bài: 180 phút

Bài 1. (3 điểm) Cho số thực c 1 và hàm số f : (0; ) liên tục, thỏa mãn

f ( x) f ( x c ) với mọi x 0.

a) Chứng minh rằng f là hàm hằng.


b) Khẳng định trên còn đúng không nếu f không phải là hàm số liên tục?

Bài 2. (3 điểm) Cho hai đa thức hệ số nguyên, monic là P( x ), Q ( x ) , trong đó deg P 3,deg Q 2.
Giả sử rằng P ( x ) có ba nghiệm vô tỷ phân biệt có tổng bằng 0 là a , b, c đồng thời Q ( a ) b.

a) Chứng minh rằng P (Q ( x )) chia hết cho P ( x ).


b) Chứng minh rằng Q (a ) Q (b) Q (c ) 0.

Bài 3. (5 điểm) Cho tam giác ABC nhọn, không cân nội tiếp trong đường tròn (O ) với trực tâm
H . Điểm R thay đổi trên cung lớn BC của (O ) sao cho AR không song song với BC . Lấy các
điểm S , T trên đường thẳng BC sao cho ( ARS ),( ART ) cùng tiếp xúc với BC. Đường thẳng qua
H , vuông góc với AS , AT lần lượt cắt ( HBC ) ở X , Y .

a) Chứng minh rằng đường thẳng XY luôn đi qua điểm cố định.


b) Chứng minh rằng tâm của đường tròn ( RST ) di chuyển trên đường thẳng cố định.

Bài 4. (5 điểm) Tìm tất cả các hàm số f : thỏa mãn điều kiện

f (x 3 f ( y )) f (x f ( y) y3) f (4 f ( y ) y 3 ) 1 với mọi x, y .

Bài 5. (4 điểm) Cho tập hợp S gồm n số square-free lớn hơn 1 có tích bằng m là một số nguyên
dương có đúng 13 ước nguyên tố phân biệt. Biết rằng bất kỳ 5 số nào trong S cũng không có ước
nguyên tố chung và tích 2 số bất kỳ trong S thì không là số square-free.
(Số square-free là số nguyên dương không có ước chính phương nào khác 1 . Chẳng hạn 30 là số
square-free, còn 24 thì không vì 24 chia hết cho 4 ).
a) Chứng minh rằng n 13.
b) Chứng minh rằng khi n 13 thì m là số chính phương và mỗi số trong S có đúng 16
ước nguyên dương.

1
Hướng tới kỳ thi chọn HSG quốc gia 2019-2020

ĐÁP ÁN CHI TIẾT


Bài 1. Cho số thực c 1 và hàm số f : (0; ) liên tục, thỏa mãn

f ( x) f ( x c ) với mọi x 0.

a) Chứng minh rằng f là hàm hằng.

b) Khẳng định trên còn đúng không nếu f không phải là hàm số liên tục?

Lời giải.

a) Theo giả thiết thì với mọi x  0, ta có f ( x )  f ( x1/ c ) nên thực hiện thế liên tiếp thì

f ( x )  f ( x1/ c ) .
n

1
Chú ý rằng lim  0 nên theo tính chất hàm liên tục, ta có
x  cn

x 
n

 x 
n


f ( x )  lim f ( x1/ c )  f lim x1/ c  f ( x 0 )  f (1).

Vì thế nên f là hàm hằng.

b) Khẳng định sẽ không còn đúng nữa. Chẳng hạn ta chọn hàm số
f (1)  1, f ( x )  2, x  1.

Rõ ràng x  1 thì ta đều có x c  1 nên ta luôn có f ( x )  f ( x c ), x  0. Ngoài ra, vì không liên


tục nên không thể thực hiện phép thế liên tiếp và lấy giới hạn như trên, giá trị của f (1) độc lập
với các giá trị còn lại, thế nên hàm số trên thỏa mãn đề bài và rõ ràng nó không phải là hàm hằng.

Nhận xét. Câu hỏi tương tự là: Chứng minh rằng hàm số liên tục f :  thỏa mãn điều kiện
 1 
sau là hàm hằng f ( x )  f  x 2   , x  .
 4

Đặc điểm chung của các bài này là có thể thực hiện phép thế liên tiếp để từ một giá trị x bất kỳ,
ta tạo được dãy hội tụ về giá trị cụ thể.

Bài 2. Cho hai đa thức hệ số nguyên, monic là P( x ), Q ( x ) , trong đó deg P 3,deg Q 2. Giả sử
rằng P ( x ) có ba nghiệm vô tỷ phân biệt có tổng bằng 0 là a , b, c đồng thời Q ( a ) b.

a) Chứng minh rằng P (Q ( x )) chia hết cho P ( x ).

b) Chứng minh rằng Q (a ) Q (b) Q (c) 0.

Lời giải.

2
Hướng tới kỳ thi chọn HSG quốc gia 2019-2020

a) Ta sẽ chứng minh rằng P ( x ) là đa thức nguyên bậc nhỏ nhất nhận x  a là nghiệm. Giả sử
ngược lại có đa thức nguyên khác hằng f ( x ) bậc nhỏ hơn 3 mà f ( a )  0 , rõ ràng f ( x ) không
thể là bậc nhất vì a là số vô tỷ. Suy ra f bậc hai. Xét phép chia đa thức

P( x )  f ( x )  g ( x )  r ( x )

thì dễ thấy r ( a )  0, và deg r  deg f  2 nên deg r  0 . Suy ra r ( x )  0, tức là P ( x ) chia hết
cho f ( x ), nên P( x )  f ( x ) g ( x ), với g ( x )  [ x ] , chứng tỏ g có nghiệm hữu tỷ, và nghiệm đó
lại là của P ( x ), vô lý.

Tiếp theo, nếu có đa thức nguyên f1 ( x ) mà f1 ( a )  0 thì xét phép chia f1 ( x )  P ( x )  g1 ( x )  r1 ( x ),


ta có r1 ( a )  0 , mà deg r1  deg P nên phải có r1 ( x )  0, tức là f1 ( x ) chia hết cho P ( x ).

Trở lại bài toán, rõ ràng P(Q ( a ))  P(b)  0 nên x  a là nghiệm của P(Q ( x )), từ các nhận xét
trên, ta phải có P (Q ( x )) chia hết cho P ( x ).

b) Theo trên, ta có P(Q (b)) P(Q ( c)) 0. Suy ra Q (b), Q ( c ) a, b, c . Ta xét các trường hợp:

Nếu Q (b) b thì đa thức Q ( x )  x có bậc hai nhận nghiệm x  b , vô lý vì P ( x ) là đa thức bậc
nhỏ nhất thỏa mãn điều này.

a  ap  q  b
2

Nếu Q (b) a thì đặt Q ( x )  x 2  px  q, ta có hệ  2 .


b  bp  q  a

Trừ từng vế, ta được ( a  b)( a  b  p )  b  a nên a  b  p  1  c  p  1, vô lý vì c vô tỷ


còn p  1 nguyên. Do đó Q (b)  c .

Chứng minh tương tự ta có Q ( c )  a nên Q (a )  Q (b)  Q (c )  a  b  c  0.

Nhận xét. Bài toán là một ứng dụng thú vị của “đa thức tối tiểu”, là đa thức nguyên bậc nhỏ nhất
nhận một số vô tỷ là nghiệm. Bài toán gốc được lấy từ đề China TST với nội dung câu hỏi như sau
(giả thiết tương tự): Chứng minh rằng p 2  2 p  4q  7 là số chính phương.

a b2 pb q
Ta xử lý tiếp như sau: từ hệ phương trình b c 2
pc q (*)
c a2 pa q

Trừ từng vế hai phương trình đầu của (*) , ta có

a  b  b 2  c 2  p (b  c)  (b  c)(b  c  p )  (b  c)( p  a ) .

Tương tự thì b  c  (c  a )( p  b) và c  a  (a  b)( p  c) nên nhân tất cả các hệ thức lại thì

( p  a )( p  b)( p  c)  1.

3
Hướng tới kỳ thi chọn HSG quốc gia 2019-2020

3
Cộng các đẳng thức trong (*) lại, ta được a 2  b 2  c 2  3q  0  ab  bc  ca  q.
2
Nhân phương trình thứ 1, 2,3 của (*) cho b, c, a rồi cộng lại, ta có

3q q
ab  bc  ca  a 3  b3  c 3  p (a 2  b2  c 2 )   3abc  3 pq  abc   pq .
2 2
Thay tất cả vào đẳng thức ( p  a )( p  b)( p  c)  1, ta được

3 pq q
p 3  p 2 (a  b  c )  p (ab  bc  ca )  abc  1  p 3    pq  1
2 2
p 1

 q  2( p  p  1)
2

(1) Nếu q  2( p 2  p  1) thì T  p 2  2 p  8( p 2  p  1)  7  (3 p  1) 2 là số chính phương.

a b2 b q
3
(2) Nếu p  1 , khi đó abc q và b c2 c q nên
2
c a2 a q

3q
ab bc ca (a 2 a q)(b 2 b q)
2
a 2b 2 b 2 c 2 c 2 a 2 3abc ab bc ca 2q(a 2 b2 c2 ) 3q 2
9q 2 9q 3q 3q 2
2q( 3q) 3q 2 3q
4 2 2 4
Do đó, ta có q 0 hoặc q 6. Rõ ràng q 0 không thỏa vì khi đó abc 0 nên phải có một
số bằng 0. Còn nếu q 6, p 1 thì T 16 , cũng là số chính phương.

Bài 3. Cho tam giác ABC nhọn, không cân nội tiếp trong đường tròn (O ) với trực tâm H . Điểm
R thay đổi trên cung lớn BC của (O ) sao cho AR không song song với BC . Lấy các điểm S , T
trên đường thẳng BC sao cho ( ARS ),( ART ) cùng tiếp xúc với BC. Đường thẳng qua H , vuông
góc với AS , AT lần lượt cắt ( HBC ) ở X , Y .

a) Chứng minh rằng đường thẳng XY luôn đi qua điểm cố định.


b) Chứng minh rằng tâm của đường tròn ( RST ) di chuyển trên đường thẳng cố định.

Lời giải.
a) Gọi K là giao điểm của AR, BC . Theo phương tích thì

KB  KC  KA  KR  KS 2  KT 2

4
Hướng tới kỳ thi chọn HSG quốc gia 2019-2020

nên theo hệ thức Newton thì ( BC , RS )  1.

Vì chùm A( ST , BC )  1 nên trực giao đỉnh H với chú ý BH  AC , CH  AB, ta có

H ( MN , BC )  A( ST , CB )  1.

Suy ra tứ giác BMCN điều hòa và MN sẽ đi qua giao điểm hai tiếp tuyến của ( HBC ) ở B, C .
Rõ ràng đó chính là điểm cố định.

R E O N
H

C
S K B T D

b) Bằng biến đổi góc, ta có


SRT  SRK  TRK  ASK  ATK  180  SAT .

Suy ra hai đường tròn ( RST ),( AST ) đối xứng nhau qua đường thẳng BC. Khi đó, để chứng minh
tâm của ( RST ) thuộc đường thẳng cố định, ta đưa về chứng minh cho tâm của ( AST ) .

Gọi D là trung điểm BC và E là giao điểm của AD với ( AST ) thì theo hệ thức Newton thì

DA  DE  DT  DS  DB 2  DC 2
nên E là điểm cố định. Do đó, đường tròn ( AST ) đi qua điểm cố định A, E nên tâm của nó sẽ
di chuyển trên trung trực của AE cũng cố định. Ta có đpcm.
Nhận xét. Các điểm R, E ở trên lần lượt chính là điểm Humpty của các tam giác AST , ABC .
Khai thác tính chất của các điểm này, ta sẽ còn nhiều kết quả thú vị.

5
Hướng tới kỳ thi chọn HSG quốc gia 2019-2020

Bài 4. Tìm tất cả các hàm số f : thỏa mãn điều kiện

f (x 3 f ( y )) f (x f ( y) y3) f (4 f ( y ) y 3 ) 1 với mọi x, y .

Lời giải.

Thay x  3 f ( y ) vào phương trình đề cho, ta có f (0)  2 f (4 f ( y )  y 3 )  1, hay

f (0)  1
f (4 f ( y )  y 3 )   a  1, y 
2
f (0)  1
trong đó a  . Thay vào phương trình đề bài, ta có
2

f ( x  3 f ( y ))  f ( x  f ( y )  y 3 )  a với mọi x, y  .

Thay y  0, x  x  f ( y )  y 3  f (0), ta có

f ( x  f ( y )  y 3  f (0)  3 f ( y ))  f ( x  f ( y )  y 3 )  a , x, y 

hay tương đương f ( x  2 f ( y )  y 3  f (0))  f ( x  f ( y )  y 3 )  a, x, y  .

So sánh hai phương trình trên ta suy ra

f ( x  3 f ( y ))  f ( x  2 f ( y )  y 3  f (0)), x, y  .

Thay x  x  3 f ( y ), ta suy ra f ( x )  f ( x  f ( y )  y 3  f (0)), x, y  .

Nếu f ( y )  y 3  f (0)  0, y thì thử lại thấy f ( y )   y 3  c không là nghiệm của phương trình.
Ngược lại, nếu tồn tại y 0 thỏa f ( y0 )  y03  f (0)  c  0 thay vào phương trình trên, ta suy ra
f ( x )  f ( x  c ), x  . Khi đó thay y  y  c vào đẳng thức trên, ta có

f ( x )  f ( x  f ( y )  ( y  c )3  f (0))  f ( x  f ( y )  y 3  f (0), x, y  .

Thay x  x  f ( y )  y 3  f (0) ta lại có

f ( x )  f ( x  ( y  c )3  y 3 ), x, y  . .

Nếu c  0 thì ( y  c )3  y 3 đa thức bậc 2 có hệ số bậc cao nhất dương nên nhận giá trị trong tập
[m;  ) với m là một số thực nào đó. Do đó f ( x )  f ( x  r ), x  [m; ).

Lấy x, y  bất kỳ mà x  y . Khi đó tồn tại q, r  [m;  ) thỏa mãn q  r  x  y . Ta có

f ( y )  f ( y  q)  f ( y  q  r )  f ( y  x  y )  f ( x ) .

Điều này chứng tỏ f là hàm hằng. Tương tự nếu c  0 ta cũng suy ra f hằng. Thay vào đề bài,
ta có f ( x )  1, x thỏa mãn. Vậy tất cả các hàm số cần tìm là f ( x )  1, x  .

6
Hướng tới kỳ thi chọn HSG quốc gia 2019-2020

Nhận xét.
Bài toán trên đây được phát triển từ bài toán

f (x f ( y )) f (x y 2n ) f ( f ( y) y 2 n ) 1 , với mọi x, y .

Ở trong bài toán này, sau một vài bước xử lí, ta thu được một trường hợp f là hàm tuần hoàn
với chu kì c. Từ đó thay vào phương trình đề cho, ta sẽ có

f ( x) f (x (y c)2n y 2 n ), với mọi x, y .

Đến đây, do c 0 nên ( y c)2n y 2 n là đa thức bậc 2n 1 theo y , tức sẽ phủ hết các giá trị
của tập . Từ đó ta có
f ( x) f (x r ), với mọi x , r .

Điều này chứng tỏ f là hàm hằng. Tuy nhiên, ở đây, đa thức ( y c )3 y 3 là đa thức bậc hai nên
không thể phủ hết giá trị của tập mà thay vào đó là [m, ) với m nào đó (nếu c 0 ) và
( , m ] (nếu c 0). Điều này đòi hỏi cách xử lí khéo léo hơn. Một trong những cách đó là: lấy
x y bất kì, ta chứng minh f ( x ) f ( y ). Thật vậy, vì f ( x ) f ( x r ) với mọi
x , r [ m, ) nên chọn q m x y.

Bài 5. Cho tập hợp S gồm n số square-free lớn hơn 1 có tích bằng m là một số nguyên dương
có đúng 13 ước nguyên tố phân biệt. Biết rằng bất kỳ 5 số nào trong S cũng không có ước nguyên
tố chung và tích 2 số bất kỳ trong S thì không là số square-free.
a) Chứng minh rằng n 13.

b) Chứng minh rằng khi n 13 thì m là số chính phương và mỗi số trong S có đúng 16 ước
nguyên dương.
Lời giải.
a) Gọi P là tập hợp các ước nguyên tố của m. Theo giả thiết, ta có các nhận xét:
- Mỗi số nguyên tố thuộc P sẽ là ước của không quá 4 số thuộc S .
- Vì tích của hai số square-free thuộc S không phải là số square-free nên chúng phải có ước nguyên
tố chung.

Từ đó, đếm số bộ {a, b}, p  mà a , b  S , p  P và p là ước chung của a , b.

 Cách 1. Đếm theo {a , b}, ta có  Cn2  1 .


 Cách 2. Đếm theo p , ta có  13  C42  78.

Do đó Cn2  78  n(n  1)  156 nên n  13.

7
Hướng tới kỳ thi chọn HSG quốc gia 2019-2020

b) Khi n  13, dễ thấy rằng khi đẳng thức xảy ra mỗi số nguyên tố là ước của đúng 4 số nên m
là một lũy thừa bậc 4 của tích các số nguyên tố, nói cách khác, nó là số chính phương. Ngoài ra,
hai số bất kỳ trong S đều phải có ước nguyên tố chung.
Cuối cùng nếu có số a  S nào đó chỉ có  3 ước nguyên tố thì mỗi ước đó sẽ thuộc về đúng 3
số nữa. Khi đó, còn lại 13  1  3  3  3 số sẽ không có ước chung với a , mâu thuẫn.

Suy ra mỗi số trong S đều phải có ít nhất 4 ước nguyên tố.


Đến đây, đếm số bộ ( a , p ) với a  S , p  P và p | a , ta thấy rằng nếu đếm theo a thì sẽ có
 13  4  52, còn đếm theo p thì sẽ có đúng 13  4  52 nên đẳng thức phải xảy ra, tức là mỗi số
trong S đều có đúng 4 ước nguyên tố, suy ra số ước của nó đúng bằng 2 4  16.
Nhận xét. Bài toán trên chính là một trường hợp đặc biệt của bài toán tổng quát thú vị sau:
Một CLB có n  1 học sinh được phân hoạch thành m  1 nhóm nhỏ thỏa mãn

 Mỗi nhóm có số thành viên bằng nhau.


 Hai nhóm tùy ý có đúng 1 thành viên chung.
 Hai thành viên tùy ý tham gia chung đúng 1 nhóm.
Chứng minh rằng 2( m  n)  3 là số chính phương.

Lời giải. Trước hết, đếm số bộ (học sinh, học sinh, nhóm) mà hai học sinh cùng tham gia vào
nhóm, ta có Cn2  mCk2  n(n  1)  k (k  1)m.

Xét một học sinh A tùy ý và gọi s A là số nhóm mà A tham gia, giả sử trong đó có nhóm 1 . Ta sẽ
đếm số bộ (học sinh, nhóm) mà học sinh khác A , nhóm khác 1 và học sinh này cùng với A thuộc
vào nhóm đó. Đếm theo học sinh, có n  k cách; còn đếm theo nhóm, có ( s A  1)(k  1) cách.

nk n 1
Từ đó suy ra s A  1   sA  . Do giá trị này cố định nên các học sinh đều tham gia vào
k 1 k 1
n 1
đúng nhóm.
k 1
* Đếm số bộ (nhóm, nhóm, học sinh) mà hai nhóm có học sinh tham gia chung, ta có

Cm2  nC n21 hay ( k  1)( m  1)  k ( n  k ) .


k 1

Giải hệ, ta có m  n  k 2  k  1 nên 2(m  n)  3  4(k 2  k  1)  3  (2k  1) 2 là số chính phương.

Khi m  k 2  k  1, cấu trúc đẹp trên bị “phá vỡ”, ta đưa về mô hình định lý sunflower: Xét một
tập hợp n phần tử và m tập con của nó, mỗi tập có k phần tử sao cho hai tập bất kỳ đều có đúng
một phần tử chung. Khi đó nếu như m  k 2  k  1 thì sẽ có 1 phần tử thuộc về tất cả các tập con.
Kết quả này chứng minh không khó bằng nguyên lý Dirichlet.

You might also like